week 4,5,6 patho/pharm

¡Supera tus tareas y exámenes ahora con Quizwiz!

A nurse is teaching about the area of the spinal cord that contains cell bodies involved in the autonomic nervous system. Which of the following area is the nurse discussing? A. Anterior horn B. Ventral horn C. Lateral horn D. Dorsal horn

C. Lateral horn

When a patient asks what causes hyperglycemia in type 2 DM, how should the nurse respond? Hyperglycemia is a result of: a. Insulin deficiency b. Hyperinsulinemia c. Glucagon deficiency d. Liver dysfunction

b

When a staff member asks the nurse what causes the chronic complications of DM such as microvascular and macrovascular disease, how should the nurse respond? These complications are primarily related to: a. Pancreatic changes b. Hyperglycemia c. Ketone toxicity d. Hyperinsulinemia

b

Which statement is appropriate for the nurse to include in patient teaching regarding type 2 diabetes? A. "Insulin injections are never used with type 2 diabetes." B. "You don't need to measure your blood glucose levels because you are not taking insulin injections." C. "A person with type 2 diabetes still has functioning beta cells in his or her pancreas." D. "Patients with type 2 diabetes usually have better control over their diabetes than those with type 1 diabetes."

C. "A person with type 2 diabetes still has functioning beta cells in his or her pancreas."

An obstruction of the anterior cerebral arteries affects A. visual imaging. B. balance and coordination. C. judgment, insight, and reasoning. D. visual and auditory integration for language comprehension.

C The anterior cerebral artery feeds the medial and anterior portions of the frontal lobes. The anterior portion of the frontal lobe controls higher-order processes such as judgment and reasoning.

Which action will occur when a patients -a1 receptors are stimulated? a. Dilation of the coronary arteries b. Vasoconstriction of arteries c. Increase in the strength of myocardial contraction d. Decrease in the rate myocardial contraction

C. Increase in the strength of myocardial contraction

The nurse is assessing the patient with a pen light. The integrity of which cranial nerve is being evaluated? a. Olfactory b. Vagus c. Oculomotor d. Trigeminal

C. Oculomotor

Treatment of medication overuse headache is A. use of an ice cap. B. skeletal muscle relaxants. C. elimination of caffeinated beverages. D. abrupt withdrawal of all nonopioid medications.

D Medication overuse headache (MOH) is the term used to describe an analgesic rebound headache. Drugs known to cause this problem are acetaminophen, aspirin, NSAIDs (e.g., ibuprofen), butalbital, sumatriptan, and opioids. Treatment involves abrupt withdrawal of the offending drug (except for opioids, which need to be tapered) and initiation of alternative drugs such as amitriptyline.

What is the most effective measure for rabies after a patient has sustained a bite by an animal thought to be infected? A. Antibiotic administration B. Contact isolation C. Wound irrigation D. Rapid postexposure prophylaxis

D Rabies usually is fatal. Management efforts are directed at preventing the transmission and onset of the disease, and postexposure prophylaxis is administered. It is more effective than the other measures.

Which information indicates a correct understanding of viral vaccines? Most viral vaccines contain: a. active viruses. b. attenuated viruses. c. killed viruses. d. viral toxins.

ANS: B Viral vaccines contain live viruses that are weakened (attenuated). Viral vaccines do not contain active viruses, killed viruses, or toxins.

18. The nurse is caring for a patient who is human immunodeficiency virus (HIV) positive and taking antiretroviral therapy (ART). Which information is most important for the nurse to address when planning care? a. The patient complains of feeling "constantly tired." b. The patient can't explain the effects of indinavir (Crixivan). c. The patient reports missing some doses of zidovudine (AZT). d. The patient reports having no side effects from the medications.

ANS: C Because missing doses of ART can lead to drug resistance, this patient statement indicates the need for interventions such as teaching or changes in the drug scheduling. Fatigue is a common side effect of ART. The nurse should discuss medication actions and side effects with the patient, but this is not as important as addressing the skipped doses of AZT. DIF: Cognitive Level: Analyze (analysis) REF: 228 OBJ: Special Questions: Prioritization TOP: Nursing Process: Planning MSC:

17. In a type III hypersensitivity reaction, the harmful effects after the immune complexes that are deposited in tissues are a result of what? a. Cytotoxic T cells b. Natural killer cells c. Complement activation d. Degranulation of mast cells

ANS: C Complement activation, particularly through the generation of chemotactic factors for neutrophils, causes the harmful effects of immune complex deposition. The neutrophils bind to antibody and C3b contained in the complexes and attempt to ingest the immune complexes. Type III hypersensitivity reactions as described are not the result of cytotoxic T cells, natural killer cells, or degranulation of mast cells.

Which are modes of heat loss in the newborn? Select all that apply. 1. Radiation 2. Urination 3. Convection 4. Conduction 5. Evaporation

1, 3, 4, 5

A client is receiving total parenteral nutrition. The nurse assesses for which client response that indicates hyperglycemia? 1.Polyuria 2.Paralytic ileus 3.Respiratory rate below 16 4.Serum glucose of 105 mg/100 mL

1.Polyuria

12. What is the most appropriate nursing intervention to help an HIV-infected patient adhere to a treatment regimen? a. "Set up" a drug pillbox for the patient every week. b. Give the patient a video and a brochure to view and read at home. c. Tell the patient that the side effects of the drugs are bad but that they go away after a while. d. Assess the patient's routines and find adherence cues that fit into the patient's life circumstances.

12. Correct answer: d Rationale: The best approach to improve adherence to a treatment regimen is to learn about the patient's life and assist with problem solving within the confines of that life.

A client treated for an episode of hyperthermia is being discharged to home. The nurse determines that the client needs clarification of discharge instructions if the client states a need to perform which action? 1. Increase fluid intake. 2. Resume full activity level. 3. Stay in a cool environment when possible. 4. Monitor voiding for adequacy of urine output.

2

Four hours after surgery the blood glucose level of a client who has type 1 diabetes is elevated. The nurse can expect to: 1.Administer an oral hypoglycemic 2.Institute urine glucose monitoring 3.Give supplemental doses of regular insulin 4.Decrease the rate of the intravenous infusion

3.Give supplemental doses of regular insulin

A male client who is receiving prolonged steroid therapy complains of always being thirsty and urinating frequently. What is the nurse's best initial action? 1.Have the client assessed for an enlarged prostate. 2.Obtain a urine specimen from the client to test for ketonuria. 3.Perform a finger stick to test the client's blood glucose level. 4.Assess the client's lower extremities for the presence of pitting edema

3.Perform a finger stick to test the client's blood glucose level.

A client with diabetes is being taught to self-administer a subcutaneous injection of insulin. Identify the preferred site for the self-administration of this drug. 1.upper arm 2.buttocks 3.abdomen 4.thigh

3.abdomen

A nurse is monitoring a client's laboratory results for a fasting plasma glucose level. Within which range of a fasting plasma glucose level does the nurse conclude that a client is considered to be diabetic? 1.40 and 60 mg/dL 2.80 and 99 mg/dL 3.100 and 125 mg/dL 4.126 and 140 mg/dL

4.126 and 140 mg/dL

Normal ICP ranges from:

5 to 15 mm Hg

You are alerted to a possible acute subdural hematoma in the patient who A. has a linear skull fracture crossing a major artery. B. has focal symptoms of brain damage with no recollection of a head injury. C. develops decreasing LOC and a headache within 48 hours of a head injury. D. has an immediate loss of consciousness with a brief lucid interval followed by decreasing LOC.

An acute subdural hematoma manifests within 24 to 48 hours of the injury. The signs and symptoms are similar to those associated with brain tissue compression by increased intracranial pressure (ICP) and include decreasing LOC and headache.

True or false? A vaccination is an example of passive immunity.

False

When a student asks which type of nerves transmit nerve impulses at the fastest rate, what is the best response by the nurse?

Large, myelinated

A neurologist is teaching about the location of the primary visual cortex in the brain. Which area is the neurologist discussing? a. Frontal lobe b. Temporal lobe c. Occipital lobe d. Parietal lobe

Occipital Lobe

Which factor related to cerebral blood flow most often determines the extent of cerebral damage from a stroke? A. Amount of cardiac output B. Oxygen content of the blood C. Degree of collateral circulation D. Level of carbon dioxide in the blood

The extent of the stroke depends on rapidity of onset, the size of the lesion, and the presence of collateral circulation.

A nurse is reviewing lab results for glycosylated hemoglobin (hemoglobin A1c) levels. A nurse recalls the purpose of this test is to: a. Measure fasting glucose levels. b. Monitor long-term serum glucose control. c. Detect acute complications of diabetes. d. Check for hyperlipidemia.

b

Diabetes insipidus, diabetes mellitus (DM), and SIADH share which of the following assessment manifestations? a. Polyuria b. Edema c. Vomiting and abdominal cramping d. Thirst

d

31. What is Graves disease a result of? a. Increased levels of circulating immunoglobulins b. The infiltration of the thyroid with T lymphocytes c. Autoantibodies binding to thyroid-stimulating hormone (TSH)-receptor sites d. Exposure to acetylates in substances such as rubber

ANS: C In the hyperthyroidism (excessive thyroid activity) of Graves disease, autoantibody binds to and activates receptors for TSH (a pituitary hormone that controls the production of the hormone thyroxine by the thyroid). Graves disease is not caused by increased levels of circulating immunoglobulins, infiltration of the thyroid by T lymphocytes, or by exposure to acetylates.

26. A nurse recalls insulin has an effect on which of the following groups of electrolytes? a. Sodium, chloride, phosphate b. Calcium, magnesium, potassium c. Hydrogen, bicarbonate, chloride d. Potassium, magnesium, phosphate

ANS: D Insulin facilitates the intracellular transport of potassium (K+), phosphate, and magnesium.

2. A nurse is teaching staff about protein hormones. Which information should the nurse include? One of the protein hormones is: a. Thyroxine (T4) b. Aldosterone c. Testosterone d.Insulin

ANS: D Protein hormones are also water-soluble hormones, and insulin is a part of this group. Thyroxine is a lipid soluble hormone and is not a protein hormone. Aldosterone is a lipid soluble hormone and is not a protein hormone. Testosterone is a lipid soluble hormone and is not a protein hormone.

24. A person with type O blood needs a blood transfusion. What blood type does the healthcare professional prepare to administer to the patient? a. A b. B c. AB d. O

ANS: D Type O individuals have neither A or B antigen but have both anti-A and anti-B antibodies and therefore cannot accept blood from any of the other three types. The healthcare professional would prepare to administer a transfusion of type O blood.

A patient with ICP monitoring has pressure of 12 mm Hg. The nurse understand that this pressure reflects a. a severe decrease in cerebral perfusion pressure b. an alteration in the production of CSF c. the loss of autoregulatory control of ICP d. a normal balance between brain tissue, blood, and CSF

D. A normal balance between brain tissue, blood, and CSF- normal is 10- 15 mm Hg

The nurse is performing an assessment on a client with a diagnosis of myxedema (hypothyroidism). Which assessment finding should the nurse expect to note in this client? 1. Dry skin 2. Thin, silky hair 3. Bulging eyeballs 4. Fine muscle tremors

1

You would expect to find which clinical manifestation in a patient admitted with a left-brain stroke? A. Impulsivity B. Impaired speech C. Left-sided neglect D. Short attention span

B Clinical manifestations of left-sided brain damage include right hemiplegia, impaired speech and language aphasias, impaired right and left discrimination, and slow and cautious performance. The other options are all manifestations of right-sided brain damage.

What would you include in your teaching about a dietary trigger of a cluster headache? A. Aged cheese B. Alcohol C. Caffeine D. Grapefruit

B Alcohol is the only dietary trigger of cluster headaches.

A 50-year-old female experiences decreased blood pressure, decreased oxygen delivery, cardiovascular shock, and subsequent death. A complication of endotoxic shock is suspected. Which of the following is the most likely cause? a. Gram-positive bacteria b. Fungi c. Gram-negative bacteria d. Virus

ANS: C Symptoms of gram-negative septic shock are produced by endotoxins. Once in the blood, endotoxins cause the release of vasoactive peptides and cytokines that affect blood vessels, producing vasodilation, which reduces blood pressure, causes decreased oxygen delivery, and produces subsequent cardiovascular shock. Gram-positive bacteria nor fungi do not produce endotoxins and thus do not manifest in shock. Viruses do not produce symptoms of shock.

12. A patient who uses injectable illegal drugs asks the nurse about preventing acquired immunodeficiency syndrome (AIDS). Which response by the nurse is best? a. "Clean drug injection equipment before each use." b. "Ask those who share equipment to be tested for HIV." c. "Consider participating in a needle-exchange program." d. "Avoid sexual intercourse when using injectable drugs."

ANS: C Participation in needle-exchange programs has been shown to decrease and control the rate of HIV infection. Cleaning drug equipment before use also reduces risk, but it might not be consistently practiced. HIV antibodies do not appear for several weeks to months after exposure, so testing drug users would not be very effective in reducing risk for HIV exposure. It is difficult to make appropriate decisions about sexual activity when under the influence of drugs. DIF: Cognitive Level: Analyze (analysis) REF: 226 TOP: Nursing Process: Implementation MSC:

6. Which patient exposure by the nurse is most likely to require postexposure prophylaxis when the patient's human immunodeficiency virus (HIV) status is unknown? a. Needle stick injury with a suture needle during a surgery b. Splash into the eyes while emptying a bedpan containing stool c. Needle stick with a needle and syringe used for a venipuncture d. Contamination of open skin lesions with patient vaginal secretions

ANS: C Puncture wounds are the most common means for workplace transmission of blood-borne diseases, and a needle with a hollow bore that had been contaminated with the patient's blood would be a high-risk situation. The other situations described would be much less likely to result in transmission of the virus. DIF: Cognitive Level: Analyze (analysis) REF: 219 TOP: Nursing Process: Implementation MSC: NCLEX:

23. The nurse designs a program to decrease the incidence of human immunodeficiency virus (HIV) infection in the adolescent and young adult populations. Which information should the nurse assign as the highest priority? a. Methods to prevent perinatal HIV transmission b. Ways to sterilize needles used by injectable drug users c. Prevention of HIV transmission between sexual partners d. Means to prevent transmission through blood transfusions

ANS: C Sexual transmission is the most common way that HIV is transmitted. The nurse should also provide teaching about perinatal transmission, needle sterilization, and blood transfusion, but the rate of HIV infection associated with these situations is lower. DIF: Cognitive Level: Analyze (analysis) REF: 218 OBJ: Special Questions: Prioritization TOP: Nursing Process: Planning MSC:

16. A type IV hypersensitivity reaction causes which result? a. Antibodies coating mast cells by binding to receptors that signal its degranulation, followed by the discharge of preformed mediators b. Antibodies binding to soluble antigens that were released into body fluids and the immune complexes being deposited in the tissues c. Lymphokine-producing Th1 cells directly attacking and destroying cellular targets d. Antibodies binding to the antigen on the cell surface

ANS: C Type I, II, and III hypersensitivity reactions are mediated by antibody, type IV reactions are mediated by T lymphocytes and do not involve antibody. Type IV mechanisms occur through either Tc cells or lymphokine-producing Th1 cells. Tc cells directly attack and destroy cellular targets.

The nurse associates which assessment finding in the diabetic patient with decreasing renal function? a. Ketone bodies in the urine during acidosis b. Glucose in the urine during hyperglycemia c. Protein in the urine during a random urinalysis d. White blood cells in the urine during a random urinalysis

ANS: C Urine should not contain protein. Proteinuria in a diabetic heralds the beginning of renal insufficiency or diabetic nephropathy with subsequent progression to end stage renal disease. Chronic elevated blood glucose levels can cause renal hypertension and excess kidney perfusion with leakage from the renal vasculature. This leaking allows protein to be filtered into the urine.

After studying about viruses, which information indicates the student has a good understanding of viruses? Viruses: a. contain no DNA or RNA. b. are capable of independent reproduction. c. replicate their genetic material inside host cells. d. are easily killed by antimicrobials.

ANS: C Virus replication depends totally on the ability of the virus to infect a permissive host cell, a cell that cannot resist viral invasion and replication. Viruses contain both DNA and RNA, are incapable of independent reproduction, and cannot be killed by antimicrobials.

When teaching about hypoglycemia, the nurse will make sure that the patient is aware of the early signs of hypoglycemia, including: a. hypothermia and seizures. b. nausea and diarrhea. c. confusion and sweating. d. fruity, acetone odor to the breath.

ANS: C Early symptoms of hypoglycemia include the central nervous system manifestations of confusion, irritability, tremor, and sweating. Hypothermia and seizures are later symptoms of hypoglycemia. The other options are incorrect. DIF: COGNITIVE LEVEL: Understanding (Comprehension) REF: p. 514 TOP: NURSING PROCESS: Implementation MSC: NCLEX: Health Promotion and Maintenance

When caring for a pregnant patient with gestational diabetes, the nurse should question a prescription for which drug? A. Insulin glargine (Lantus) B. Glipizide (Glucotrol) C. Insulin glulisine (Apidra) D. NPH insulin

B. Glipizide (Glucotrol) Correct Oral antidiabetic drugs are classified as pregnancy B or C drugs and are generally not recommended for pregnant patients.

A patient with type 2 diabetes is scheduled for a follow-up visit in the clinic several months from now. Which test will the nurse schedule to evaluate the effectiveness of treatment for the patient? a. Fasting blood glucose b. Glycosylated hemoglobin c. Oral glucose tolerance test d. Urine dipstick for glucose and ketones

B. Glycosylated hemoglobin

The nurse makes a home-care visit to a client with Bell's palsy. Which of statement by the client requires clarification by the nurse? A. I wear an eye patch at night B. I am staying on a liquid diet C. I wear dark glasses when I go out D. I have been gently massaging my face.

B. It is not necessary for a client with Bell's palsy to stay on a liquid diet. The client should be encouraged to chew on the unaffected side. Options A, C, and D identify accurate statements related to the management of Bell's palsy.

Vasogenic cerebral edema increases ICP by A. shifting fluid in the gray matter. B. altering the endothelial lining of cerebral capillaries. C. leaking molecules from the intracellular fluid to the capillaries. D. altering the osmotic gradient flow into the intravascular component.

B Vasogenic cerebral edema occurs mainly in the white matter and is caused by changes in the endothelial lining of cerebral capillaries.

A priority goal for the patient with restless legs syndrome is to A. increase exercise and activity throughout the day time. B. improve sleep quality. C. promote weight loss. D. increase nighttime consumption warm beverages.

B he goal of collaborative management is to reduce patient discomfort and distress and to improve sleep quality. Reference: 1514

Which nursing diagnosis is more appropriate for a patient with advanced Parkinson's disease? A. Urinary urge incontinence related to effects of drug therapy B. Ineffective breathing pattern related to diaphragm fatigue C. Risk for aspiration related to impaired swallowing D. Risk for injury related to limited vision

C

Which laboratory value reported by the unlicensed assistive personnel (UAP) indicates an urgent need for the nurse to assess the patient? a. Bedtime glucose of 140 mg/dL b. Noon blood glucose of 52 mg/dL c. Fasting blood glucose of 130 mg/dL d. 2-hr postprandial glucose of 220 mg/dL

B. Noon blood glucose of 52 mg/dL

The nurse is assessing a client who has a diagnosis of goiter. Which should the nurse expect to note during the assessment of the client? 1. An enlarged thyroid gland 2. The presence of heart damage 3. Client complaints of chronic fatigue 4. Client complaints of slow wound healing

1

When monitoring a patient's response to oral antidiabetic drugs, the nurse knows that which laboratory result would indicate a therapeutic response? a. Random blood glucose level 180 mg/dL b. Blood glucose level of 50 mg/dL after meals c. Fasting blood glucose level between 92 mg/dL d. Evening blood glucose level below 80 mg/dL

ANS: C The American Diabetes Association recommends a fasting blood glucose level of between 80 and 130 mg/dL for diabetic patients. The other options are incorrect. DIF: COGNITIVE LEVEL: Applying (Application) REF: p. 504 TOP: NURSING PROCESS: Evaluation MSC: NCLEX: Physiological Integrity: Physiological Adaptation

A patient who has type 2 diabetes is scheduled for an oral endoscopy and has been NPO (nothing by mouth) since midnight. What is the best action by the nurse regarding the administration of her oral antidiabetic drugs? a. Administer half the original dose. b. Withhold all medications as ordered. c. Contact the prescriber for further orders. d. Give the medication with a sip of water.

ANS: C When the diabetic patient is NPO, the prescriber needs to be contacted for further orders regarding the administration of the oral antidiabetic drugs. The other options are incorrect. DIF: COGNITIVE LEVEL: Applying (Application) REF: p. 518 TOP: NURSING PROCESS: Implementation MSC: NCLEX: Physiological Integrity: Pharmacological and Parenteral Therapies

5. The nurse is providing care for a patient who has been living with human immunodeficiency virus (HIV) for several years. Which assessment finding most clearly indicates an acute exacerbation of the disease? A. A new onset of polycythemia B. Presence of mononucleosis-like symptoms C. A sharp decrease in the patient's CD4+ count D. A sudden increase in the patient's WBC count

C A decrease in CD4+ count signals an exacerbation of the severity of HIV. Polycythemia is not characteristic of the course of HIV. A patient's WBC count is very unlikely to suddenly increase, with decreases being typical. Mononucleosis-like symptoms such as malaise, headache, and fatigue are typical of early HIV infection and seroconversion.

16. The nurse is monitoring the effectiveness of antiretroviral therapy (ART) for a patient with acquired immunodeficiency syndrome (AIDS). What laboratory study result indicates the medications are effective? A. Increased viral load B. Decreased neutrophil count C. Increased CD4+ T cell count D. Decreased white blood cell count

C Antiretroviral therapy is effective if there are decreased viral loads and increased CD4+ T cell counts.

The nurse recognizes which patient as having the greatest risk for undiagnosed diabetes mellitus? a. Young white man b. Middle-aged African-American man c. Young African-American woman d. Middle-aged Native American woman

ANS: D The highest incidence of diabetes in the United States occurs in Native Americans. With age, the incidence of diabetes increases in all races and ethnic groups.

14. A patient with human immunodeficiency virus (HIV) infection has developed Mycobacterium avium complex infection. Which outcome would be appropriate for the nurse to include in the plan of care? a. The patient will be free from injury. b. The patient will receive immunizations. c. The patient will have adequate oxygenation. d. The patient will maintain intact perineal skin.

ANS: D The major manifestation of M. avium infection is loose, watery stools, which would increase the risk for perineal skin breakdown. The other outcomes would be appropriate for other complications (e.g., pneumonia, dementia, influenza) associated with HIV infection. DIF: Cognitive Level: Apply (application) REF: 222 TOP: Nursing Process: Planning MSC:

16. The nurse cares for a patient who is human immunodeficiency virus (HIV) positive and taking antiretroviral therapy (ART). Which information is most important for the nurse to address when planning care? a. The patient's blood glucose level is 142 mg/dL. b. The patient complains of feeling "constantly tired." c. The patient is unable to state the side effects of the medications. d. The patient states, "Sometimes I miss a dose of zidovudine (AZT).

ANS: D Because missing doses of ART can lead to drug resistance, this patient statement indicates the need for interventions such as teaching or changes in the drug scheduling. Elevated blood glucose and fatigue are common side effects of ART. The nurse should discuss medication side effects with the patient, but this is not as important as addressing the skipped doses of AZT.

34. When catecholamines are released in a patient, what should the nurse assess for? a. Nutrient absorption b. Fluid retention c. Hypotension d. Hyperglycemia

ANS: D Catecholamines cause hyperglycemia and immune suppression.

10. When the endocrinologist asks the staff how the releasing hormones that are made in the hypothalamus travel to the anterior pituitary, how should the staff reply? Via the: a. Vessels of the zona fasciculata b. Chromophils c. Median eminence d. Hypophysial portal system

ANS: D Neurons in the hypothalamus secrete releasing hormones into veins that carry the releasing hormones directly to the vessels of the adenohypophysis via the hypophysial portal system, thus bypassing the normal circulatory route. Zona fasciculata secretes abundant amounts of cortisol from the adrenal gland. Chromophils are the secretory cells of the adenohypophysis. The median eminence is a part of the posterior pituitary, not the anterior.

A nurse recalls bacteria become resistant to antimicrobials by: a. proliferation. b. attenuation. c. specialization. d. plasmid exchange.

ANS: D Antibiotic resistance is usually a result of genetic mutations that can be transmitted directly to neighboring microorganisms by plasmid exchange. Antibiotic resistance is not a result of proliferation, attenuation, or specialization.

9. The nurse will most likely prepare a medication teaching plan about antiretroviral therapy (ART) for which patient? a. Patient who is currently HIV negative but has unprotected sex with multiple partners b. Patient who was infected with HIV 15 years ago and now has a CD4+ count of 840/µL c. HIV-positive patient with a CD4+ count of 160/µL who drinks a fifth of whiskey daily d. Patient who tested positive for HIV 2 years ago and now has cytomegalovirus (CMV) retinitis

ANS: D CMV retinitis is an AIDS-defining illness and indicates that the patient is appropriate for ART even though the HIV infection period is relatively short. An HIV-negative patient would not be offered ART. A patient with a CD4+ count in the normal range would not typically be started on ART. A patient who drinks alcohol heavily would be unlikely to be able to manage the complex drug regimen and would not be appropriate for ART despite the low CD4+ count. DIF: Cognitive Level: Analyze (analysis) REF: 221 OBJ: Special Questions: Multiple Patients TOP: Nursing Process: Planning MSC:

when the nurse is teaching a pt who is taking acyclovir for genital herpes, which statement by the nurse is accurate? 1. this drug will help the lesions to dry and crust over 2. acyclovir will eradicate the herpes virus 3. this drug will prevent the spread of this virus to others 4. be sure to give this drug to your partner too

1. this drug will help the lesions to dry and crust over

A client is admitted to the hospital with diabetic ketoacidosis. The nurse concludes that the client's elevated ketone level is caused by incomplete oxidation of which nutrient? 1.Fats 2.Protein 3.Potassium 4.Carbohydrates

1.Fats

Levothyroxine (Synthroid) 12.5 mcg orally each day is prescribed for a client with hypothyroidism. Six weeks later, the health care provider increases the client's dose to 25 mcg daily and gives the client a prescription to be filled at the pharmacy. The client asks the nurse whether the original pill prescription can be completed before starting the new dose. How many of the original pills should the nurse instruct the client to take daily? Record your answer using a whole number. __________ tablets

2 tablets

The nurse asks a nursing student about the uses of the medication dantrolene (Dantrium). The nursing student correctly states that dantrolene is used to manage hypermetabolism of skeletal muscle that occurs in which condition? 1. Low back pain 2. General anesthesia 3. Malignant hyperthermia 4. Hyperplasia of the prostate

3

A client who is receiving therapy with a hypothermia blanket starts to shiver. The nurse raises the blanket temperature and monitors the client. After 15 minutes the client's temperature has not increased and the client is still shivering. What should the nurse do next? 1. Apply a smaller heating pad to the client's axillae and neck areas. 2. Wait 10 more minutes and then check the client's temperature again. 3. Remove the hypothermia blanket and notify the client's health care provider (HCP). 4. Increase the blanket's temperature again and recheck the client's temperature in 15 minutes.

3

A client with a neurological problem is experiencing hyperthermia. Which measure would be least appropriate for the nurse to use in trying to lower the client's body temperature? 1. Giving tepid sponge baths 2. Applying a hypothermia blanket 3. Placing ice packs in the axilla and groin areas 4. Administering acetaminophen (Tylenol) per protocol

3

Metformin (Glucophage) 2 g by mouth is prescribed for a client with type 2 diabetes. Each tablet contains 500 mg. How many tablets should the nurse administer? Record your answer using a whole number. _____ tablets

4 tablets

7. Screening for HIV infection generally involves a. detecting CD8+ cytotoxic T cells in saliva. b. laboratory analysis of saliva to detect CD4+ T cells. c. analysis of lymph tissues for the presence of HIV RNA. d. laboratory analysis of blood to detect HIV antigen or antibody.

7. Correct answer: d Rationale: The most useful screening tests for HIV detect HIV-specific antibodies and/or antigen.

The nurse is monitoring a patient for increased ICP following a head injury. Which of the following manifestations indicate an increased ICP (select all that apply) a. fever b. oriented to name only c. narrowing pulse pressure d. dilated right pupil > left pupil e. decorticate posturing to painful stimulus

A, B, D, E- The first sign of increased ICP is a change in LOC. Other manifestations are dilated ipsilateral pupil, changes in motor response such as posturing, and fever, which may indicate pressure on the hypothalamus. Changes in vital signs would be an increased systolic BP with widened pulse pressure and bradycardia

he classic symptoms of Parkinson's disease include (select all that apply) A. tremor. B. rigidity. C. loss of balance. D. bradykinesia. E. nystagmus.

A,B,D The classic manifestations of PD often include tremor, rigidity, and bradykinesia, which are often called the triad of PD. Reference: 150

Cerebrospinal fluid (CSF) can accumulate around the brain when there is injury to the sites of CSF reabsorption, which are called the: a. Arachnoid villi b. Epidural foramina c. Lateral apertures d. Choroid plexuses

A. Arachnoid villi

A 12-year-old presents with hydrocephalus. Blockage of which of the following would cause this condition? A. Cerebral aqueduct B. Inferior Colliculi C. Red nucleus D. Tegmentum

A. Cerebral Aqueduct

When a student asks in which region of the neuron do nerve impluses travel the fastest, how should the nurse respond? The: A. Large Axon B. Axon Hillock C. Cell Body D. Dendrites

A. Large Axon

Which information should be included in a teaching plan for patients taking oral hypoglycemic drugs? (Select all that apply.) A. Limit your alcohol consumption. B. Report symptoms of anorexia and fatigue. C. Take your medication only as needed. D. Notify your physician if blood glucose levels rise above the level set for you.

A. Limit your alcohol consumption. Correct B. Report symptoms of anorexia and fatigue. Correct D. Notify your physician if blood glucose levels rise above the level set for you.Correct Oral hypoglycemic drugs must be taken on a daily scheduled basis to maintain euglycemia and prevent long-term complications of diabetes. All other options are correct.

Which intervention should the nurse initiate secondary to a change in level of consciousness​ (LOC) for a client experiencing increased intracranial pressure​ (IICP)? (Select all that​ apply.) A. Maintain the head of the bed at 30 degrees B. Assess​ LOC, pupillary​ response, and neurologic status C. Encourage family to visit and keep client engaged in normal activities as possible D. Implement deep vein thrombosis prophylaxis E. Monitor​ ICP/cerebral perfusion pres

A. Maintain the head of the bed at 30 degrees B. Assess​ LOC, pupillary​ response, and neurologic status D. Implement deep vein thrombosis prophylaxis E. Monitor​ ICP/cerebral perfusion pres

During admission of a patient with a severe head injury to the ED, the nurse places highest priority on assessment for a. patency of of airway b. presence of a neck injury c. neurologic status with Glascow Coma Scale d. CSF leakage from ears and nose

A. Patency of airway is the #1 priority with all head injuries

A 27-yr-old patient admitted with diabetic ketoacidosis (DKA) has a serum glucose level of 732 mg/dL and serum potassium level of 3.1 mEq/L. Which action prescribed by the health care provider should the nurse take first? a. Place the patient on a cardiac monitor. b. Administer IV potassium supplements. c. Ask the patient about home insulin doses. d. Start an insulin infusion at 0.1 units/kg/hr.

A. Place the patient on a cardiac monitor

An aide asks a nurse what neurotransmitters interact with. Which response is the nurses best answer? Neurotransmitters interact with the postsynaptic membrane by binding to a: A. Receptor B. Nissl Body C. Glial Cell D. Neurofibril

A. Receptor

A nurse is discussing the membrane that separates the cerebellum from the cerebrum. What term should the nurse use to describe this membrane? a. Tentorium cerebelli b. Fall cerebri c. Arachnoid membrane d. Temporal lobe

A. Tentorium cerebelli

The nurse notes that a client has muscle fasciculations of both bicep muscles. Which additional information should the nurse​ assess? (Select all that​ apply.) A. Urine output B. Last solid food intake C. Blood pressure and pulse D. Body temperature E. List of medications taken

A. Urine output D. Body temperature E. List of medications taken

The nurse is caring for a client with increased intracranial pressure secondary to a brain tumor. Which position should the nurse place the client in to help decrease intracranial​ pressure? A. ​Semi-Fowler B. Left lateral recumbent C. Fowler D. High Fowler

A. ​Semi-Fowler

An Asian American client is experiencing a fever. The nurse recognizes that the client is likely to self-treat the disorder, using which method? 1. Prayer 2. Magnetic therapy 3. Foods considered to be yin 4. Foods considered to be yang

3

41. Which statements best define acute rejection? (Select all that apply.) a. Acute rejection is a cell-mediated immune response. b. Acute rejection is usually a type III rejection. c. Immunosuppressive drugs delay or lessen the intensity of an acute rejection. d. Acute rejection is associated with the body's response to an organ transplant. e. Acute rejection is a response against unmatched human leukocyte antigens (HLAs).

ANS: A, C, D, E Acute rejection is primarily a cell-mediated immune response that occurs within days to months after transplantation. This type of rejection occurs when the recipient develops an immune response against unmatched HLAs after transplantation. A biopsy of the rejected organ usually shows an infiltration of lymphocytes and macrophages characteristic of a type IV reaction. Immunosuppressive drugs may delay or lessen the intensity of an acute rejection.

38. When a tuberculin skin test is positive, the hard center and erythema surrounding the induration are a result of which of these? (Select all that apply.) a. Histamine b. T lymphocytes c. Immune complexes d. Products of complement e. Macrophages

ANS: B,E The reaction site is infiltrated with T lymphocytes and macrophages, resulting in a clear hard center (induration) and a reddish surrounding area (erythema).

15. A patient treated for human immunodeficiency virus (HIV) infection for 6 years has developed fat redistribution to the trunk with wasting of the arms, legs, and face. What recommendation will the nurse give to the patient? a. Review foods that are higher in protein. b. Teach about the benefits of daily exercise. c. Discuss a change in antiretroviral therapy. d. Talk about treatment with antifungal agents.

ANS: C A frequent first intervention for metabolic disorders is a change in antiretroviral therapy (ART). Treatment with antifungal agents would not be appropriate because there is no indication of fungal infection. Changes in diet or exercise have not proven helpful for this problem. DIF: Cognitive Level: Apply (application) REF: 230 TOP: Nursing Process: Planning MSC:

5. During an IgE-mediated hypersensitivity reaction, which leukocyte is activated? a. Neutrophils b. Monocytes c. Eosinophils d. T lymphocytes

ANS: C Of the options provided, only eosinophils are activated during IgE-mediated hypersensitivity reactions.

When providing community health care teaching regarding the early warning signs of AD, which signs would you advise family members to report (select all that apply)? A. Misplacing car keys B. Loses the sense of time C. Difficulty performing familiar tasks D. Problems with performing basic calculations E. Becoming lost in a usually familiar environment

B,C,D,E Difficulty performing familiar tasks, problems with performing basic calculations, and becoming lost in a usually familiar environment are early warning signs of AD. Misplacing car keys is a normal frustrating event for many people.

Which statement by the patient who has newly diagnosed type 1 diabetes indicates a need for additional instruction from the nurse? a. "I will need a bedtime snack because I take an evening dose of NPH insulin." b. "I can choose any foods, as long as I use enough insulin to cover the calories." c. "I can have an occasional beverage with alcohol if I include it in my meal plan." d. "I will eat something at meal times to prevent hypoglycemia, even if I am not hungry."

B. "I can choose any foods, as long as I use enough insulin to cover the calories."

The nurse recognizes the presence of Cushing's triad in the patient with a. Increased pulse, irregular respiration, increased BP b. decreased pulse, irregular respiration, increased pulse pressure c. increased pulse, decreased respiration, increased pulse pressure d. decreased pulse, increased respiration, decreased systolic BP

B. Cushing's triad consists of three vital sign measures that reflect ICP and its effect on the medulla, the hypothalamus, the pons, and the thalamus. Because these structures are very deep, Cushing's triad is usually a late sign of ICP. The signs include an increasing systolic BP with a widening pulse pressure, a bradycardia with a full and bounding pulse, and irregular respirations.

The nurse assesses a client -admitted​ post-head trauma and notes small reactive​ pupils, an intact oculocephalic​ reflex, decorticate​ posturing, and respirations. The nurse should suspect that damage has progressed to which area of the​ brain? A. Midbrain B. Diencephalon C. Pons D. Medulla

B. Diencephalon

during treatment with zidovudine, the nurse needs to monitor for which potential adverse effect? A) retinitis B) DVT C) karposi's sarcoma D) bone marrow supression

D) bone marrow supression

Which cell type is the nurse discussing? A cell that is involved in forming the blood-brain barrier is the: a. Endothelial b. Schwann c. Oligodendrocyte d. Astrocyte

D. Astrocyte

Which immune cell creates antibodies in response to antigens? A. Cytotoxic T lymphocytes B. Helper T lymphocytes C. Cytokines D. B lymphocytes

D. B lymphocytes

A neurologist is teaching the staff about motor neurons. Which information should be included? Motor neurons are structurally classified as _______ neurons. A. Unipolar B. Pseudounipolar C. Bipolar D. Multipolar

D. Multipolar

3. Transmission of HIV from an infected individual to another most commonly occurs as a result of a. unprotected anal or vaginal sexual intercourse. b. low levels of virus in the blood and high levels of CD4+ T cells. c. transmission from mother to infant during labor and delivery and breastfeeding. d. sharing of drug-using equipment, including needles, syringes, pipes, and straws.

3. Correct answer: a Unprotected sexual contact (semen, vaginal secretions, or blood) with a partner infected with human immunodeficiency virus (HIV) is the most common mode of HIV transmission.

A nurse is performing an assessment on a client after a thyroidectomy and notes that the client has developed hoarseness and a weak voice. Which nursing action is appropriate? 1. Check for signs of bleeding. 2. Administer calcium gluconate. 3. Notify the health care provider (HCP) immediately. 4. Reassure the client that this is usually a temporary condition.

4

A client with type 2 diabetes is admitted for elective surgery. The health care provider prescribes regular insulin even though oral antidiabetics were adequate before the client's hospitalization. The nurse concludes that regular insulin is needed because the: 1. Client will need a higher serum glucose level while on bed rest. 2. Possibility of acidosis is greater when a client is on oral hypoglycemics. 3 Dosage can be adjusted to changing needs during recovery from surgery. 4. Stress of surgery may precipitate uncontrollable periods of hypoglycemia.

3. There is better control of blood glucose levels with short-acting (regular) insulin. The level of glucose must be maintained as close to normal as possible. The occurrence of acidosis is greater when the client is receiving exogenous insulin. The stress of surgery will precipitate hyperglycemia, which is best controlled with exogenous insulin

The nurse is instructing the nursing assistant to prevent pressure ulcers in a frail older patient. Which action indicates the nursing assistant has understood the nurse's teaching? a. Bathing and drying the skin vigorously to stimulate circulation b. Keeping the head of the bed elevated 30 degrees c. Limiting intake of fluid and offer frequent snacks d. Turning the patient at least every 2 hours

ANS: D The patient should be turned at least every 2 hours as permanent damage can occur in 2 hours or less. If skin assessment reveals a stage I ulcer while on a 2-hour turning schedule, the patient must be turned more frequently. Limiting fluids will prevent healing; however, offering snacks is indicated to increase healing particularly if they are protein based, because protein plays a role in healing. Use of doughnuts, elevated heads of beds, and overstimulation of skin may all stimulate, if not actually encourage, dermal decline. REF: Page 164

7. A young adult female patient who is human immunodeficiency virus (HIV) positive has a new prescription for efavirenz (Sustiva). Which information is most important to include in the medication teaching plan? a. Take this medication on an empty stomach. b. Take this medication with a full glass of water. c. You may have vivid and bizarre dreams as a side effect. d. Continue to use contraception while taking this medication.

ANS: D To prevent harm, it is most critical to inform patients that efavirenz can cause fetal anomalies and should not be used in patients who may be or may become pregnant. The other information is also accurate, but it does not directly prevent harm. The medication should be taken on an empty stomach with water and patients should be informed that many people who use the drug have reported vivid and sometimes bizarre dreams. DIF: Cognitive Level: Analyze (analysis) REF: 224 TOP: Nursing Process: Implementation MSC:

To help decrease the threat of a melanoma in a blonde-haired, fair-skinned patient at risk, the nurse would advise the patient to do which of the following? a. Apply sunscreen 1 hour prior to exposure. b. Drink plenty of water to prevent hot skin. c. Use vitamins to help prevent sunburn by replacing lost nutrients. d. Apply sunscreen 30 minutes prior to exposure.

ANS: D Wearing sunglasses and sunscreen are recommended by the National Cancer Institute. Drinking water will help with heat exhaustion but will not prevent melanoma. Green tea, fish oil, soy products, and vitamin E are thought to be helpful in minimizing the risk of developing melanoma; however, vitamins do not prevent burn. REF: Page 266

The nurse is teaching a review class to nurses about diabetes mellitus. Which statement by the nurse is correct? a. "Patients with type 2 diabetes will never need insulin." b. "Oral antidiabetic drugs are safe for use during pregnancy." c. "Pediatric patients cannot take insulin." d. "Insulin therapy is possible during pregnancy if managed carefully."

ANS: D Oral medications are generally not recommended for pregnant patients because of a lack of firm safety data. For this reason, insulin therapy is the only currently recommended drug therapy for pregnant women with diabetes. Insulin is given to pediatric patients, with extreme care. Patients with type 2 diabetes may require insulin in certain situations or as their disease progresses. DIF: COGNITIVE LEVEL: Applying (Application) REF: p. 506 TOP: NURSING PROCESS: Implementation MSC: NCLEX: Physiological Integrity: Pharmacological and Parenteral Therapies

The nurse is teaching patients about self-injection of insulin. Which statement is true regarding injection sites? a. Avoid the abdomen because absorption there is irregular. b. Choose a different site at random for each injection. c. Give the injection in the same area each time. d. Rotate sites within the same location for about 1 week before rotating to a new location.

ANS: D Patients taking insulin injections need to be instructed to rotate sites, but to do so within the same location for about 1 week (so that all injections are rotated in one area—for example, the right arm—before rotating to a new location, such as the left arm). Also, each injection needs to be at least 1/2 to 1 inch away from the previous site. DIF: COGNITIVE LEVEL: Applying (Application) REF: p. 517 TOP: NURSING PROCESS: Implementation MSC: NCLEX: Physiological Integrity: Pharmacological and Parenteral Therapies

When teaching a patient who is starting metformin (Glucophage), which instruction by the nurse is correct? a. "Take metformin if your blood glucose level is above 150 mg/dL." b. "Take this 60 minutes after breakfast." c. "Take the medication on an empty stomach 1 hour before meals." d. "Take the medication with food to reduce gastrointestinal (GI) effects."

ANS: D The GI adverse effects of metformin can be reduced by administering it with meals. The other options are incorrect. DIF: COGNITIVE LEVEL: Analyzing (Analysis) REF: p. 508 TOP: NURSING PROCESS: Implementation MSC: NCLEX: Physiological Integrity: Pharmacological and Parenteral Therapies

A health care provider prescribes 36 units of NPH insulin (Novolin N) and 12 units of regular insulin (Novolin R). The nurse plans to administer these drugs in one syringe. Identify the steps in this procedure by listing the numbers by each picture next to the step below in priority order. (Start with the number of the picture that represents the first step and end with the number by the picture that represents the last step.)

Air should be injected into the NPH insulin vile first, which allows withdrawal of the NPH insulin at a later step in the procedure without having to instill air into the vial from a syringe that contains regular insulin. Instilling air into the regular insulin vile increases the pressure in the vile, facilitating removal of the required dose. Removing the desired dose of insulin while the needle is still in the vile reduces the risk of contamination by repeated punctures, and maintains the sharpness of the needle. Having the syringe contain regular insulin first prevents the need to withdraw the regular insulin into a syringe that contains NPH insulin and inadvertently contaminating the regular insulin vial with the longer-acting NPH insulin; contaminating regular insulin with NPH insulin will reduce the speed at which the regular insulin functions, which in turn will delay treatment of a hyperglycemic event. Finally, the required dose of NPH insulin can be removed from the NPH insulin vile.

The nurse is caring for a postoperative patient who had an open appendectomy. The nurse understands that this patient should have some erythema and edema at the incision site 12 to 24 hours post operation dependent on which condition? a. His immune system is functioning properly. b. He is properly vaccinated. c. He has an infection. d. The suppressor T-cells in his body are activated.

a. His immune system is functioning properly. Normal immune function involves an initial inflammatory response, which would cause redness and swelling at the incision site for the first 12-24 hours. This does not indicate an infection.

Which is an example of an alloimmune disease? a. Tuberculin reaction b. Graves disease c. Contact dermatitis d. Penicillin allergy

B Graves disease is a classic autoimmune, or alloimmune, disease. Tuberculin reaction is a type IV reaction. Contact dermatitis is a type I reaction. Penicillin is an allergen that induces type I hypersensitivity.

The nurse is planning care for a client in the acute stage of a burn injury. Which areas will be included in the plan of care? Select all that apply. A) Nutrition B) Psychosocial support C) Pain management D) Fluid resuscitation E) Wound care

Answer: A, C, E Explanation: Nursing care for the client during the acute stage of burn injuries will include wound care, nutritional therapy, and pain management. Fluid resuscitation occurs during the emergency phase of burn care. Psychosocial support will be needed once the client has stabilized.

A nurse working in the pediatric intensive care unit (PICU) is planning care for a pediatric client who is being admitted with a partial-thickness thermal burn. What is true regarding this type of burn? A) Partial-thickness burns are deeper than superficial burns but still involve the epidermis only. B) A superficial partial-thickness burn extends from the skin's surface into the papillary layer of the dermis. C) A deep partial-thickness burn is often bright red and has a moist, glistening appearance with blister formation. D) A superficial partial-thickness burn is less painful than a deep partial-thickness burn.

Answer: B Explanation: A superficial partial-thickness burn extends from the skin's surface into the papillary layer of the dermis. Partial-thickness burns are deeper than superficial burns, extending from the epidermis into the dermis layer as well. A superficial partial-thickness burn is often bright red and has a moist, glistening appearance with blister formation. A deep partial-thickness burn is less painful than a superficial partial-thickness burn because sensation is decreased at the site.

A client who sustained burns to both lower extremities complains to the nurse about feeling frustrated by not being able to provide self-care. Which nursing diagnosis would be appropriate for the client at this time? A) Ineffective Coping B) Powerlessness C) Anxiety D) Situational Low Self-Esteem

Answer: B Explanation: The client is expressing frustration over not being able to provide self-care. The nursing diagnosis most appropriate for the client at this time would be Powerlessness. There is not enough information to determine whether the client is or is not experiencing situational low-self-esteem, ineffective coping, or anxiety.

The nurse is evaluating the adequacy of the burn-injured client's nutritional intake. Which laboratory value is the best indicator of nutritional status? A) Creatine phosphokinase (CPK) B) BUN levels C) Hemoglobin D) Albumin level

Answer: D Explanation: Albumin level is used to indicate protein synthesis and nutritional status. Creatine phosphokinase is used to identify the presence of muscle injuries. BUN levels are used to evaluate kidney function. Hemoglobin levels will fluctuate with the stages of the burn injury, dependent upon the fluid status.

A client is being evaluated after suffering severe burns to the torso and upper extremities. The nurse notes edema at the burned areas. What best describes the underlying cause for this manifestation? A) Decreased osmotic pressure in the burned tissue B) Reduced vascular permeability at the site of the burned area C) Increased fluids in the extracellular compartment D) Inability of the damaged capillaries to maintain fluids in the cell walls

Answer: D Explanation: Burn shock occurs during the first 24-36 hours after the injury. During this period, there is an increase in microvascular permeability at the burn site. The osmotic pressure is increased, causing fluid accumulation. There is a reduction of fluids in the extracellular body compartments.

After a skin graft procedure to the leg, a client is returned to the burn care unit. How will the nurse position the client? A) Place the client flat with the affected extremity abducted. B) Elevate the head of bed 30°. C) Maintain the head of the bed flat. D) Elevate the affected extremity.

Answer: D Explanation: Elevating the affected extremity will reduce edema and promote perfusion. Elevating the head of bed, leaving the head of bed flat, and abducting the extremity will not increase healing or improve the client's long-range prognosis.

The patient has rhinorrhea after a head injury. What action should you take? A. Pack the nares with sterile gauze. B. A loose collection pad may be placed under the nose. C. Suction the drainage with an inline suction catheter. D. Obtain a sample for culture.

B A loose collection pad may be placed under the nose. Do not place a dressing in the nasal cavity, and nothing should be placed inside the nostril. There is no need to culture the drainage. The concern is whether it is spinal fluid, which is determined by a test for glucose or the halo or ring sign.

A female patient has left-sided hemiplegia after an ischemic stroke that occurred 2 weeks earlier. How should you best promote the integrity of the patient's skin? A. Position the patient on her weak side most of the time. B. Alternate the patient between supine and side-lying positions. C. Avoid the use of pillows to promote independence in positioning. D. Establish a schedule for the massage of areas where skin breakdown emerges.

B A position change schedule should be established for stroke patients. An example is side-backside positioning, with a maximum duration of 2 hours for any position. The patient should be positioned on the weak or paralyzed side for only 30 minutes. Pillows may be used to facilitate positioning. Areas of skin breakdown should never be massaged.

The patient has West Nile virus encephalitis and seizures for which phenytoin (Dilantin) is prescribed. What is essential for you do regarding administration? A. Dilute the intravenous (IV) drug in 5% dextrose. B. Assist the patient to maintain good oral hygiene. C. Notify the health care provider if the patient becomes drowsy during IV administration. D. Verify that the levels are between 40 and 60 μg/mL before administration.

B A side effect of Dilantin is gingival hyperplasia, and good oral care minimizes this side effect. The IV drug is diluted in normal saline and will precipitate in D5W. Drowsiness is an expected side effect of IV administration of the drug. Therapeutic levels are between 10 and 20 μg/mL.

You are obtaining a history and physical assessment for a patient with a suspected brain tumor. Which option would be a correct interpretation of findings? A. Balance issues indicate a tumor related in the occipital lobe. B. Personality changes can be caused by a tumor in the frontal lobe. C. Impulsivity is related to a tumor in the temporal lobe. D. Vision is affected by a tumor in the parietal lobe.

B A tumor of the frontal lobe can cause behavioral and personality changes. Balance is related to the parietal lobe or cerebellum. Memory loss and impulsivity are related to alterations in the frontal lobe. Vision is affected by the occipital lobe, and parietal lobe function affects movement.

During the nursing assessment the patient states that he has been experiencing headaches that last for weeks to months at a time and then go into remission. These signs and symptoms are consistent with which type of headache? A. Sinus B. Cluster C. Tension D. Migraine

B Cluster headaches are a rare form of headache, affecting less than 0.1% of the population. Cluster headaches involve repeated headaches that can occur for weeks to months at a time, followed by periods of remission.

Creutzfeldt-Jakob disease is characterized by A. remissions and exacerbations over many years. B. memory impairment, muscle jerks, and blindness. C. Parkinsonian symptoms including muscle rigidity and tremors at rest. D. increased intracranial pressure secondary to decreased cerebrospinal fluid drainage.

B Creutzfeldt-Jakob disease is a fatal brain disorder caused by a prion protein. The earliest symptom of the disease may be memory impairment and behavioral changes. The disease progresses rapidly with mental deterioration, involuntary movements (muscle jerks), weakness in the limbs, blindness, and eventually coma. Reference: 1533

The patient with Parkinson's disease is beginning therapy with carbidopa/levodopa (Sinemet).What teaching should you provide to this patient? A. The medication should be taken every other day. B. It may take several weeks for effects to be observed. C. Side effects are limited to rash and photophobia. D. Constipation may occur.

B Effects of carbidopa/levodopa (Sinemet) may be delayed for several weeks to months. Reference: 1508

You are caring for a patient with peripheral neuropathy who is going to have electromyographic (EMG) studies tomorrow morning. What should you do to prepare the patient? A. Ensure the patient has an empty bladder. B. Instruct the patient that there is no risk of electric shock. C. Ensure the patient has no metallic jewelry or metal fragments. D. Instruct the patient that he or she may experience pain during the study.

B Electromyography (EMG) assesses electrical activity associated with nerves and skeletal muscles. Needle electrodes are inserted to detect muscle and peripheral nerve disease. You should inform the patient that pain and discomfort are associated with insertion of needles. There is no risk of electric shock with this procedure.

The patient is newly diagnosed with encephalitis caused by herpes simplex virus (HSV) infection. What is essential for you to do? A. Administer penicillin. B. Administer acyclovir. C. Perform a Glasgow Coma Scale assessment. D. Facilitate a magnetic resonance imaging (MRI) study.

B Encephalitis is usually viral, and it is treated with antiviral acyclovir (Zovirax). For maximum benefit, the drug should be started in a timely manner. Penicillin is used for bacterial meningitis. Because the diagnosis is known, treatment of the cause is more important than additional assessment.

You are counseling the family of a patient with Huntington's disease (HD) about the genetics involved in the disease. You would be correct in informing the family that the genetic risk for manifestation of the disease in genetic transmission is A. 1 in every 4 pregnancies. B. 1 in every 2 pregnancies. C. only evident in male children. D. impossible to predict.

B HD is a genetically transmitted, autosomal dominant disorder that affects both men and women of all races. The offspring of a person with this disease have a 50% risk of inheriting it. Reference: 1515

You would correctly identify the pathophysiologic etiology of myasthenia gravis as a deficit of A. dopamine. B. acetylcholine. C. myelin. D. albumin.

B MG is caused by an autoimmune process in which antibodies attack acetylcholine (ACh) receptors, resulting in a decreased number of ACh receptor (AChR) sites at the neuromuscular junction. This prevents ACh molecules from attaching and stimulating muscle contraction. Reference: 1510

What is the treatment of choice for normal pressure hydrocephalus? A. Donepezil (Aricept) B. Shunt C. Furosemide (Lasix) D. Aspiration

B Normal pressure hydrocephalus results from an obstruction in the flow of cerebrospinal fluid (CSF), which causes a buildup of CSF fluid in the brain. Manifestations of the condition include dementia, urinary incontinence, and difficulty walking. Meningitis, encephalitis, or head injury may cause the condition. If diagnosed early, it is treated by surgically inserting a shunt to divert the fluid.

Which statement is true of serum sickness? a. Immune complexes are formed and deposited in target issues. b. Immune complexes are formed and deposited in the blood vessels. c. It is considered a Type II hypersensivity response. d. Cytotoxic T lymphocytes are responsible for the condition.

A Serum sickness reactions are caused by the formation of immune complexes in the blood and their deposition in target tissues. An Arthus reaction deposits complexes into the walls of blood vessels. Serum sickness is a Type III hypersensitivity reaction. Type IV mechanisms occur through either cytotoxic T lymphocytes or lymphokine-producing T-helper (Th) 1 cells.

4. The nurse teaches the staff ensuring that standard precautions should be used when providing care for which type of patient? A. All patients regardless of diagnosis B. Pediatric and gerontologic patients C. Patients who are immunocompromised D. Patients with a history of infectious diseases

A Standard precautions are designed for all care of all patients in hospitals and health care facilities.

11. An older adult patient is brought to the primary health care provider by an adult child reporting confusion. What testing should the nurse anticipate obtaining from this patient? A. Urinalysis B. Sputum culture C. Red blood cell count D. White blood cell count

A The developments of urinary tract infections commonly contribute to atypical manifestations such as cognitive and behavior changes in older adults. Sputum culture, red blood cell count, and white blood cell count may be done, but the first step would be to assess for a possible urinary tract infection.

Which assessment finding is most important for you to follow-up in a newly admitted adult patient diagnosed with viral encephalitis? A. Positive Babinski sign B. Negative Kernig sign C. Doll's-eye reflex D. Deep tendon reflex 2+

A Adults have a negative Babinski sign (toes curl downward, plantar reflex). A positive sign in an adult can indicate disease of the brain or spinal cord. The other signs are normal findings.

The patient is admitted with a diagnosis of bacterial meningitis. The patient has a temperature of 101° F and a headache rated as an 8. Which prescription has a priority for you to administer? A. IV cefuroxime (Ceftin) B. Vital signs C. PO acetaminophen (Tylenol) D. Neurologic check

A Bacterial meningitis is a medical emergency, and treating the cause is a priority over treating the symptoms or further assessing effects of the disease process. The antibiotic may be given after cultures are obtained but before the diagnosis is confirmed.

9. A nurse was accidently stuck with a needle used on a patient who is infected with human immunodeficiency virus (HIV). After reporting the incident, what care should this nurse first receive? A. Personal protective equipment B. Combination antiretroviral therapy C. Counseling to report blood exposures D. A negative evaluation by the manager

B Postexposure prophylaxis with combination antiretroviral therapy can significantly decrease the risk of infection. Personal protective equipment should be available, although it may not have stopped this needle stick. The needle stick has been reported. The negative evaluation may or may not be needed but would not occur first.

After you administer a dose of risperidone (Risperdal) to a patient with delirium, which intended effect of the medication do you assess for? A. Lying quietly in bed B. Alleviation of depression C. Reduction in blood pressure D. Disappearance of confusion

A Risperidone is an antipsychotic drug that reduces agitation and produces a restful state in patients with delirium. However, it should be used with caution

What is important when obtaining a history of a patient with a neurologic problem? A. Have patient agree or disagree with suggested symptoms to obtain a thorough history. B. Mode of onset and course of illness are essential aspects. C. Check out neurologic problems caused by nutrition by asking about sodium. D. Assess for dementia using the Confusion Assessment Method (CAM).

B The mode of onset and the course of the illness are especially important aspects of the history. The nature of a neurologic disease process often can be described by these facts alone. Avoid suggesting certain symptoms or using leading questions. Nutritional deficits of B vitamins are most likely to cause neurologic problems. CAM is used to assess for delirium.

What is a major goal of treatment for the patient with AD? A. To maintain patient safety B. To maintain or increase body weight C. To return to a higher level of self-care D. To enhance functional ability over time

A The overall goals are that the patient with AD will (1) maintain functional ability for as long as possible, (2) be maintained in a safe environment with a minimum of injuries, (3) have personal care needs met, and (4) have dignity maintained. You should emphasize patient safety while planning and providing nursing care. Reference: 1527

A clients with Parkinson's disease is experiencing tremors, rigidity, and bradykinesia. The nurse anticipates that the physician will prescribe which medication to control these symptoms? A. Phenytoin (Dilantin) B. Carbidopa-levodopa (Sinemet) C. Pyridostigmine (Mestinon) D. Warfarin (Coumadin)

B. Carbidopa-levodopa is an antiparkinsonian agent and is used to control symptoms of Parkinson's disease. Phenytoin is an anticonvulsant and antidysrhythmic. Pyridostigmine is a cholinergic medication often used to treat myasthenia gravis. Warfarin is an anticoagulant.

A client experiences an episode of Bell's palsy and complains about increasing clumsiness. The nurse should prepare the client for which diagnostic study(ies) to determine the cause of the assessment findings? A. Serum sodium level B. Cerebral angiography C. Lumbar puncture (LP) D. Oculovestibular reflex E. Electroencephalogram F. Computed tomography

B, C, F The most sensitive and specific tests that provide relevant diagnostic information for these types of pathology are cerebral angiography, LP, and CT. The imaging studies potentially illustrate central nervous system lesions and the LP enables the care provider to analyze cerebrospinal fluid for immunoglobulins (antibodies) and other components. Because the client's neurological problem unlikely to be metabolic, the sodium level is unlikely to be helpful (option A). Usually electroencephalogram and oculovestibular reflex are tests that are reserved to evaluate electrical activity of the brain in seizure disorders and to determine brain death (options D and E). In addition, the oculovestibular reflex is not performed on a client who is conscious.

The nurse is taking a health history from a 29-yr-old patient at the first prenatal visit. The patient reports that she has no personal history of diabetes, but her mother has diabetes. Which action will the nurse plan to take? a. Teach the patient about administering regular insulin. b. Schedule the patient for a fasting blood glucose level. c. Teach about an increased risk for fetal problems with gestational diabetes. d. Schedule an oral glucose tolerance test for the twenty-fourth week of pregnancy.

B. Schedule the patient for a fasting blood glucose level.

The nurse is obtaining a medication history from an 18-year-old female patient who has been diagnosed with genital herpes. Which drug would the nurse expect this patient to be prescribed? A. amantadine (Symmetrel) B. acyclovir (Zovirax) C. zidovudine (Retrovir) D. ribavirin (Virazole)

B. acyclovir (Zovirax) Acyclovir is the drug of choice to treat herpes simplex infections. Ribavirin is effective against respiratory syncytial virus (RSV), zidovudine against human immunodeficiency virus (HIV), and amantadine against Haemophilus influenzae type A.

The dose-limiting toxicity of ganciclovir treatment is assessed by monitoring A. liver function tests (LFTs). B. complete blood count (CBC). C. blood urea nitrogen (BUN). D. creatine phosphokinase (CPK).

B. complete blood count (CBC) Bone marrow suppression is a dose-limiting toxicity of ganciclovir; therefore, CBC should be monitored

Which of the following patients has the best chance of recovery from nerve injury? A. Torn nerve B. Crushed nerve C. Injury located closer to the cell body of the nerve D. Injury located closer to the synapse

B. crushed nerve

The nurse plans care for a patient with increased ICP with the knowledge that the best way to position the patient is to a. keep the head of the bed flat b. elevate the head of the bed to 30 degrees c. maintain patient on the left side with the head supported on a pillow d. use a continuous rotation bed to continuously change patient position

B. elevate the head of the bed to 30 degrees

The nurse is providing discharge teaching to an older adult. Which information should the nurse include in the session to help the older adult avoid development of​ IICP? A. ​"With any early signs of​ infection, be sure to call your healthcare​ provider." B. ​"Make sure to use your walker as we​ discussed." C. ​"Be sure to check your blood sugar more frequently than previously to manage your​ diabetes." D. ​"You should stop driving and exchange your license for a​ non-driver identification​ card."

B. ​"Make sure to use your walker as we​ discussed."

A 45-year-old male was previously diagnosed with Parkinson disesase. He has impaired fine repetitive motor movements. Which areas does the nurse suspect is most likely damaged?

Basal Ganglia

The nurse is reviewing the intracranial pressure​ (ICP) readings for a child with a brain tumor. Which reading should the nurse interpret as normal for the​ child? A. 12 mmHg B. 3 mmHg C. 8 mmHg D. 1 mmHg

B. 3 mmHg

A patient who has type 1 diabetes plans to swim laps for an hour daily at 1:00 PM. What advice should the clinic nurse plan to give the patient? a. Increase the morning dose of NPH insulin (Novolin N). b. Check glucose level before, during, and after swimming. c. Time the morning insulin injection to peak while swimming. d. Delay eating the noon meal until after finishing the swimming.

B. Check glucose level before, during, and after swimming.

Which action by the patient who is self-monitoring blood glucose indicates a need for additional teaching? a. Washes the puncture site using warm water and soap. b. Chooses a puncture site in the center of the finger pad. c. Hangs the arm down for a minute before puncturing the site. d. Says the result of 120 mg indicates good blood sugar control.

B. Chooses a puncture site in the center of the finger pad.

A patient who has a new diagnosis of type 2 diabetes asks the nurse about a new insulin that can be inhaled. "Is there a reason I can't take that drug?" Which condition, if present in the patient, would be a concern? A. Atrial fibrillation B. Chronic lung disease C. Hypothyroidism D. Rheumatoid arthritis

B. Chronic lung disease

A client with a minor head injury has a Glasgow Coma Scale score of 15 out of 15. What does this score indicate to the​ nurse? (Select all that​ apply.) A. Client withdraws to touch. B. Client uses appropriate words and phrases. C. Client spontaneously opens the eyes. D. Client is oriented to​ person, place, and time. E. Client withdraws to pain.

B. Client uses appropriate words and phrases. C. Client spontaneously opens the eyes. D. Client is oriented to​ person, place, and time.

The nurse is assessing a client who fell into a cold lake. Which assessment finding indicates that the​ client's body is attempting to regulate its​ temperature? (Select all that​ apply.) A. Sweating B. Cold hands C. Thirst D. Shivering E. Sleepiness

B. Cold hands D. shivers ​Rationale: When the skin is​ chilled, the body attempts to regulate temperature by vasoconstriction of blood vessels. This could be why the​ client's hands are cold. The body also shivers to increase heat production. The body does not regulate temperature through​ sleep, thirst, or by sweating.

A 29 year old male presents at the clinic complaining of severe fatigue, night sweats and fever. He reports having multiple sexual partners and unprotected sex. HIV/AIDS is suspected. What diagnostic test would be ordered? A. PCR B. EIA C. Western Blot assay D. Complete metabolic panel

B. EIA

If a neurons membrane potential is held close to the threshold potential by EPSPs (excitatory postsynaptic potentials), the neuron is said to be: A. Hyperpolarized B. Facilitated C. Integrated D. Inhibited

B. Facilitated

Which action should the nurse take after a patient treated with intramuscular glucagon for hypoglycemia regains consciousness? a. Assess the patient for symptoms of hyperglycemia. b. Give the patient a snack of peanut butter and crackers. c. Have the patient drink a glass of orange juice or nonfat milk. d. Administer a continuous infusion of 5% dextrose for 24 hours.

B. Give the patient a snack of peanut butter and crackers.

A new patient presents at the clinic with the following history: A CD4 cell count of 500, generalized lymphadenopathy, and a positive HIV test 8 years ago. What phase of HIV infection is the patient in? A. Primary infection phase B. Latent phase C. Conversion phase D. Overt AIDS phase

B. Latent phase

Which statement is true regarding atopic individuals? a. If one parent has allergies, then a 4% chance exists that the offspring will have similar allergies. b. If two parents have allergies, then a 50% chance exists that their offspring will have similar allergies. c. Atopic individuals tend to produce higher quantities of IgE. d. No genes are associated with an atopic state.

C Higher quantities of IgE are present in atopic individuals. If one parent has an allergy, then the individual has a 40% of having allergies; with two parents, the individual has an 80% of having allergies. Multiple genes have been associated with the atopic state.

Which statement is true regarding unmatched packed red blood cell (RBC) transfusions? a. Only three different RBC antigens have been identified. b. Approximately 80 major carbohydrate antigens exist. c. People with O type blood have neither A or B antigens. d. A person with type A blood contains anti-O antibodies.

C Type O blood does not contain type A or B antigens. However, 80 major different RBC antigens are present. A person with type A blood carries anti-B antibodies and a person with type B blood carries anti-A antibodies. Those with type O blood have anti-A and anti-B antibodies.

The patient had an ischemic stroke 5 hours earlier. What treatment do you anticipate? A. Administer nicardipine (Cardene) for the patient's blood pressure of 200/100 mm Hg. B. Administer systemic thrombolytic tissue plasminogen activator (tPA). C. Make patient NPO. D. Administer acetaminophen (Tylenol) prophylactically.

C About 25% of patients worsen in the first 24 to 48 hours after a stroke. Patients should have nothing by mouth (NPO) until the stroke has stabilized to ensure there is no progression to loss of gag reflex and aspiration. Elevated blood pressure is common immediately after a stroke and may be a protective response to maintain cerebral perfusion. A drug is not used to lower blood pressure unless the systolic pressure is more than 220 mm Hg. Systemic tPA must be administered within 3 to 4.5 hours of stroke onset. The patient's temperature is treated, but not prophylactically.

You explain to the patient with a stroke who is scheduled for angiography that the test is used to determine the A. presence of increased intracranial pressure (ICP). B. site and size of the infarction. C. patency of the cerebral blood vessels. D. presence of blood in the cerebrospinal fluid.

C Angiography provides visualization of cerebral blood vessels, can provide an estimate of perfusion, and can detect filling defects in the cerebral arteries.

What is a common occurrence due to the strange behavior in Pick's disease? A. Patients are institutionalized sooner than those with other dementias. B. Polypharmacy results from multiple drug regimens. C. Psychiatrists often see these patients first. D. Electroconvulsive therapy (ECT) is often used.

C Because of the strange behavior associated with frontotemporal dementia, psychiatrists often see these patients first.

Benzodiazepines are indicated in the treatment of cases of delirium that have which cause? A. Polypharmacy B. Cerebral hypoxia C. Alcohol withdrawal D. Electrolyte imbalances

C Benzodiazepines can be used to treat delirium associated with sedative and alcohol withdrawal. However, these drugs may worsen delirium caused by other factors and must be used cautiously.

You would identify which tool as the most useful for diagnosing a tension headache? A. CT scan B. Electromyography (EMG) C. Careful history taking D. Assessment of deep tendon reflexes

C Careful history taking is probably the most important tool for diagnosing tension-type headache. Electromyography (EMG) may be performed. This test may reveal sustained contraction of the neck, scalp, or facial muscles. However, many patients may not show increased muscle tension with this test, even when the test is done during the actual headache. Reference: 1486

What does the pathophysiology of Alzheimer's disease (AD) most commonly involve? A. Presence of presenilin-1 and presenilin-2 genes B. Dissolving of plaques in brain tissue C. Changes in brain structure and function D. Residual inflammation from arboviruses

C Characteristic findings of AD are related to changes in the brain's structure and functions: (1) amyloid plaques (more in certain parts of the brain); (2) neurofibrillary tangles (more plentiful than normally seen); and (3) loss of connections between cells and cell death (and atrophy). Three genes are important in the cause of early-onset AD, but only a small percentage of people younger than 60 years develop AD. All individuals develop plaques in their brain tissue as part of aging, but in AD, there are more plaques in certain parts of the brain. Encephalitis is a treatable cause of dementia.

A patient with right-sided hemiplegia and aphasia resulting from a stroke most likely has involvement of the A. brainstem. B. vertebral artery. C. left middle cerebral artery. D. right middle cerebral artery.

C If the middle cerebral artery is involved in a stroke, the expected clinical manifestations include aphasia, motor and sensory deficit, and hemianopsia on the dominant side and include neglect, motor and sensory deficit, and hemianopsia on the nondominant side.

A patient experiencing TIAs is scheduled for a carotid endarterectomy. You explain that this procedure is done to A. decrease cerebral edema. B. reduce the brain damage that occurs during a stroke in evolution. C. prevent a stroke by removing atherosclerotic plaques blocking cerebral blood flow. D. provide a circulatory bypass around thrombotic plaques obstructing cranial circulation.

C In carotid endarterectomy, the atheromatous lesions are removed from the carotid artery to improve blood flow.

What term should the nurse use when talking about the outermost membrane surrounding the brain? a. Dura mater B. Arachnoid mater c. Pita mater d. Fall cerebi

A. Dura mater

A cell was isolated from the CNS. A researcher revealed that its main function was to clear cellular debris. What type of cell is the researcher studying? A. Astrocyte B. Ependymal Cell C. Microglia D. Schwann Cell

C. Microglia

An 11-year-old male is newly diagnosed with type 1 DM. Which classic symptoms should the nurse assess the patient for? a. Recurrent infections, visual changes, fatigue, and paresthesias b. Polydipsia, polyuria, polyphagia, and weight loss c. Vomiting; abdominal pain; sweet, fruity breath; dehydration; and Kussmaul breathing d. Weakness, vomiting, hypotension, and mental confusion

b

A client with trigeminal neuralgia asks a nurse what can be done to minimize the episodes of pain. The nurse's response is based on an understanding that the symptoms can be triggered by: A. infection or stress B. Excessive watering of the eyes or nasal stuffiness C. Sensations of pressure or extreme temperature D. Hypoglycemia and fatigue

C. Pain that accompany this neuralgia are triggered by stimulation of the terminal branches of the trigeminal nerve. Symptoms can be triggered by pressure from washing the face, brushing the teeth, shaving, eating and drinking. Symptoms also can be triggered by thermal stimuli such as a draft of cold air. The symptoms listed in the other options do not trigger the pain.

Which patient statement indicates that the nurse's teaching about exenatide (Byetta) has been effective? a. "I may feel hungrier than usual when I take this medicine." b. "I will not need to worry about hypoglycemia with the Byetta." c. "I should take my daily aspirin at least an hour before the Byetta." d. "I will take the pill at the same time I eat breakfast in the morning."

C. "I should take my daily aspirin at least an hour before the Byetta."

A nurse remembers the brain receives approximately ___% of the cardiac output. a. 80 b. 40 c. 20 d. 10

C. 20

Assuming the patient eats breakfast at 8:30 AM, lunch at noon, and dinner at 6:00 PM, he or she is at highest risk of hypoglycemia after an 8:00 AM dose of NPH insulin at what time? A. 10:00 AM B. 2:00 PM C. 5:00 PM D. 8:00 PM

C. 5:00 PM Correct Breakfast eaten at 8:30 AM would cover the onset of NPH insulin, and lunch will cover the 2 PM time frame. However, if the patient does not eat a mid-afternoon snack, the NPH insulin may be peaking just before dinner without sufficient glucose on hand to prevent hypoglycemia.

When a patient with type 2 diabetes is admitted for a cholecystectomy, which nursing action can the nurse delegate to a licensed practical/vocational nurse (LPN/VN)? a. Communicate the blood glucose level and insulin dose to the circulating nurse in surgery. b. Discuss the reason for the use of insulin therapy during the immediate postoperative period. c. Administer the prescribed lispro (Humalog) insulin before transporting the patient to surgery. d. Plan strategies to minimize the risk for hypoglycemia or hyperglycemia during the postoperative period.

C. Administer the prescribed lispro (Humalog) insulin before transporting the patient to surgery.

Which leukocytes participate in the inflammatory response? A. Eosinophils B. Monocytes C. Neutrophils D. All of the above

C. All of the above

Which statement, if made by a patient with HIV infection, demonstrates a need for continued patient teaching? A) "I will change my position slowly to prevent dizziness and potential injury." B) "I must take these medications exactly as prescribed for the rest of my life." C) "I don't need to use condoms as long as I take my medication as prescribed." D) "I should take my zidovudine on an empty stomach to enhance absorption."

C. Antiretroviral drugs do not stop the transmission of HIV, and patients need to continue standard precautions.

A client with a traumatic brain injury is intubated and placed on mechanical ventilation. Which measurement should the nurse use to evaluate the effectiveness of these respiratory​ interventions? A. Glasgow Coma Scale score B. Cranial nerve function C. Arterial blood gas results D. Motor and sensory function

C. Arterial blood gas results

A 30-yr-old patient has a new diagnosis of type 2 diabetes. When should the nurse recommend the patient schedule a dilated eye examination? a. Every 2 years b. Every 6 months c. As soon as available d. At the age of 39 years

C. As soon as available

An active 32-yr-old male who has type 1 diabetes is being seen in the endocrine clinic. Which finding indicates a need for the nurse to discuss a possible a change in therapy with the health care provider? a. Hemoglobin A1C level of 6.2% b. Heart rate at rest of 58 beats/min c. Blood pressure of 140/88 mmHg d. High-density lipoprotein (HDL) level of 65 mg/dL

C. Blood pressure of 140/88 mmHg

Which statement by a nurse to a patient newly diagnosed with type 2 diabetes is accurate? a. Insulin is not used to control blood glucose in patients with type 2 diabetes. b. Complications of type 2 diabetes are less serious than those of type 1 diabetes. c. Changes in diet and exercise may control blood glucose levels in type 2 diabetes. d. Type 2 diabetes is usually diagnosed when a patient is admitted in hyperglycemic coma.

C. Changes in diet and exercise may control blood glucose levels in type 2 diabetes.

The health care provider suspects the Somogyi effect in a 50-yr-old patient whose 6:00 AMblood glucose is 230 mg/dL. Which action will the nurse teach the patient to take? a. Avoid snacking right before bedtime. b. Increase the rapid-acting insulin dose. c. Check the blood glucose during the night. d. Administer a larger dose of long-acting insulin.

C. Check the blood glucose during the night.

What structure ensures collateral blood flow from blood vessels supplying the brain. a. Carotid arteries b. Basalartery c. Circle of willis d. Vertebral arteries

C. Circle of Willis

A male patient who has a history of type 2 diabetes mellitus is admitted to the medical unit with a diagnosis of pneumonia. The patient has many questions regarding his care and asks the nurse why everyone keeps telling him about hemoglobin A1C. The nurse will inform the patient that hemoglobin A1C provides information regarding: C)patient compliance with treatment regimen for several months previously.

Correct answer: C Rationale: Hemoglobin A1C is a good indicator of the patient's compliance with the therapy regimen for several months previously.

After the 0700 report, the day shift nurse notices that a patient has a 0730 dose of insulin due and goes to the automated dispensing machine to retrieve the insulin. The nurse sees that the night shift nurse had removed the 0730 dose of insulin, but the medication administration record (MAR) has not been signed by the nurse. The patient is confused and says "she thinks" the night nurse gave her the insulin. The patient's blood glucose level is 142 mg/dL. What will the day shift nurse do? C)Ask the charge nurse to call the night nurse at home to clarify whether the insulin was given.

Correct answer: C Rationale: Never guess whether a drug was given. Taking the drug out of the machine does not mean it was given. The nurse should ask the night nurse what was done.

The patient was taking metformin before this hospitalization. To facilitate better glucose control, the patient has been switched to insulin therapy while hospitalized. The patient asks the nurse why it is so important to time meals with the insulin injection and to give him an example of a long-acting insulin. Which drug will the nurse tell the patient is a long-acting insulin? C)Insulin detemir (Levemir)

Correct answer: C Rationale: The nurse should inform the patient that timing of meals with insulin and oral antidiabetic therapy is important to prevent hypoglycemia and to obtain the most optimal results from the antidiabetic therapy. Insulin detemir (Levemir) is a long-acting insulin while insulin glulisine (Apidra) is a rapid-acting insulin. Insulin isophane suspension (NPH) is an intermediate-acting insulin, and regular insulin (Humulin R) is a short-acting insulin.

The earliest signs of increased ICP the nurse should assess for include a. Cushing's triad b. unexpected vomiting c. decreasing level of consciousness (LOC) d. dilated pupil with sluggish response to light

C. One of the most sensitive signs of increased intracranial pressure (ICP) is a decreasing LOC. A decrease in LOC will occur before changes in vital signs, ocular signs, and projectile vomiting occur

While the nurse performs ROM on an unconscious patient with increased ICP, the patient experiences severe decerebrate posturing reflexes. The nurse should a. use restraints to protect the patient from injury b. administer CNS depressants to lightly sedate the patient c. perform the exercises less frequently because posturing can increase ICP d. continue the exercises because they are necessary to maintain musculoskeletal function

C. Perform the exercises less frequently because posturing can increase ICP- If reflex posturing occurs during ROM or positioning of the patient, these activities should be done less frequently until the patient's condition stabilizes, because posturing can case increases in ICP. Neither restraints nor CNS depressants would be indicated.

An experiment looking at an isolated neuron revealed a sensory nerve with one process containing a dendritic portion extending away from the CNS and an axon extending toward the CNS. Which of the following classifications would this neuron fall into? A) Bipolar B) Multipolar C) Pseudounipolar D) Interpolar

C. Pseudounipolar

When caring for an older adult experiencing problems with intracranial​ regulation, which change noted by the nurse during care would be attributed to normal​ age-associated changes versus those indicative of issues requiring​ attention? (Select all that​ apply.) A. Some decline in mental status B. Alterations in​ long-term memory C. Slower fine finger movements D. Slower impulse transmission and reaction to stimuli E. Decreased perception of temperature sensation

C. Slower fine finger movements D. Slower impulse transmission and reaction to stimuli E. Decreased perception of temperature sensation

A neurologist is teaching about sensory pathways. Which information should the neurologist include? Sensory pathways in the spinal cored include the: a. Corticospinal tract b. Pyramids c. Spinothalamic tract d. Anterior column

C. Spinalthalamic tract

The nurse on the clinical unit is assigned to four patients. Which patient should she assess first? a. patient with a skull fracture whose nose is bleeding b. elderly patient with a stroke who is confused and whose daughter is present c. patient with meningitis who is suddenly agitated and reporting a HA of 10 on a 0 to 10 scale d. patient who had a craniotomy for a brain tumor who is now 3 days postoperative and has had continued emesis

C. patient with meningitis who is suddenly agitated and reporting a HA of 10 on a 0 to 10 scale

The nurse is preparing to use the tympanic membrane to measure the temperature of a​ 4-year-old child. Which approach should the nurse take when completing this​ measurement? A. Pull the earlobe back and up. B. Pull the pinna back and down. C. Pull the pinna back and up. D. Pull the earlobe back and down.

C. pull the pinna back and up ​Rationale: The pinna is pulled straight back and upward when taking temperature in children over 3 years of age. To measure temperature using the tympanic membrane in an​ infant, the pinna is pulled straight back and slightly downward. The earlobe is not manipulated to measure temperature using the tympanic membrane.

after giving an ijection to a patient with HIV infection, the nurse accidentally recieves a needlestick from a too-full needle disposal box. Reccomendations for occupation HIV exposure may include the use of which drugs? A. didanoside B. lamivudine and enfuviritude C. zidovudine lamivudine and indinavir D. acyclovir

C. zidovudine lamivudine and indinavir

The nurse is providing a​ community-based teaching course to a group of high school parents concerning brain injury. Which participant statement indicates accurate understanding of the population most at risk for traumatic brain injury​ (TBI)? A. ​"TBIs most commonly occur in men between the ages of 18 and 25 years of​ age." B. ​"TBIs are common across the​ lifespan, affecting men and women fairly​ equally." C. ​"They are most common in the very young ​(0 -4 ​years) or old​ (65 years and​ above)." D. ​"Due to their age and the high number of falls and​ injuries, toddlers and children have higher levels of​ TBIs."

C. ​"They are most common in the very young ​(0 -4 ​years) or old​ (65 years and​ above)."

During a neurologic assessment of a​ client, the nurse received an affirmative response to the​ question, "Have you ever been diagnosed with a neurologic​ illness?" Which​ follow-up question would be most important for the nurse to ask this​ client? A. ​"Are you experiencing​ pain?" B. ​"When was your last blood work​ done?" C. ​"What helped the​ problem? What made it​ worse?" D. ​"Do you have noticeable​ tremors? Do you feel you are​ clumsier

C. ​"What helped the​ problem? What made it​ worse?"

The nurse enters the patient's room to complete the discharge process and finds the patient to be lying in bed unresponsive and breathing. The patient has a blood glucose reading of 48 mg/dL. What is the most appropriate response by the nurse? C) Roll the patient to the side and administer the ordered glucagon.

Correct answer: C Rationale: Glucagon, a natural hormone secreted by the pancreas, is available as a subcutaneous injection to be given when a quick response to severe hypoglycemia is needed. Because glucagon injection may induce vomiting, roll an unconscious patient onto his or her side before injection. Glucagon is useful in the unconscious hypoglycemic patient without established intravenous access. The patient is at risk for aspiration so nothing should be administered by mouth. CPR is not indicated.

The nurse is caring for a patient who is being discharged home after a splenectomy. What information on immune function needs to be included in this patient's discharge planning? a. The mechanisms of the inflammatory response b. Basic infection control techniques c. The importance of wearing a face mask in public d. Limiting contact with the general population

b. Basic infection control techniques

A patient with type 1 diabetes is admitted to the medical unit with an acute exacerbation of chronic obstructive pulmonary disease (COPD). He is placed on IVPB antibiotics, nebulizer treatments with albuterol, and an IV corticosteroid, and he is also taking a proton pump inhibitor for gastrointestinal esophageal reflux disease (GERD). He takes a dose of glargine insulin every evening. That evening the nurse notes that his blood glucose level is 170 mg/dL. The next morning, his fasting glucose level is 202 mg/dL. What is the most likely cause of his elevated glucose levels? D)The corticosteroid

Correct answer: D Rationale: Corticosteroids antagonize the hypoglycemic effects of insulin, resulting in elevated blood glucose levels.

The nurse has just administered the morning dose of a patient's lispro (Humalog) insulin. Just after the injection, the dietary department calls to inform the patient care unit that breakfast trays will be 45 minutes late. What will the nurse do next? D)Give the patient food, such as cereal and skim milk, and juice.

Correct answer: D Rationale: Lispro insulin's onset of action is 15 minutes. It is essential that a diabetic patient eat a meal after injection. Otherwise profound hypoglycemia may result.

The patient is being discharged home with insulin aspart (NovoLog) and insulin isophane suspension (NPH). Which information does the nurse include when providing discharge teaching to the patient? D)Draw up the insulin aspart (NovoLog) first, then the insulin isophane suspension (NPH) into the same syringe.

Correct answer: D Rationale: The rapid-acting (clear) then the intermediate-acting (cloudy) insulins should be mixed in the syringe after the appropriate amount of air has been injected. Insulin is stored at room temperature when it will be used within the month. The injection should be administered at a 90-degree angle for patients who have adequate body fat and at a 45-degree angle for patients who are very thin. Insulins should be rolled prior to administration and not shaken.

The nurse instructs a patient with type 1 diabetes mellitus to avoid which of the following drugs while taking insulin? a. Furosemide (Lasix) b. Dicumarol (Bishydroxycoumarin) c. Reserpine (Serpasil) d. Cimetidine (Tagamet)

ANS: A Furosemide is a loop diuretic and can increase serum glucose levels; its use is contraindicated with insulin. Dicumarol, an anticoagulant; reserpine, an anti-hypertensive; and cimetidine, an H2 receptor antagonist, do not affect blood glucose levels.

36. Immunoglobulin E (IgE) is associated with which type of hypersensitivity reaction? a. I b. II c. III d. IV

ANS: A Hypersensitivity reactions have been divided into four distinct types: type I (IgE-mediated) hypersensitivity reactions, type II (tissue-specific) hypersensitivity reactions, type III (immune complex-mediated) hypersensitivity reactions, and type IV (cell-mediated) hypersensitivity reactions.

Which statement is true regarding a type IV allergic reaction? a. Is immediate in its action b. Is infiltrated with B cells c. Is mediated by antibody production d. Can be transferred by cells

D Type IV hypersensitivity reactions can be transferred by cells but not by serum. Their actions are delayed because their onset takes from 24 to 72 hours. The site is infiltrated with T lymphocytes and macrophages. A clear hard center surrounds the erythema (redness).

The patient had a blunt head injury. What is most important for you to do before the patient's discharge? A. Have the patient sign the discharge papers. B. Teach the patient how to perform the Glasgow Coma Scale (GCS). C. Tell the patient to return if he has a headache. D. Ensure there is a responsible adult to check on the patient

D Complications from a head injury can arise 2 to 3 days later, and the discharged patient must have a responsible adult who can stay with or check on the patient. The patient may understand the instructions but without an objective observer, he or she would not be aware whether some of the key symptoms were occurring. A patient would not know how to do the GCS if impaired or confused. A headache is not a concern, but a worsening headache unrelieved by over-the-counter medications needs to be checked.

. During the nursing assessment the patient identifies experiencing episodic headaches with pain described as a 10 on a scale of 1 to 10. The patient describes the pain as unilateral and located on the left side, lasting for 2 to 3 days. The patient also experiences nausea. You would determine the patient was experiencing which type of headache? A. Sinus B. Cluster C. Tension D. Migraine

D Migraine headache is a recurring headache characterized by unilateral (sometimes bilateral) throbbing pain, a triggering event or factor, strong family history, and manifestations associated with neurologic and autonomic nervous system dysfunction.

Which statement is true regarding dementia? A. The patient is often the first one to be aware of the problem. B. Onset is usually relatively sudden. C. Initial memory loss consists of long-term memories. D. Thyroid deficiency is ruled out before the diagnosis.

D Screening for cobalamin (vitamin B12) deficiency and hypothyroidism are usually done before making the diagnosis. It is often a family member, particularly the spouse, who reports the patient's declining memory to the heath care provider. The onset is usually insidious and gradual. In dementia, the memory loss initially relates to recent events, with long-term (remote) memories still intact. With time and progression of the dementia, memory loss includes short-term (recent) and long-term (remote) memory.

A patient with a glioma is receiving temozolomide (Temodar). Prior to administration of this drug which laboratory value should you assess because of a common side effect of this drug? A. Potassium B. ALT and AST C. BUN and creatinine D. Platelets

D Temodar causes myelosuppression. Before giving a dose, the absolute neutrophil count should be at least 1500/μL, and the platelet count should be at least 100,000/μL. That is more important than noting the potassium level, liver enzymes, or laboratory results related to kidney function.

Which of the following are characteristic of a tension-type headache? A. The patient experiences an aura. B. They are aggravated by physical activity. C. Nausea and vomiting are present. D. They involve photosensitivity.

D Tension-type headaches are usually of mild or moderate intensity and not aggravated by physical activity. Tension-type headaches are subcategorized as episodic or chronic. Tension-type headaches can last from minutes to days. There is no prodrome (early manifestation of impending disease) in tension-type headache. The headache does not involve nausea or vomiting but may involve sensitivity to light (photophobia) or sound (phonophobia).

The clinical diagnosis of dementia is based on A. computed tomography (CT) or magnetic resonance spectroscopy (MRS). B. brain biopsy. C. electroencephalography. D. patient history and cognitive assessment.

D The diagnosis of dementia focuses on determining the cause. A thorough physical examination is performed to rule out other potential medical conditions. Cognitive testing (e.g., Mini-Mental State Examination) is used to evaluate memory, ability to calculate, language, visual-spatial skills, and degree of alertness. A diagnosis of dementia related to vascular causes is based on cognitive loss, vascular brain lesions demonstrated by neuroimaging techniques, and exclusion of other causes of dementia. Structural neuroimaging with CT or magnetic resonance imaging (MRI) is used to evaluate patients with dementia. A psychologic evaluation is indicated to assess for depression.

The nursing student reports to the nurse in charge that the patient experienced a generalized tonic-clonic seizure. This means that the patient is exhibiting A. a momentary loss of consciousness. B. jerking movements throughout the body. C. rigidity for several seconds, then flaccidity. D. rigidity of muscles followed by muscle jerking.

D The most common generalized seizure is the generalized tonic-clonic (formerly known as grand mal) seizure. Tonic-clonic seizure is characterized by loss of consciousness and falling to the ground if the patient is upright, followed by stiffening of the body (tonic phase) for 10 to 20 seconds and subsequent jerking of the extremities (clonic phase) for another 30 to 40 seconds

What is the best patient to assign to a new graduate nurse on her first week of orientation? A. Patient with bacterial meningitis admitted from the emergency department today B. Patient returning from a craniotomy for a pituitary brain tumor C. Patient with head trauma with suspected epidural bleed admitted 3 hours earlier D. Patient with viral meningitis who is being discharged today

D The new nurse should have the patient who is the most stable and has a predictable outcome. That is the patient with the less serious condition of viral meningitis who is being discharged. The other three patients are potentially unstable and need skilled nursing assessments.

Which statement is true regarding the prevalence of Huntington's disease? A. Death occurs in less than one year. B. Survival rate is less than two years. C. Rates are higher in males than females. D. Onset occurs in the 30 to 50 year old age group.

D The onset of HD is usually between 30 and 50 years of age. HD is a genetically transmitted, autosomal dominant disorder that affects both men and women of all races. Death usually occurs 10 to 20 years after the onset of symptoms. Reference: 1515

What action should you take as part of care for a patient who had a craniotomy? A. Use promethazine (Phenergan) for nausea. B. Position the patient on the operative side if a bone flap was removed. C. Administer phenytoin (Dilantin) by rapid intravenous push (IVP) every 6 hours. D. Keep the head in alignment with the trunk.

D The primary goal of care after cranial surgery is prevention of increased intracranial pressure (ICP), which includes keeping the body in alignment. Use of promethazine is discouraged because it can increase somnolence and alter the accuracy of a neurologic assessment. The patient is not positioned on the operative side if a bone flap was removed (craniectomy). Dilantin is administered slowly, no faster than 25 to 50 mg/min. Reference: 1450

A client with multiple sclerosis is experiencing muscle weakness, spasticity and an ataxic gait. On the basis of this information, the nurse would formulate which of the following nursing diagnoses for the client? A. Self-care deficit B. Risk for activity intolerance C. Impaired tissue integrity D. Impaired physical mobility

D. NANDA Impaired Physical Mobility is defined "a state in which the individual experiences a limitation of ability for independent physical movement." The client's muscle weakness, muscle spasticity, and ataxic gait meet the defining characteristics for this nursing diagnosis.

A nurse would use which standardized tool as a guide in assessing a client with a head injury and increased intracranial pressure (ICP)? A. Snellen chart B. Pulse oximetry graph C. Visual Analogue Scale D. Glasgow Coma Scale

D. GCS is a method of assessing consciousness. The Visual Analogue Scale can be used to determine pain rating. A pulse oximetry graph would be used to document pulse ox readings. A Snellen chart would be used to assess visual acuity.

An older adult client asks the​ nurse, "Why is my body temperature only​ 99°F if I have this serious​ infection?" Which is the​ nurse's best​ response? A. "Your body temperature fluctuates​ significantly, so a true temperature is difficult to​ obtain." B. "I will to take your temperature​ rectally, since it is the only reliable route in somebody your​ age." C. "The true temperature will not register because you are a mouth​ breather." D. "Body temperature in an older adult is not a reliable indicator of the seriousness of an​ illness."

D. "Body temperature in an older adult is not a reliable indicator of the seriousness of an illness." ​Rationale: Body temperature may not be a valid indication of serious illness in an older adult. The older adult may have an infection and exhibit only a slight temperature elevation. Other​ symptoms, such as confusion and​ restlessness, may be present. These require​ follow-up to determine whether an underlying disease process is present. There is no evidence to support that the client is a mouth breather. Rectal temperatures in older adult clients may be contraindicated if hemorrhoids are present. Body temperature in an older adult does not fluctuate significantly.

The patient is prescribed 30 units of regular insulin and 70 units of insulin isophane suspension (NPH insulin) subcutaneously every morning. The nurse should provide which instruction to the patient for insulin administration? A. "Inject the needle at a 30-degree angle." B. "Rotate sites at least once or twice a week." C. "Use a 23- to 25-gauge syringe with a 1-inch needle to increase insulin absorption." D. "Draw up the regular insulin into the syringe first, followed by the cloudy NPH insulin."

D. "Draw up the regular insulin into the syringe first, followed by the cloudy NPH insulin." Correct When insulins are mixed, withdraw the regular insulin (clear) first, followed by withdrawing the NPH insulin (cloudy).

The nurse has been teaching a patient with type 2 diabetes about managing blood glucose levels and taking glipizide (Glucotrol). Which patient statement indicates a need for additional teaching? a. "If I overeat at a meal, I will still take the usual dose of medication." b. "Other medications besides the Glucotrol may affect my blood sugar." c. "When I am ill, I may have to take insulin to control my blood sugar." d. "My diabetes won't cause complications because I don't need insulin."

D. "My diabetes won't cause complications because I don't need insulin."

Successful achievement of patient outcomes for the patient with cranial surgery would be best indicated by the a. ability to return home in 6 days b. ability to meet all self-care needs c. acceptance of residual neurologic deficits d. absence of signs and symptoms of increased ICP

D. Absence of signs and symptoms of increased ICP- The primary goal after cranial surgery is prevention of increased ICP, and interventions to prevent ICP and infection postoperatively are nursing priorities. The residual deficits, rehabilitation potential, and ultimate function of the patient depend on the reason for surgery, the postoperative course, and the patient's general state of health

The nurse suspects the presence of an arterial epidural hematoma in the patient who experiences a. failure to regain consciousness following a head injury b. a rapid deterioration of neurologic function within 24 to 48 hours following a head injury c. nonspecific, nonlocalizing progression of alteration in LOC occurring over weeks or months d. unconsciousness at the time of a head injury with a brief period of consciousness followed by a decrease in LOC

D. An arterial epidural hematoma is the most acute neurologic emergency, and the typical symptoms include unconsciousness at the scene, with a brief lucid interval followed by a decrease in LOC. An acute subdural hematoma manifests signs within 48 hours of an injury; a chronic subdural hematoma develops over weeks or months

What pathophysiological changes should the nurse expect to see in a client with increased intracranial​ pressure? A. Removal of fluid from interstitial spaces reducing excess body fluid B. Transmission of sensory and motor impulses to the cerebrum for interpretation C. Alteration of electrical discharges in the brain to cause involuntary movement D. An oxygen deficit that leads to changes in​ personality, memory, and judgment

D. An oxygen deficit that leads to changes in​ personality, memory, and judgment

Metabolic and nutritional needs of the patient with increased ICP are best met with a. enteral feedings that are low in sodium b. the simple glucose available in D5W IV solutions c. a fluid restriction that promotes a moderate dehydration d. balanced, essential nutrition in a form that the patient can tolerate

D. Balanced, essential nutrition in a form that the patient can tolerate= A patient with increased ICP is in a hypermetabolic and hypercatabolic state and needs adequate glucose to maintain fuel for the brain and other nutrients to meet metabolic needs. Malnutrition promotes cerebral edema, and if a patient cannot take oral nutrition, other means of providing nutrition should be used, such as tube feedings or parenteral nutrition. Glucose alone is not adequate to meet nutritional requirements, and 5% dextrose solutions may increase cerebral edema by lowering serum osmolarity. Patients should remain in a normovolemic fluid state with close monitoring of clinical factors such as urine output, fluid intake, serum and urine osmolality, serum electrolytes, and insensible losses.

When assessing the body function of a patient with increased ICP, the nurse should initially assess a. corneal reflex testing b. extremity strength testing c. pupillary reaction to light d. circulatory and respiratory status

D. Circulatory and respiratory status- Of the body functions that should be assessed in an unconscious patient, cardiopulmonary status is the most vital function and gives priorities to the ABCs (airway, breathing, and circulation)

The nurse is discussing the multiple medications with the family of a client diagnosed with increased intracranial pressure​ (IICP). The family member asks why the client is being given a gastric acid reducer. Which response by the nurse provides the correct ​explanation? A. ​"We use a gastric acid reducer to adhere to ulcer sites and protect them from​ acids, bile​ salts, and​ enzymes." B. ​"Since they are not​ eating, we use a gastric acid reducer to neutralize the acid in their​ stomach." C. ​"A gastric acid reducer helps to protect the inner lining of the stomach from​ ulcer-producing effects." D. ​"There is a higher risk for stress​ ulcers; therefore, we use an acid reducer to block gastric​ secretion."

D. ​"There is a higher risk for stress​ ulcers; therefore, we use an acid reducer to block gastric​ secretion."

The cardinal signs of inflammation are swelling, redness, pain, and heat. What is the 5th cardinal sign? A. Sepsis B. Rubor C. Fever D. Loss of function

D. Loss of function

Which insulin can be administered by continuous intravenous (IV) infusion? A. Insulin glargine (Lantus) B. Insulin aspart (Novolog) C. Insulin detemir (Levemir) D. Regular insulin (Humulin R)

D. Regular insulin (Humulin R) Correct Regular insulin is the only insulin used for IV therapy.

A 20-year-old male was brought to the ER for sever burns. He requested something for the excruciating pain he was experiencing. Blocking which of the following neurotransmitters would reduce his pain? A.Enkephalin B. Dopamine C. Acetylcholine D. Substance P

D. Substance P

The nurse is interviewing a new patient with diabetes who takes rosiglitazone (Avandia). Which information would the nurse anticipate resulting in the health care provider discontinuing the medication? a. The patient's blood pressure is 154/92. b. The patient's blood glucose is 86 mg/dL. c. The patient reports a history of emphysema. d. The patient has chest pressure when walking.

D. The patient has chest pressure when walking.

A 28-yr-old male patient with type 1 diabetes reports how he manages his exercise and glucose control. Which behavior indicates that the nurse should implement additional teaching? a. The patient always carries hard candies when engaging in exercise. b. The patient goes for a vigorous walk when his glucose is 200 mg/dL. c. The patient has a peanut butter sandwich before going for a bicycle ride. d. The patient increases daily exercise when ketones are present in the urine.

D. The patient increases daily exercise when ketones are present in the urine.

Which finding indicates a need to contact the health care provider before the nurse administers metformin (Glucophage)? a. The patient's blood glucose level is 174 mg/dL. b. The patient is scheduled for a chest x-ray in an hour. c. The patient has gained 2 lb (0.9 kg) in the past 24 hours. d. The patient's estimated glomerular filtration rate is 42 mL/min.

D. The patient's estimated glomerular filtration rate is 42 mL/min.

Skull radiographs and a computed tomography (CT) scan provide evidence of a depressed parietal fracture with a subdural hematoma in a patient admitted to the emergency department following an automobile accident. In planning care for the patient, the nurse anticipates that a. the patient will receive life-support measures until the condition stabilizes b. immediate burr holes will be made to rapidly decompress the intracranial activity c. the patient will be treated conservatively with close monitoring for changes in neurologic condition d. the patient will be taken to surgery for a craniotomy for evacuation of blood and decompression of the cranium

D. When there is a depressed fracture and fractures with loose fragments, a craniotomy is indicated to elevate the depressed bone and remove free fragments. A craniotomy is also indicated in cases of acute subdural and epidural hematomas to remove the blood and control the bleeding. Burr holes may be used in an extreme emergency for rapid decompression, but with a depressed fracture, surgery would be the treatment of choice

A patients presents with altered respiratory patterns following head trauma. Based upon the symptoms, which of the following areas does the nurse suspect is injured? A. cerebrum B. cerebellum C. midbrain D. reticular formation

D. reticular formation

Which causes the redness (erythema) associated with the inflammatory process? A. Prostaglandins B. Histamine C. Macrophages D. All of the above E. A and B

E. A and B

9. When a patient asks about target cell receptors, which is the nurses best response? Target cell receptors for most water-soluble hormones are located in the: a. Cytosol b. Cell membrane c. Endoplasmic reticulum d. Nucleus

ANS: B Water-soluble hormones bind to cell surface receptors.

A client with a stroke is demonstrating signs of increasing intracranial pressure. Which actions should the nurse take at this​ time? ​(Select all that​ apply.) a. Provide hypotonic fluids b. Reduce environmental stimuli c. Assess cranial nerve function d. Monitor pupillary response e. Assess vital signs

b. Reduce environmental stimuli c. Assess cranial nerve function d. Monitor pupillary response e. Assess vital signs Rationale Nursing actions for the client demonstrating signs of increasing intracranial pressure include assessing vital​ signs, monitoring pupillary​ response, assessing cranial nerve​ function, and reducing environmental stimuli. Intravenous fluids administered at this time would be isotonic or hypertonic.

19. Deficiencies in which element can produce depression of both B- and T-cell function? a. Iron b. Zinc c. Iodine d. Magnesium

ANS: B Of the options available, only deficient zinc intake can profoundly depress T- and B-cell function.

2. According to the Center for Disease Control and Prevention (CDC) guidelines, which personal protective equipment will the nurse put on before assessing a patient who is on contact precautions for Clostridium difficile diarrhea (select all that apply)? a. Mask b. Gown c. Gloves d. Shoe covers e. Eye protection

ANS: B, C Because the nurse will have substantial contact with the patient and bedding when doing an assessment, gloves and gowns are needed. Eye protection and masks are needed for patients in contact precautions only when spraying or splashing is anticipated. Shoe covers are not recommended in the CDC guidelines. DIF: Cognitive Level: Apply (application) REF: 218 TOP: Nursing Process: Implementation MSC: NCLEX:

*Following the initiation of a pain management plan, pain should be reassessed and documented on a regular basis as a way to evaluate the effectiveness of treatments. Pain should be reassessed at which minimum interval?* A. With each new report of pain B. Before and after administration of narcotic analgesics C. Every 10 minutes D. Every shift

*Answer: A & B* Rationale: Following the initiation of a pain management plan, pain should be reassessed and documented on a regular basis as a way to evaluate the effectiveness of treatments. At a minimum, pain should be reassessed with each new report of pain and before and after administration of analgesics.

*Which of the following is a priority for a nurse to include in a teaching plan for a patient who desires self-management and alternative strategies?* A. Body alignment and superficial heat and cooling B. Patient-controlled analgesia (PCA) pump C. Neurostimulation D. Peripheral nerve blocks

*Answer: A* Rationale: Body alignment and thermal management are examples of nonpharmacological measures to manage pain. They can be used individually or in combination with other nondrug therapies. Proper body alignment achieved through proper positioning can help prevent or relieve pain. Thermal measures such as the application of localized, superficial heat and cooling may relieve pain and provide comfort. PCA, neurostimulation, and peripheral nerve blocks are not totally self-managed or alternative therapies, because they are used under the direction of medical professionals.

*Stephanie is a 70-year-old retired schoolteacher who is interested in nondrug, mind-body therapies, self-management, and alternative strategies to deal with joint discomfort from rheumatoid arthritis. Which of the following options should you suggest for her plan of care, considering her expressed wishes?* A. Using a stationary exercise bicycle and free weights and attending a spinning class B. Using mind-body therapies such as music therapy, distraction techniques, meditation, prayer, hypnosis, guided imagery, relaxation techniques, and pet therapy C. Drinking chamomile tea and applying icy/hot gel D. Receiving acupuncture and attending church services

*Answer: B* Rationale: Mind-body therapies are designed to enhance the mind's capacity to affect bodily functions and symptoms and include music therapy, distraction techniques, meditation, prayer, hypnosis, guided imagery, relaxation techniques, and pet therapy, among many others. Although getting exercise, drinking chamomile tea and applying gels, and receiving acupuncture and attending church services may be beneficial, they are not classified as mind-body therapies in combination as specified in these answer choices.

*A 65-year-old woman has fallen while sweeping her driveway, sustaining a tissue injury. She describes her condition as an aching, throbbing back. Which type of pain are these complaints most indicative of?* A. Neuropathic pain B. Nociceptive pain C. Chronic pain D. Mixed pain syndrome

*Answer: B* Rationale: Nociceptive pain refers to the normal functioning of physiological systems that leads to the perception of noxious stimuli (tissue injury) as being painful. Patients describe this type of pain as dull or aching, and it is poorly localized. Neuropathic pain is described as shooting, tingling, burning, or numbness that is constant in the extremities, as in diabetic neuropathy. Chronic pain lasts longer than 30 days and is characterized by a disease affecting brain structure and function, such as chronic headaches or open wounds. Mixed pain syndromes are caused by different pathophysiological mechanisms such as a combination of neuropathic and nociceptive pain; this occurs in syndromes such as sciatica, spinal cord injuries, and cervical or lumbar spinal stenosis.

*Postoperative surgical patients should be given alternating doses of acetaminophen and which medication throughout the postoperative course, unless contraindicated?* A. Antihistamine B. Local anesthetic C. Opioids D. Nonsteroidal anti-inflammatory drug (NSAID)

*Answer: D* Rationale: Unless contraindicated, all surgical patients should routinely be given acetaminophen and an NSAID in scheduled doses throughout the postoperative course. Opioid analgesics are added to the treatment plan to manage moderate-to-severe postoperative pain. A local anesthetic is sometimes administered epidurally or by continuous peripheral nerve block.

The primary goal for the treatment of restless legs syndrome is to A. reduce episodes. B. improve sleep quality. C. increase exercise tolerance. D. increase iron stores.

The goal of collaborative management is to reduce patient discomfort and distress and to improve sleep quality. When RLS is secondary to renal failure or iron deficiency, correction of these conditions will decrease symptoms. Reference: 1514

A client with an endocrine disorder complains of weight loss and diarrhea, and says that he can "feel his heart beating in his chest." The nurse interprets that which gland is most likely responsible for these symptoms? 1. Thyroid 2. Pituitary 3. Parathyroid 4. Adrenal cortex

1

During physical examination of a client, which finding is characteristic of hypothyroidism? 1. Periorbital edema 2. Flushed warm skin 3. Hyperactive bowel sounds 4. Heart rate of 120 beats/min

1

The nurse is developing a plan of care for an older client that addresses interventions to prevent cold discomfort and the development of accidental hypothermia. The nurse should document which desired outcome in the plan of care? 1. The client's body temperature is 98° F. 2. The client's fingers and toes are cool to touch. 3. The client remains in a fetal position when in bed. 4. The client complains of coolness in the hands and feet only.

1

The nurse is monitoring a client with Graves' disease for signs of thyrotoxic crisis (thyroid storm). Which signs or symptoms, if noted in the client, will alert the nurse to the presence of this crisis? 1. Fever and tachycardia 2. Pallor and tachycardia 3. Agitation and bradycardia 4. Restlessness and bradycardia

1

The nurse is performing a physical assessment on a client during her first prenatal visit to the clinic. The nurse takes the client's temperature and notes that the temperature is 99.2° F. Based on this finding, which nursing action is most appropriate? 1. Document the temperature. 2. Notify the health care provider. 3. Retake the temperature by the rectal route. 4. Inform the client that the temperature is elevated and antibiotics may be required.

1

An adolescent who has been prescribed prednisone (Meticorten) and vincristine (Oncovin) for leukemia tells the nurse that he is very constipated. What should the nurse cite as the probable cause of the constipation? 1 It is a side effect of the vincristine. 2 The spleen is compressing the bowel. 3 It is a toxic effect from the prednisone. 4 The leukemic mass is obstructing the bowel.

1. Constipation is a side effect of vincristine (Oncovin) because it slows gastrointestinal motility. An enlarged spleen will put pressure on the stomach and diaphragm, not on the large bowel. Constipation is not a toxic effect of prednisone (Meticorten). It is unlikely that leukemia is causing an obstruction.

A 55-year-old female is admitted to the medical unit for complications of long-term, poorly controlled type 2 DM. Which of the following would the nurse expect to find in addition to elevated glucose? a. Atherosclerosis b. Metabolic alkalosis c. Elevated liver enzymes d. Anemia

a

A client newly diagnosed with type 2 diabetes is receiving glyburide (Micronase) and asks the nurse how this drug works. The nurse explains that glyburide: 1 Stimulates the pancreas to produce insulin 2 Accelerates the liver's release of stored glycogen 3 Increases glucose transport across the cell membrane 4 Lowers blood glucose in the absence of pancreatic function

1. Glyburide, an antidiabetic sulfonylurea, stimulates insulin production by the beta cells of the pancreas. Accelerating the liver's release of stored glycogen occurs when serum glucose drops below normal levels. Increasing glucose transport across the cell membrane occurs in the presence of insulin and potassium. Antidiabetic medications of the chemical class of biguanide improve sensitivity of peripheral tissue to insulin, which ultimately increases glucose transport into cells. Beta cells must have some function to enable this drug to be effective.

The concept of immunity is most closely related to which other concepts? (Select all that apply.) a. Protection b. Integrity c. Inflammation d. Restoration e. Infection

a, c, e The concept of immunity is most closely related to the concepts of protection, because protection is its primary function, and inflammation and infection. It is related to these last two concepts because of overlap between the pathologic, diagnostic, and laboratory tests; clinical findings; and nursing care and outcomes for these and immunity.

The nurse is caring for a child after surgical removal of a brain tumor. The nurse should assess the child for which sign that would indicate that brainstem involvement occurred during the surgical procedure? 1. Inability to swallow 2. Elevated temperature 3. Altered hearing ability 4. Orthostatic hypotension

2

2. Which types(s) of isolation precautions is (are) appropriate for a patient with tuberculosis (select all that apply)? a. contact precautions b. droplet precautions c. airborne precautions d. standard precautions e. neutropenic precautions

2. Correct answers: c, d Standard precautions should be used for all patients. In addition to standard precautions, patients with TB should be placed on airborne precautions to minimize risk of transmission and infection of HCPs and visitors.

A client with history of multiple chronic illnesses comes to the emergency department (ED) complaining of a small progressive weight loss over the last month and feeling lethargic and thirsty all the time. The client's fasting blood glucose is 180 mg/dL and vital signs are blood pressure (BP) 118/78 mm Hg, oral temperature 99.6º F, pulse 72 beats per minute and regular, and respirations 22 breaths per minute and irregular. The nurse reviews the assessment findings and the client's medical record. What condition does the nurse conclude the client is experiencing? 1.Hypervolemia 2.Hyperglycemia 3.Infectious process 4.Respiratory distress

2.Hyperglycemia

A nurse is caring for a client with a diagnosis of type 1 diabetes who has developed diabetic coma. Which element excessively accumulates in the blood to precipitate the signs and symptoms associated with this condition? 1.Sodium bicarbonate, causing alkalosis 2.Ketones as a result of rapid fat breakdown, causing acidosis 3.Nitrogen from protein catabolism, causing ammonia intoxication 4.Glucose from rapid carbohydrate metabolism, causing drowsiness

2.Ketones as a result of rapid fat breakdown, causing acidosis

16. The nurse prepares to administer the following medications to a hospitalized patient with human immunodeficiency (HIV). Which medication is most important to administer at the scheduled time? a. Nystatin tablet b. Oral acyclovir (Zovirax) c. Oral saquinavir (Invirase) d. Aerosolized pentamidine (NebuPent)

ANS: C It is important that antiretrovirals be taken at the prescribed time every day to avoid developing drug-resistant HIV. The other medications should also be given as close as possible to the correct time, but they are not as essential to receive at the same time every day. DIF: Cognitive Level: Analyze (analysis) REF: 228 TOP: Nursing Process: Implementation MSC:

A previously asymptomatic 84-year-old client presents with aphasia but is otherwise alert and responsive. The nurse would suspect that this client has which condition? a. A stroke in the left cerebral hemisphere b. Damage to the brainstem c. A stroke in the right cerebral hemisphere d. A lesion in the occipital lobe

a. A stroke in the left cerebral hemisphere Rationale Aphasia, defective or absent language function, can occur as a result of a stroke in speech center of the brain located in the left cerebral hemisphere. An individual with damage to the brainstem would present with marked cognitive, motor, and sensory dysfunction and a client with a lesion in the occipital area of the brain would manifest problems with vision.

A newborn is delivered via spontaneous vaginal delivery. On reception of the crying newborn, the nurse's priority is to perform which action? 1. Determine Apgar score. 2. Auscultate the heart rate. 3. Thoroughly dry the newborn. 4. Take the newborn's rectal temperature.

3

A nurse is caring for a client with a dysfunctional thyroid gland and is concerned that the client will exhibit signs of thyroid storm. Which is an early indicator of this complication? 1. Constipation 2. Bradycardia 3. Hyperreflexia 4. Low-grade temperature

3

A nurse is caring for a client with a thyrotoxicosis who is at risk for the development of thyroid storm. To detect this complication, the nurse should assess for which sign or symptom? 1. Bradycardia 2. Constipation 3. Hypertension 4. Low-grade temperature

3

The clinic nurse is caring for an infant who has been diagnosed with primary hypothyroidism. The nurse is reviewing the results of the laboratory tests for a T4 and thyroid-stimulating (TSH) hormone. Which laboratory finding indicates a diagnosis of primary hypothyroidism? 1. A normal T4 level 2. An elevated T4 level 3. An elevated TSH level 4. A decreased TSH level

3

The nurse assisted with the delivery of a newborn. Which nursing action is most effective in preventing heat loss by evaporation? 1. Warming the crib pad 2. Closing the doors to the room 3. Drying the infant with a warm blanket 4. Turning on the overhead radiant warmer

3

The nurse in the newborn nursery is preparing to complete an initial assessment on a newborn infant who was just admitted to the nursery. The nurse should place a warm blanket on the examining table to prevent heat loss in the infant caused by which method? 1. Radiation 2. Convection 3. Conduction 4. Evaporation

3

Thyroid replacement therapy is prescribed for the client diagnosed with hypothyroidism. The client asks the nurse when the medication will no longer be needed. Which is the appropriate nursing response? 1. "It depends on the results of the laboratory tests." 2. "Most clients require medication for about 1 year." 3. "The medication will need to be continued for life." 4. "You will need to ask your health care provider."

3

Which interventions are appropriate when administering a tepid bath to a child with a fever? Select all that apply. 1. Allow the child's skin to air dry. 2. Apply alcohol-soaked cloths over the child's body. 3. Use a water toy to distract the child during the bath. 4. Place lightweight pajamas on the child after the bath. 5. Squeeze water over the child's body, using the washcloth.

3, 4, 5

A 40-year-old male is prescribed Metformin XL (Glucophage) to control his type 2 diabetes mellitus. Which statement made by this client indicates the need for further education? 1"I will take the drug with food." 2 "I must swallow my medication whole and not crush or chew it." 3 "I will stop taking Metformin for 24 hours before and after having a test involving dye." 4 "I will notify my doctor if I develop muscular or abdominal discomfort."

3. Metformin must be withheld for 48 hours before the use of iodinated contrast materials to prevent lactic acidosis. Metformin is restarted when kidney function has returned to normal. Metformin is taken with food to avoid adverse gastrointestinal effects. If crushed or chewed, Metformin XL will be released too rapidly and may lead to hypoglycemia. Muscular and abdominal discomfort is a potential sign of lactic acidosis and must be reported to the health care provider.

A client with a minor head injury has a Glasgow Coma score of 15. What does this score indicate to the​ nurse? ​(Select all that​ apply.) a. Client uses appropriate words and phrases. b. Client spontaneously opens the eyes. c. Client is oriented to​ person, place, and time. d. Client withdraws to pain. e. Client withdraws to touch.

a. Client uses appropriate words and phrases. b. Client spontaneously opens the eyes. c. Client is oriented to​ person, place, and time. Rationale The maximum Glasgow Coma score is 15. This means that the client uses appropriate words and​ phrases, spontaneously opens the​ eyes, and is oriented to​ person, place, and time. Withdrawing to pain or touch would cause the Glasgow Coma score to be less than 15.

A nurse is giving a presentation at the local community center regarding the prevention of vision loss. She wants to discuss modifiable risk factors to help people protect their eyes and vision. What modifiable risk factors would be discussed during this​ presentation? ​(Select all that​ apply.) a. Ultraviolet light exposure b. Smoking c. Annual eye exams d. Isolation e. Eye injuries

a. Ultraviolet light exposure b. Smoking e. Eye injuries Rationale Modifiable risk factors for vision loss include decreasing ultraviolet light​ exposure, not​ smoking, and preventing eye injuries by using protective eye wear. Isolation of adults is associated with sensory deprivation in both infants and in adults. Isolation is a modifiable risk​ factor, but it is not associated with vision loss. Annual eye exams may detect early changes in vision​ loss, but are not considered a risk factor.

The mother of an adolescent daughter wants to know what they should look for in case the daughter gets hit in the head playing soccer. The nurse notes that four areas of functioning should be assessed if a concussion is suspected, including physical, cognitive, and emotional. The fourth area of functioning to assess is: a. Sleep b. School performance c. Musculoskeletal complaints d. Personality

a. Sleep Rationale Sleep is the fourth area of functioning that the mother and daughter should monitor. Individuals experiencing concussion may be drowsy, sleep more or less than usual, or may have trouble falling asleep.

Which condition suppresses immunity and places the patient at increased risk for infection? a. Stress b. Hypertension c. Edema d. Depression

a. Stress Stress increases the production of cortisol. Cortisol suppresses immunity and increases the risk for infection. Option B is incorrect because hypertension is not associated with immune suppression. Option C is incorrect because edema does not suppress immunity but could be a sign of inflammation or infection. Option D is incorrect because depression is not associated with immune suppression.

A woman brings in her​ 82-year-old mother for a​ check-up at the local clinic. The woman reports that her mother does not respond to her appropriately at​ times, and that she seems to be forgetful. The nurse wants to do a complete​ sensory-perceptual functioning assessment. What components would the nurse include in her​ assessment? ​(Select all that​ apply.) a. The whisper test b. Asking the daughter about the client​'s social support network c. Status of the client​'s insurance d. Discussing the client​'s history of wearing hearing aids e. Mental status exam

a. The whisper test b. Asking the daughter about the client​'s social support network d. Discussing the client​'s history of wearing hearing aids e. Mental status exam Rationale A complete nursing assessment of the​ sensory-perceptual functioning includes a client​ history, a mental​ exam, a physical​ exam, social support​ network, the client​'s ​environment, and identifying people at high risk. The whisper test would be included in the physical exam. The client​'s history of wearing hearing aids would be appropriate in collection of the client​'s medical history. A mental status exam would help the nurse determine whether the client answering her daughter inappropriately is due to senility or a sensory deficit. Insurance status would not be appropriate during the assessment of the client​'s sensory function.

After a nursing​ assessment, the nurse documents that a client is confused. Which behaviors did the nurse assess to determine this client​'s level of​ consciousness? ​(Select all that​ apply.) a. Uses inappropriate words to describe situations b. Moans in response to painful stimuli c. Does not know why hospitalization is required d. Responds to verbal stimuli but quickly falls back asleep e. Does not remember home address

a. Uses inappropriate words to describe situations c. Does not know why hospitalization is required e. Does not remember home address Rationale Confusion is the inability to think rapidly and clearly. Additional characteristics include easily​ bewildered, poor​ memory, short attention​ span, misinterprets stimuli and impaired judgment. Semicomatose is moaning in response to painful stimuli. Obtundation is responding to verbal stimuli but quickly falling asleep.

The nurse is preparing to conduct a neurologic assessment interview with a client. Which general question should the nurse ask when conducting this​ assessment? (Select all that​ apply.) A. ​"Are you having any problems with your​ memory?" B. ​"Do you have any problems with balance or​ coordination?" C. ​"Are you experiencing any​ pain?" D. ​"How many fingers am I holding up at this​ time?" E. ​"Do you have a history of seizures or​ fainting?"

A. ​"Are you having any problems with your​ memory?" B. ​"Do you have any problems with balance or​ coordination?" C. ​"Are you experiencing any​ pain?" E. ​"Do you have a history of seizures or​ fainting?"

11. Which information about a patient population would be most useful to help the nurse plan for human immunodeficiency virus (HIV) testing needs? a. Age c. Symptoms b. Lifestyle d. Sexual orientation

ANS: A The current Centers for Disease Control and Prevention policy is to offer routine testing for HIV to all individuals age 13 to 64 years. Although lifestyle, symptoms, and sexual orientation may suggest increased risk for HIV infection, the goal is to test all individuals in this age range. DIF: Cognitive Level: Apply (application) REF: 226 TOP: Nursing Process: Planning MSC:

27. A child in the clinic has an absence of a parathyroid gland, structural heart defects, and a shortened structure of the upper lip. What immune dysfunction does the healthcare professional suspect? a. Partial-to-complete absence of T-cell immunity b. X-linked recessive microcephaly c. An autoimmune disease like systemic lupus erythematosus d. Adenosine deaminase deficiency

ANS: A This child has the clinical features of DiGeorge disease, which is caused by a partial-to-complete absence of T-cell immunity.

A 75-year-old woman with type 2 diabetes has recently been placed on glipizide (Glucotrol), 10 mg daily. She asks the nurse when the best time would be to take this medication. What is the nurse's best response? a. "Take this medication in the morning, 30 minutes before breakfast." b. "Take this medication in the evening with a snack." c. "This medication needs to be taken after the midday meal." d. "It does not matter what time of day you take this medication."

ANS: A Glipizide is taken in the morning, 30 minutes before breakfast. When taken at this time, it has a longer duration of action, causing a constant amount of insulin to be released. This may be beneficial in controlling blood glucose levels throughout the day. DIF: COGNITIVE LEVEL: Applying (Application) REF: p. 511 TOP: NURSING PROCESS: Implementation MSC: NCLEX: Physiological Integrity: Pharmacological and Parenteral Therapies

The nurse is administering insulin lispro (Humalog) and will keep in mind that this insulin will start to have an effect within which time frame? a. 15 minutes b. 1 to 2 hours c. 80 minutes d. 3 to 5 hours

ANS: A The onset of action for insulin lispro is 15 minutes. The peak plasma concentration is 1 to 2 hours; the elimination half-life is 80 minutes; and the duration of action is 3 to 5 hours. DIF: COGNITIVE LEVEL: Remembering (Knowledge) REF: p. 516 TOP: NURSING PROCESS: Implementation MSC: NCLEX: Physiological Integrity: Pharmacological and Parenteral Therapies

Which of the following would be included in the assessment of a patient with diabetes mellitus who is experiencing a hypoglycemic reaction? (Select all that apply.) a. Tremors b. Nervousness c. Extreme thirst d. Flushed skin e. Profuse perspiration f. Constricted pupils

ANS: A, B, E When hypoglycemia occurs, blood glucose levels fall, resulting in sympathetic nervous system responses such as tremors, nervousness, and profuse perspiration. Dilated pupils would also occur, not constricted pupils. Extreme thirst, flushed skin, and constricted pupils are consistent with hyperglycemia.

4. What are blood transfusion reactions an example of? a. Autoimmunity b. Alloimmunity c. Homoimmunity d. Hypersensitivity

ANS: B Alloimmunity (also termed isoimmunity) occurs when the immune system of one individual produces an immunologic reaction against tissues of another individual. Autoimmunity is a disturbance in the immunologic tolerance of self-antigens. Homoimmunity refers to the resistance of a lysogenic bacterium that is carrying a phage to an infection by the same type of phage. Hypersensitivity is an altered immunologic response to an antigen that results in disease or damage to the host.

20. An Rh-negative woman gave birth to an Rh-positive baby. When discussing Rho[D] immunoglobulin with her, what information should the healthcare professional provide? a. It provides protection against infection from poor immunity in the baby. b. It prevents alloimmunity and hemolytic anemia of the newborn. c. It provides necessary antibodies in case the mother doesn't breastfeed. d. It causes the intestinal tract of the newborn to produce antibodies.

ANS: B Alloimmunity occurs when an individual's immune system reacts against antigens on the tissues of other members of the same species. This can occur when a woman is Rh-negative and gives birth to an Rh-positive baby, leading to hemolytic anemia of the newborn. Rho[D] immunoglobulin does not provide protection against infection, provide antibodies to a bottle-fed baby, or cause the intestine to produce antibodies.

15. Type III hypersensitivity reactions are a result of which of these? a. Antibodies coating mast cells by binding to receptors that signal its degranulation, followed by the discharge of preformed mediators b. Antibodies binding to soluble antigens that were released into body fluids and the immune complexes being deposited in the tissues c. Tc cells or lymphokine-producing Th1 cells directly attacking and destroying cellular targets d. Antibodies binding to the antigen on the cell surface

ANS: B Antigen-antibody (immune) complexes that are formed in the circulation and then deposited later in vessel walls or extravascular tissues cause most type III hypersensitivity diseases. Type III hypersensitivity reactions are not the result of antibodies coating mast cells to signal their degranulation, immune cells directly attacking and destroying targets, or antibodies binding to the antigen on the cell surface.

5. A pregnant woman with asymptomatic chronic human immunodeficiency virus (HIV) infection is seen at the clinic. The patient states, "I am very nervous about making my baby sick." Which information will the nurse include when teaching the patient? a. The antiretroviral medications used to treat HIV infection are teratogenic. b. Most infants born to HIV-positive mothers are not infected with the virus. c. Because it is an early stage of HIV infection, the infant will not contract HIV. d. Her newborn will be born with HIV unless she uses antiretroviral therapy (ART).

ANS: B Only 25% of infants born to HIV-positive mothers develop HIV infection, even when the mother does not use ART during pregnancy. The percentage drops to 2% when ART is used. Perinatal transmission can occur at any stage of HIV infection (although it is less likely to occur when the viral load is lower). ART can safely be used in pregnancy, although some ART drugs should be avoided. DIF: Cognitive Level: Understand (comprehension) REF: 219 TOP: Nursing Process: Implementation MSC:

10. The nurse palpates enlarged cervical lymph nodes on a patient diagnosed with acute human immunodeficiency virus (HIV) infection. Which action would be appropriate for the nurse to take? a. Instruct the patient to apply ice to the neck. b. Explain to the patient that this is an expected finding. c. Request that an antibiotic be prescribed for the patient. d. Advise the patient that this indicates influenza infection.

ANS: B Persistent generalized lymphadenopathy is common in the early stages of HIV infection. No antibiotic is needed because the enlarged nodes are probably not caused by bacteria. Lymphadenopathy is common with acute HIV infection and is therefore not likely the flu. Ice will not decrease the swelling in persistent generalized lymphadenopathy DIF: Cognitive Level: Apply (application) REF: 225 TOP: Nursing Process: Assessment MSC:

A patient in the outpatient setting was diagnosed with atopic dermatitis. What interventions will the plan of care focus primarily on? a. Decreasing pain b. Decreasing pruritus c. Preventing infection d. Promoting drying of lesions

ANS: B Pruritus is the major manifestation of atopic dermatitis and causes the greatest morbidity. The urge to scratch may be mild and self-limiting, or it may be intense, leading to severely excoriated lesions, infection, and scarring. REF: Page 267

The nurse is reviewing a patient's medication list and notes that sitagliptin (Januvia) is ordered. The nurse will question an additional order for which drug or drug class? a. Glitazone b. Insulin c. Metformin (Glucophage) d. Sulfonylurea

ANS: B Sitagliptin is indicated for management of type 2 diabetes either as monotherapy or in combination with metformin, a sulfonylurea, or a glitazone, but not with insulin. DIF: COGNITIVE LEVEL: Applying (Application) REF: p. 512 TOP: NURSING PROCESS: Implementation MSC: NCLEX: Physiological Integrity: Pharmacological and Parenteral Therapies

After starting treatment for type 2 diabetes mellitus 6 months earlier, a patient is in the office for a follow-up examination. The nurse will monitor which laboratory test to evaluate the patient's adherence to the antidiabetic therapy over the past few months? a. Hemoglobin levels b. Hemoglobin A1C level c. Fingerstick fasting blood glucose level d. Serum insulin levels

ANS: B The hemoglobin A1C level reflects the patient's adherence to the therapy regimen for several months previously, thus evaluating how well the patient has been doing with diet and drug therapy. The other options are incorrect. DIF: COGNITIVE LEVEL: Applying (Application) REF: p. 517 TOP: NURSING PROCESS: Evaluation MSC: NCLEX: Health Promotion and Maintenance

The insulin order reads, "Give 10 units of NPH insulin and 5 units of regular insulin, subcut, every morning before breakfast." Choose the proper syringe for this injection. a. 1 mL syringe b. 100 unit syringe c. 3 mL syringe d. 5 mL syringe

ANS: B The proper syringe for insulin injection is the insulin syringe, which is marked in units. The other syringes listed are not correct for use with insulin because they are not marked in units. DIF: COGNITIVE LEVEL: Analyzing (Analysis) REF: p. 506 TOP: NURSING PROCESS: Planning MSC: NCLEX: Safe and Effective Care Environment: Safety and Infection Control

11. An aide asks the nurse what activates tyrosine. What is the nurses best response? a. GH b. PRL c. Insulin d. Estrogen

ANS: C Insulin receptor binding activates tyrosine kinase autophosphorylation and sends a cascade of signals to activate glucose transporters. Insulin binding, not growth hormone, activates tyrosine. Insulin, not PRL, activates tyrosine. Insulin, not estrogen, activates tyrosine.

1. How is hypersensitivity best defined? a. A disturbance in the immunologic tolerance of self-antigens b. An immunologic reaction of one person to the tissue of another person c. An altered immunologic response to an antigen that results in disease d. An undetectable immune response in the presence of antigens

ANS: C Hypersensitivity is an altered immunologic response to an antigen that results in disease or damage to the host. Autoimmunity is a disturbance in the immunologic tolerance of self-antigens. Alloimmunity is the immunologic reaction of one person to the tissue of another person. An immune deficiency of some type would cause undetectable immune response in the presence of antigens.

A patient is to receive phototherapy for the treatment of psoriasis. What is the nursing priority for this patient? a. Obtaining a complete blood count (CBC) b. Protection from excessive heat c. Protection from excessive ultraviolet (UV) exposure d. Instructing the patient to take their multivitamin prior to treatment

ANS: C Protection from excessive UV exposure is important to prevent tissue damage. Protection from heat is not the most important priority for this patient. There is no need for vitamins or a CBC for patients with psoriasis. REF: Page 267

An older patient has developed age spots and is concerned about skin cancer. How would the nurse instruct the patient to perform skin checks to assess for signs of skin cancer? a. "Limit the time you spend in the sun." b. "Monitor for signs of infection." c. "Monitor spots for color change." d. "Use skin creams to prevent drying."

ANS: C The ABCD method (check for asymmetry, border irregularity, color variation, and diameter) should be used to assess lesions for signs associated with cancer. Color change could be a sign of cancer and needs to be looked at by a dermatologist. Limiting time spent in the sun is a preventative measure but will not assist the patient in checking the skin or detecting skin cancer. Infection is usually not found in skin cancer. Skin creams have not been shown to prevent cancer nor would they assist in detecting skin cancer. REF: Page 266

25. Which class of immunoglobulins forms isohemagglutinins? a. IgA b. IgE c. IgG d. IgM

ANS: D Naturally occurring antibodies, called isohemagglutinins, are immunoglobulins of only the IgM class.

26. Which component of the immune system is deficient in individuals with infections caused by viruses, fungi, or yeast? a. Natural killer cells b. Macrophages c. B cells d. T cells

ANS: D Of the available options, deficiencies in T-cell immune responses are suggested when certain viruses (e.g., varicella, vaccinia, herpes, cytomegalovirus), fungi, and yeasts (e.g., Candida, Histoplasma) or certain atypical microorganisms (e.g., Pneumocystis jiroveci) cause recurrent infections.

A patient asks the nurse what the purpose of the Wood's light is. Which response by the nurse is accurate? a. "We will put an anesthetic on your skin to prevent pain." b. "The lamp can help detect skin cancers." c. "Some patients feel a pressure-like sensation." d. "It is used to identify the presence of infectious organisms and proteins associated with specific skin conditions."

ANS: D The Wood's light examination is the use of a black light and darkened room to assist with physical examination of the skin. The examination does not cause discomfort. REF: Page 265

What characteristic differentiates delirium from dementia? A. Answers orientation questions with "I don't know" B. Altered alertness (hypovigilant or hypervigilant) C. Has a consistent course D. Is treated with donepezil (Aricept)

B Delirium is characterized by an acute onset, a fluctuating course, inattention, disorganized thinking, and altered level of consciousness. Dementia is characterized by a normal level or alertness but confusion. Answers to orientation questions such as "I don't know" are elicited from persons with depression. Dementia has a consistent course; delirium fluctuates. Dementia is treated with Aricept. Treatment of delirium focuses on the treatment of precipitating factors.

The patient has been receiving scheduled doses of phenytoin (Dilantin) and begins to experience diplopia. You immediately assess the patient for A. an aura. B. nystagmus or confusion. C. abdominal pain or cramping. D. irregular pulse or palpitations

B Diplopia is a sign of phenytoin toxicity. You should assess for other signs of toxicity, which include neurologic changes such as nystagmus, ataxia, confusion, dizziness, or slurred speech. Reference: 1496

For a patient with a suspected stroke, which important piece of information should you obtain? A. Time of the patient's last meal B. Time at which stroke symptoms first appeared C. Patient's hypertension history and management D. Family history of stroke and other cardiovascular diseases

B During initial evaluation, the single most important point in the patient's history is the time of onset of stroke symptoms. If the stroke is ischemic, recombinant tissue plasminogen activator (tPA) must be administered within 3 to 4.5 hours of the onset of clinical signs; tPA reestablishes blood flow through a blocked artery and prevents brain cell death in patients with an acute onset of symptoms.

What is the difference between viral and bacterial meningitis? A. Viral meningitis is treated with sulfa drugs, and bacterial meningitis is treated with penicillin. B. Full recovery is expected with the viral form, but more serious residual effects are seen with the bacterial form. C. Viral meningitis is considered more ominous because serious brain involvement is experienced. D. Typical signs of viral meningitis are diplopia and fasciculations, whereas signs of bacterial meningitis are fever and nuchal rigidity.

B Full recovery is expected from viral meningitis, and any residual effects are rare and minor compared with those of bacterial meningitis. The bacterial form can cause dementia, seizures, deafness, hemiplegia, and hydrocephalus. Viral meningitis is managed symptomatically because the disease is self-limiting, a full recovery is expected, and there usually is no brain involvement. Antibiotics are not used for viruses. Both forms manifest with the symptoms of headache, fever, photophobia, and stiff neck.

The patient had an ischemic stroke and is undergoing rehabilitation. He is diagnosed with homonymous hemianopsia. What should you do? A. Better arrange the environment to suit the patient's needs. B. Teach the patient to turn his head to scan the environment. C. Obtain prescriptive glasses for the patient. D. Have the patient wear an eye patch.

B Homonymous hemianopsia (blindness in the same half of each visual field) is a common problem. Persistent disregard of objects in part of the visual field should alert you to this possibility. In rehabilitation, the patient should learn to compensate by consciously attending to or by scanning the neglected side. Early immediate intervention involves arranging the environment within the patient's perceptual field. Glasses do not help the problem. Diplopia (double vision) can be a problem and is treated with an eye patch.

During the postictal period of a seizure, you would expect the patient to A. demonstrate minor jerking and eye fluttering. B. sleep for several hours. C. be incontinent of urine and feces. D. require ventilator assistance.

B In the postictal phase the patient usually has muscle soreness, is very tired, and may sleep for several hours. Some patients may not feel normal for several hours or days after a seizure. The patient has no memory of the seizure. Reference: 1493

Along with dementia, what additional symptom is characteristic of Lewy body dementia? A. Myasthenia gravis B. Extrapyramidal signs C. Ataxia D. Multiple sclerosis

B Lewy body dementia has Lewy bodies in the brainstem and cortex. In addition to dementia, it is characterized by at least two of the following: extrapyramidal signs, fluctuating cognitive ability, and hallucinations.

Which statement is true regarding the prevalence of restless legs syndrome? A. Rates are great than 50%. B. Rates are lowest in the Asian population. C. Rates are higher in males than females. D. Rates are highest in the age group of 20 to 30 year olds.

B Restless legs syndrome (RLS) is a relatively common condition characterized by unpleasant sensory (paresthesias) and motor abnormalities of one or both legs. Prevalence rates vary from 5% to 15%. However, the prevalence may be higher because the condition is underdiagnosed. The prevalence of RLS is lower in Asian populations, and RLS is more common in older adults. It is more common in women than men, and women may have an earlier age of onset. Reference: 1514

The patient is seen in the clinic due to an increase in the frequency of seizure activity. In addition to a thorough health history you should draw blood for A. anemia. B. serum drug levels. C. arterial blood gases. D. electrolytes.

B Serum levels of the drug should be monitored if seizures continue to occur, if seizure frequency increases, or if drug compliance is questioned. The therapeutic range for each drug indicates the serum level above which most patients experience toxic side effects and below which most continue to have seizures. Reference: 1495

How should you most accurately assess the position sense of a patient with a recent traumatic brain injury? A. Ask the patient to close his or her eyes and slowly bring the tips of the index fingers together. B. Ask the patient to maintain balance while standing with his or her feet together and eyes closed. C. Ask the patient to close his or her eyes and identify the presence of a common object on the forearm. D. Place the two points of a calibrated compass on the tips of the fingers and toes, and ask the patient to discriminate the points.

B The Romberg test is an assessment of position sense in which the patient stands with the feet together and then closes his or her eyes while attempting to maintain balance. The other tests of neurologic function do not directly assess position sense.

After a major head trauma, the patient's respiratory and cardiac functions are affected. Which area of the brain is damaged? A. Temporal lobe of the cerebrum B. Brainstem C. Cerebellum D. Spinal nerves

B The brainstem includes the midbrain, pons, and medulla. The vital centers concerned with respiratory, vasomotor, and cardiac function are located in the medulla. Integration of somatic, visual, and auditory data occurs in the temporal lobe. The cerebellum coordinates voluntary movement, trunk stability, and equilibrium. Motor and spinal nerves serve particular areas of the body.

When working with the family of a patient with amyotrophic lateral sclerosis (ALS) in the later stages of the disease, what teaching should you reinforce? A. Perform frequent passive range of motion to maintain joint function. B. Communicate with patient normally because cognition remains intact. C. Provide foods high in fiber to prevent constipation. D. Speak in a loud clear voice to facilitate the patient's hearing.

B The illness trajectory for ALS is devastating because the patient remains cognitively intact while wasting away. Active range of motion is needed to maintain muscle function. Foods high in fiber may be difficult to chew. Patients with ALS usually do not have a hearing impairment. Reference: 1514

. Which characteristic of a patient's recent seizure indicates a partial seizure? A. The patient lost consciousness during the seizure. B. The seizure involved lipsmacking and repetitive movements. C. The patient fell to the ground and became stiff for 20 seconds. D. The etiology of the seizure involved both sides of the patient's brain.

B The most common complex partial seizure involves lip smacking and automatisms (repetitive movements that may not be appropriate). Loss of consciousness, bilateral brain involvement, and a tonic phase are associated with generalized seizure activity. Reference: 1494

he nursing student would be correct in identifying that the pain of a cluster headache is characterized as A. dull and heavy. B. sharp and stabbing. C. preceded by an aura. D. worsened by light.

B The pain of cluster headache is sharp and stabbing, which is in contrast to the pulsing pain of the migraine headache. Reference: 1487

Preventing which problem is a priority nursing goal for a patient who had cranial surgery today? A. Pain B. Increased ICP C. Infection D. Malnutrition

B The primary goal of care after cranial surgery is prevention of increased ICP. Other priorities are monitoring neurologic function, fluid and electrolyte levels, and serum osmolality. The brain does not have pain receptors, although the patient can have a headache. However, increased ICP remains a priority. Infection is not a priority the day of surgery, and nutrition is important, but increased ICP is the priority.

How do you assess the accessory nerve? A. Assess the gag reflex by stroking the posterior pharynx. B. Ask the patient to shrug the shoulders against resistance. C. Ask the patient to push the tongue to either side against resistance. D. Have the patient say "ah" while visualizing elevation of the soft palate

B The spinal accessory nerve is tested by asking the patient to shrug the shoulders against resistance and to turn the head to either side against resistance. The other options are used to test the glossopharyngeal and vagus nerves.

What are the two most common causes of dementia? A. Diabetes mellitus and hypercholesterolemia B. Neurodegenerative conditions and vascular disorders C. Effects of smoking and coronary artery disease D. Metabolic syndrome and systemic diseases

B The two most common causes of dementia are neurodegenerative conditions (e.g., Alzheimer's disease) and vascular disorders. Risk factors include advanced age, family history, history of smoking, cardiac dysrhythmias, hypertension, hypercholesterolemia, diabetes mellitus, and coronary artery disease, and metabolic syndrome.

The patient asks you, "How can I be certain I have Parkinson's disease?" Your response is based on the knowledge that the absolute confirmation of the diagnosis is A. decreased serum dopamine level. B. positive response to medication administration. C. nerve biopsy. D. electromyography (EMG).

B The ultimate confirmation of PD is a positive response to antiparkinsonian drugs. Reference: 1508

You plan care for the patient with increased ICP with the knowledge that the best way to position the patient is to A. keep the head of the bed flat. B. elevate the head of the bed to 30 degrees. C. maintain patient on the left side with the head supported on a pillow. D. use a continuous-rotation bed to continuously change patient position.

B You should maintain the patient with increased ICP in the head-up position. Elevation of the head of the bed to 30 degrees enhances respiratory exchange and aids in decreasing cerebral edema. You should position the patient to prevent extreme neck flexion, which can cause venous obstruction and contribute to elevated ICP. Elevation of the head of the bed reduces sagittal sinus pressure, promotes drainage from the head through the valveless venous system in the jugular veins, and decreases the vascular congestion that can produce cerebral edema. However, raising the head of the bed above 30 degrees may decrease the cerebral perfusion pressure (CPP) by lowering systemic blood pressure. Careful evaluation of the effects of elevation of the head of the bed on the ICP and the CPP is required.

A patient who has undergone a lung transplant has contracted cytomegalovirus (CMV) retinitis. The nurse expects which drug to be ordered for this patient? a. Acyclovir (Zovirax) b. Ganciclovir (Cytovene) c. Ribavirin (Virazole) d. Amantadine (Symmetrel)

B Ganciclovir is indicated for the treatment of cytomegalovirus retinitis. Acyclovir is used for herpes simplex types 1 and 2, herpes zoster, and chickenpox; amantadine is used for influenza type A; and zanamivir is used for influenza types A and B.

The nurse is providing counseling to a woman who is HIV positive and has just discovered that she is pregnant. Which anti-HIV drug is given to HIV-infected pregnant women to prevent transmission of the virus to the infant? a. Acyclovir (Zovirax) b. Zidovudine (Retrovir) c. Ribavirin (Virazole) d. Foscarnet (Foscavir)

B Zidovudine, along with various other antiretroviral drugs, is given to HIV-infected pregnant women and even to newborn babies to prevent maternal transmission of the virus to the infant. The other drugs are non-HIV antiviral drugs.

20. A patient is admitted to the emergency department (ED) with fever, swollen lymph glands, sore throat, headache, malaise, joint pain, and diarrhea. What nursing actions will help identify the need for further assessment of the cause of this patient's manifestations (select all that apply.)? A. Assessment of lung sounds B. Assessment of sexual behavior C. Assessment of living conditions D. Assessment of drug and syringe use E. Assessment of exposure to an ill person

B, D With these symptoms, assessing this patient's sexual behavior and possible exposure to shared drug equipment will identify if further assessment for the HIV virus should be made or the manifestations are from some other illness (e.g., lung sounds and living conditions may indicate further testing for TB).

The nurse provided teaching to an older adult client about fevers. Which client statement indicates that the teaching was​ effective? A. "The rectal route is the best way to have my temperature​ taken." B. "I may not have a fever when I get sick or have an​ infection." C. "I am less sensitive to environmental temperatures than when I was​ younger." D. "Cancer is the top source of fever in older​ adults."

B. "I may not have a fever when I get sick or have an infection." Rationale: Older adults do not exhibit the​ sign/symptom of fever with​ infection, as do younger persons.​ However, the top source of fever is still infection or an inflammatory​ process, not cancer. Rectal route for taking a temperature is not the best route due to discomfort and increased prevalence of hemorrhoids. Older adults are more sensitive to extreme environmental temperature changes due to decreased thermoregulatory controls.

Which patient statement to the nurse indicates a need for additional instruction in administering insulin? a. "I can buy the 0.5-mL syringes because the line markings are easier to see." b. "I need to rotate injection sites among my arms, legs, and abdomen each day." c. "I do not need to aspirate the plunger to check for blood before injecting insulin." d. "I should draw up the regular insulin first, after injecting air into the NPH bottle."

B. "I need to rotate injection sites among my arms, legs, and abdomen each day."

A patient with an intracranial problem does not open his eyes to any stimulus, has no verbal response except moaning and muttering when stimulated, and flexes his arm in response to painful stimuli. The nurse records the patients GCS score as a. 6 b. 7 c. 9 d. 11

B. 7- no opening of eyes = 1; incomprehensible words= 2, flexion withdrawal = 4 Total = 7

After change-of-shift report, which patient should the nurse assess first? a. A 19-yr-old patient with type 1 diabetes who has a hemoglobin A1C of 12% b. A 23-yr-old patient with type 1 diabetes who has a blood glucose of 40 mg/dL c. A 50-yr-old patient who uses exenatide (Byetta) and is reporting acute abdominal pain d. A 40-yr-old patient who is pregnant and whose oral glucose tolerance test is 202 mg/dL

B. A 23-yr-old patient with type 1 diabetes who has a blood glucose of 40 mg/dL

A pathology report notes that there is an abcess in your patient's lung. The patient asks for an explanation of this finding. How would you describe this? A. A small area of scar tissue containing new blood vessels and fibroblasts B. A small area containing pus made up of apoptotic neutrophils walled off by fibroblasts C. An accumulation of macrophages and fibroblasts D. An accumulation of macrophages and connective tissue encapsulating an persistent infection

B. A small area containing pus made up of apoptotic neutrophils walled off by fibroblasts

The nurse is obtaining a medication history from an 18-year-old female patient who has been diagnosed with genital herpes. Which drug would the nurse expect this patient to be prescribed? A) amantadine (Symmetrel) B) acyclovir (Zovirax) C) zidovudine (Retrovir) D) ribavirin (Virazole)

B. Acyclovir is the drug of choice to treat herpes simplex infections. Ribavirin is effective against respiratory syncytial virus (RSV), zidovudine against HIV, and amantadine against H. influenzae type A.

When checking a patient's fingerstick blood glucose level, the nurse obtains a reading of 42 mg/dL. The patient is awake but states he feels a bit "cloudy-headed." After double-checking the patient's glucose level and getting the same reading, which action by the nurse is most appropriate? A. Administer two packets of table sugar. B. Administer oral glucose in the form of a semisolid gel. C. Administer 50% dextrose IV push. D. Administer the morning dose of lispro insulin.

B. Administer oral glucose in the form of a semisolid gel.

A patient who has diabetes and reports burning foot pain at night receives a new prescription. Which information should the nurse teach the patient about amitriptyline? a. Amitriptyline decreases the depression caused by your foot pain. b. Amitriptyline helps prevent transmission of pain impulses to the brain. c. Amitriptyline corrects some of the blood vessel changes that cause pain. d. Amitriptyline improves sleep and makes you less aware of nighttime pain.

B. Amitriptyline helps prevent transmission of pain impulses to the brain.

When a nurse is teaching about the transverse fiber tract that connects the two cerebral hemispheres, what term should the nurse use? a. Peduncle b. Corpus callosum c. Basal ganglia d. Pons

B. Corpus Callosum

The nurse observes a mother stroking her​ child's arms and legs with a​ cool, damp washcloth. Which method of heat transfer is the mother using to reduce the​ fever? A. Radiation B. Evaporation C. Conduction D. Metabolism

B. Evaporation Rationale: Heat can be transferred between places or objects. Evaporation is the conversion of water to​ vapor, which is what occurs when the mother applies cool water to the​ child's limbs. Radiation is the release of heat through no physical contact. Conduction is the release of heat through physical contact. Metabolism is not a method of heat transfer

The nurse is caring for a client with an altered level of consciousness. To assess the level of​ consciousness, the nurse administers the Glasgow Coma Scale. Which response does this scale​ measure? (Select all that​ apply.) A. Cerebellar function B. Eye opening C. Verbal response D. Corneal reflex E. Motor response

B. Eye opening C. Verbal response E. Motor response

A few weeks after an 82-yr-old patient with a new diagnosis of type 2 diabetes has been placed on metformin (Glucophage) therapy, the home health nurse makes a visit. Which finding should the nurse promptly discuss with the health care provider? a. Hemoglobin A1C level is 7.9%. b. Glomerular filtration rate is decreased. c. Last eye examination was 18 months ago. d. Patient has questions about the prescribed diet.

B. Glomerular filtration rate is decreased.

Which information will the nurse include when teaching a patient who has type 2 diabetes about glyburide? a. Glyburide decreases glucagon secretion from the pancreas. b. Glyburide stimulates insulin production and release from the pancreas. c. Glyburide should be taken even if the morning blood glucose level is low. d. Glyburide should not be used for 48 hours after receiving IV contrast media.

B. Glyburide stimulates insulin production and release from the pancreas.

A new vaccination, Zostavax, has been approved to prevent the development of what condition in adults over the age of 50? A. Herpes simplex B. Herpes zoster C. Influenza A D. Avian influenza

B. Herpes zoster Zostavax is a new vaccination developed and approved for use in adults over the age of 50 to prevent the onset or outbreak of herpes zoster (shingles)

An unresponsive patient who has type 2 diabetes is brought to the emergency department and diagnosed with hyperosmolar hyperglycemia syndrome (HHS). What should the nurse anticipate doing? a. Giving 50% dextrose b. Inserting an IV catheter c. Initiating O2 by nasal cannula d. Administering glargine (Lantus) insulin

B. Inserting an IV catheter

Which is a rapid-acting insulin with an onset of action of less than 15 minutes? A. Insulin glargine (Lantus) B. Insulin aspart (NovoLog) C. Insulin detemir (Levemir) D. Regular insulin (Humulin R)

B. Insulin aspart (NovoLog) Correct Insulin aspart is a rapid-acting insulin. Insulin glargine and insulin detemir are long-acting insulins. Regular insulin is short acting.

Which information should the nurse include in a teaching plan for patients taking oral hypoglycemic drugs? (Select all that apply.) A. Take your medication only as needed. B. Report symptoms of anorexia and fatigue. C. Explain dietary changes are not necessary. D. Advise to avoid smoking and alcohol consumption. E. Instruct that it is okay to skip breakfast 1 to 2 times per week.

B. Report symptoms of anorexia and fatigue. Correct D. Advise to avoid smoking and alcohol consumption. Corre long-term complications of diabetes. Skipping meals can cause low blood glucose levels and should be avoided. Patients with type 2 diabetes mellitus are managed with lifestyle changes. All other options are correct.

A 19-year-old male college student reports to his primary care provider that he cannon stay awake in class regardless of how much sleep he gets. A drug that stimulates which of the following areas would best treat his problem? A. Copora quadrigemina B. Reticular activating system C. Cerebellum D. Hypothalamus

B. Reticular Activating system

The Arthus reaction is an example of which type of sensitivity reaction? a. I b. II c. III d. IV

C The Arthus reaction is a type III hypersensitivity reaction.

A patient's sudden onset of hemiplegia has necessitated a computed tomography (CT) of her head. Which assessment should you complete before this diagnostic study? A. Assess the patient's immunization history. B. Screen the patient for any metal parts or a pacemaker. C. Assess the patient for allergies to shellfish, iodine, or dyes. D. Assess the patient's need for tranquilizers or antiseizure medications.

C Allergies to shellfish, iodine, or dyes contraindicate the use of contrast media for CT. The patient's immunization history is not a central consideration, and the presence of metal in the body does not preclude the use of CT as a diagnostic tool. The need to assess for allergies supersedes the need for tranquilizers or antiseizure medications in most patients.

Which statement is true regarding the prevalence of amyotrophic lateral sclerosis (ALS)? A. Death occurs in less than one year B. Survival rate is less than two years C. Rates are higher in males than females D. Rates are highest in the age group of 20 to 30 year olds

C Amyotrophic lateral sclerosis (ALS) is a rare progressive neurologic disorder characterized by loss of motor neurons. ALS usually leads to death within 2 to 6 years after diagnosis, but many patients may survive for more than 10 years. The onset is usually between 40 and 70 years of age. ALS is more common in men than women by a ratio of 2:1. Reference: 1514-1515

Which option is most indicative of a skull fracture after blunt head trauma? A. Facial edema B. Epitasis C. Otorrhea positive for glucose D. Laceration oozing blood

C An indication of a basal fracture is cerebrospinal fluid (CSF) leakage from the ear, which confirms that the fracture has traversed the dura. Periorbital ecchymosis can indicate a skull fracture, but generalized facial edema does not. The head is vascular, and it is not unusual to have a nosebleed; a positive ring sign (halo sign) indicates a skull fracture. A superficial laceration does not indicate a skull fracture.

Why does the health care provider prescribe a cholinesterase inhibitor (e.g., donepezil [Aricept]) for a patient with AD? A. Cure the disease B. Stop any further decline C. Slow the rate of decline D. Treat co-morbidity of depression

C Cholinesterase inhibitors are used to slow the rate of decline of the disease and the worsening of symptoms. There is no cure for AD, which is a chronic, progressive disease. Treating depression that is often associated with AD may improve cognitive ability, and antidepressants are prescribed.

What distinguishes Creutzfeldt-Jakob disease from Alzheimer disease (AD)? A. A curative drug exists. B. Rheumatoid arthritis develops. C. Involuntary muscle jerks are present. D. A fine, papular, pink rash develops.

C Creutzfeldt-Jakob disease is a rare and fatal brain disorder. The earliest symptom may be memory impairment and behavioral changes. The disease progresses rapidly, with mental deterioration, involuntary movement (muscle jerks), weakness in the limbs, blindness, and eventually coma. There is no treatment. Reference: 1533

What sign would make you suspect the cause of increased ICP involves the hypothalamus? A. Contralateral hemiparesis B. Ipsilateral pupil dilation C. Rise in temperature D. Decreased urine output

C If the ICP affects the hypothalamus, there can be a change in the body temperature. Increasing ICP can cause changes in motor ability, with contralateral hemiparesis. Compression of the cranial nerve III causes dilation of the pupil on the side of the mass (ipsilateral). Decreased urine output is not specific for hypothalamic function.

A patient with increased ICP has mannitol (Osmitrol) prescribed. Which option is the best indication that the drug is achieving the desired therapeutic effects? A. Urine output increases from 30 mL to 50 mL/hour. B. Blood pressure remains less than 150/90 mm Hg. C. The LOC improves. D. No crackles are auscultated in the lung fields.

C LOC is the most sensitive indicator of ICP. Mannitol is an osmotic diuretic that works to decrease the ICP by plasma expansion and an osmotic effect. Although the other options may indicate a therapeutic effect of a diuretic, they are not the main reason this drug is given.

You are answering questions at a class on brain tumors for nursing students. Which information related to brain tumors should you include in this class? A. Brain tumors are not removed unless than they cause headaches or seizures. B. Seizures are an uncommon symptom unless there is metastasis. C. The most common type of brain tumor is from metastasis of cancer outside of the brain. D. Brain tumors commonly metastasize to the lungs because of high vascularity.

C Metastatic brain tumors from a malignant neoplasm elsewhere in the body are the most common type of brain tumors. All brain tumors eventually cause death from increasing tumor volume that leads to increased ICP, and all tumors should be removed. Seizures are common in patients with gliomas and brain metastases. Brain tumors rarely metastasize outside the CNS because they are contained by structural (meninges) and physiologic (blood-brain) barriers.

What is the purpose of the blood-brain barrier? A. To protect the brain by cushioning B. To inhibit damage from external trauma C. To keep harmful agents away from brain tissue D. To provide the blood supply to brain tissue

C The blood-brain barrier is a physiologic barrier between capillaries and brain tissue. The structure of the brain's capillaries is different from others, and substances that are harmful are not allowed to enter brain tissue. Lipid-soluble compounds enter the brain easily, but water-soluble and ionized drugs enter slowly. The spinal fluid and meninges help cushion the brain. The skull protects from external trauma. Blood is supplied to the brain from the internal carotid arteries and the vertebral arteries.

You would expect the patient with restless legs syndrome to identify the discomfort or pain to present A. within the first 30 minutes after getting out of bed in the morning. B. after vigorous exercise. C. when sedentary. D. during a shower or bath.

C The discomfort occurs when the patient is sedentary and is most common in the evening or at night. Reference: 1514

What is the most likely cause of a brain abscess? A. Secondary metastasis from another site B. Infection from a tick or mosquito bite C. Direct extension from a sinus infection D. Spread of Neisseria meningitidis infection

C The primary cause of a brain abscess is direct extension from ear, tooth, mastoid, or sinus infection. Secondary metastasis is the source of brain tumors. Tick or mosquito bites are associated with viral meningitis. N. meningitidis is a cause of bacterial meningitis.

Which nursing diagnosis is likely to be a priority in the care of a patient with myasthenia gravis? A. Acute confusion B. Bowel incontinence C. Activity intolerance D. Disturbed sleep pattern

C The primary feature of MG is fluctuating weakness of skeletal muscle. Bowel incontinence and confusion are unlikely signs of MG, and while sleep disturbance is likely, activity intolerance is usually of primary concern.

Treatment of status epilepticus requires initiation of a rapid-acting antiseizure drug that can be given intravenously. You would anticipate which drugs to be administered (select all that apply)? A. phenytoin (Dilantin) B. phenobarbital C. lorazepam (Ativan) D. diazepam (Valium) E. carbemazepine (Tegretol)

C,D Treatment of status epilepticus requires initiation of a rapid-acting antiseizure drug that can be given intravenously. The drugs most commonly used are lorazepam (Ativan) and diazepam (Valium).

The post-head injury client opens eyes to sound, has no verbal response and localizes to painful stimuli when applied to each extremity. How should the nurse document the Glasgow Coma Scale (GCS) score? A. GCS= 3 B. GCS= 6 C. GCS= 9 D. GCS= 11

C. Score of 3 is given when the client opens the eyes to sound. Localization to pain is scored as 5. When there is no verbal response the score is a 1. The total score is then equal to 9.

A client is being prepared for lumbar puncture (LP). The nurse assists the client into which of the following positions for the procedure? A. Prone, in slight Trendelenburg's position B. Prone, with a pillow under the abdomen C. Side-lying, with the legs pulled up and the head bent down onto the chest. D. Side-lying, with a pillow under the hip

C. The client undergoing LP is positioned lying on the side, with the legs pulled up against the abdomen and with the head bent down toward the chest. This position helps widen the spaces between the vertebrae.

The nurse is assessing a 22-yr-old patient experiencing the onset of symptoms of type 1 diabetes. To which question would the nurse anticipate a positive response? a. "Are you anorexic?" b. "Is your urine dark colored?" c. "Have you lost weight lately?" d. "Do you crave sugary drinks?"

C. "Have you lost weight lately?"

A patient with diabetes rides a bicycle to and from work every day. Which site should the nurse teach the patient to use to administer the morning insulin? a. Thigh b. Buttock c. Abdomen d. Upper arm

C. Abdomen

Which neurotransmitter is released when a patients parasympathetic motor neurons are stimulated? a. Epinephrine b. Serotonin c. Acetylcholine d. Substances P

C. Acetylcholine

Eosinophils are predominant in ____ and ____. A. Allergic reactions and autoimmune response B. Chronic inflammation and cell mediated hypersensitivity C. Allergic reactions and parasitic infections D. Chronic inflammation and antibody response

C. Allergic reactions and parasitic infections

A patient with type 2 diabetes is scheduled for magnetic resonance imaging (MRI) with contrast dye. The nurse reviews the orders and notices that the patient is receiving metformin (Glucophage). Which action by the nurse is appropriate? A. Proceed with the MRI as scheduled. B. Notify the radiology department that the patient is receiving metformin. C. Expect to hold the metformin the day of the test and for 48 hours after the test is performed. D. Call the prescriber regarding holding the metformin for 2 days before the MRI is performed.

C. Expect to hold the metformin the day of the test and for 48 hours after the test is performed.

The nurse will instruct the patient to treat hypoglycemia with which drug? A. Acarbose (Precose) B. Propranolol (Inderal) C. Glucagon (GlucaGen) D. Bumetanide (Bumex)

C. Glucagon (GlucaGen) Correct Glucagon stimulates glycogenolysis, raising serum glucose levels.

Medications used to treat human immunodeficiency virus (HIV) infections are most specifically classified as A) antifungal drugs. B) antiviral drugs. C) antiretroviral drugs. D) antiparasitic drugs.

C. HIV is a member of the retrovirus family; therefore drugs used to treat this virus are classified as antiretroviral drugs. Although antiretroviral drugs also fall under the broader category of antiviral drugs in general, their mechanisms of action are unique to the AIDS virus. So, they are more commonly referred to by their subclassification as antiretroviral drugs.

A patient screened for diabetes at a clinic has a fasting plasma glucose level of 120 mg/dL (6.7 mmol/L). What should the nurse plan to teach the patient? a. Self-monitoring of blood glucose b. Using low doses of regular insulin c. Lifestyle changes to lower blood glucose d. Effects of oral hypoglycemic medications

C. Lifestyle changes to lower blood glucose

The nurse is evaluating teaching provided to the parents of​ 2-year-old twins regarding temperature measurement devices in the home. Which observation indicates that teaching has been​ effective? (Select all that​ apply.) A. Separate mercury glass thermometers are available for each child. B. There is no evidence of a​ temperature-measuring device in the home. C. Mercury glass thermometers are removed from the home. D. An electronic thermometer with disposable covers is available for use. E. Pad and pencil are placed next to the​ temperature-measuring device.

C. Mercury glass thermometers are removed from the home D. an electronic thermometer with disposable covers is available for use E. Pad and pencil are placed next to the thermometer measuring device

A 26-yr-old female who has type 1 diabetes develops a sore throat and runny nose after caring for her sick toddler. The patient calls the clinic for advice about her symptoms and reports a blood glucose level of 210 mg/dL despite taking her usual glargine (Lantus) and lispro (Humalog) insulin. What should the nurse advise the patient to do? a. Use only the lispro insulin until the symptoms are resolved. b. Limit intake of calories until the glucose is less than 120 mg/dL. c. Monitor blood glucose every 4 hours and contact the clinic if it rises. d. Decrease carbohydrates until glycosylated hemoglobin is less than 7%.

C. Monitor blood glucose every 4 hours and contact the clinic if it rises.

Which answer indicates a nurse understands regeneration of neurons? Neurons that have the capacity for regeneration include: A. Unmyelinated neurons in the brain B. Myelinated neurons in the spinal cord C. Myelinated peripheral neurons D. Postganglionic motor neurons

C. Myelinated peripheral neurons

A patient who was admitted with diabetic ketoacidosis secondary to a urinary tract infection has been weaned off an insulin drip 30 minutes ago. The patient reports feeling lightheaded and sweaty. Which action should the nurse take first? a. Infuse dextrose 50% by slow IV push. b. Administer 1 mg glucagon subcutaneously. c. Obtain a glucose reading using a finger stick. d. Have the patient drink 4 ounces of orange juice.

C. Obtain a glucose reading using a finger stick.

Which type of body temperature changes in response to the environment? A. core B. metabolic C. surface D. physiologic

C. Surface

Which action by a patient indicates that the home health nurse's teaching about glargine and regular insulin has been successful? a. The patient administers the glargine 30 minutes before each meal. b. The patient's family prefills the syringes with the mix of insulins weekly. c. The patient discards the open vials of glargine and regular insulin after 4 weeks. d. The patient draws up the regular insulin and then the glargine in the same syringe.

C. The patient discards the open vials of glargine and regular insulin after 4 weeks.

When a patient who takes metformin (Glucophage) to manage type 2 diabetes develops an allergic rash from an unknown cause, the health care provider prescribes prednisone. What should the nurse anticipate? a. The patient may need a diet higher in calories while receiving prednisone. b. The patient may develop acute hypoglycemia while taking the prednisone. c. The patient may require administration of insulin while taking prednisone. d. The patient may have rashes caused by metformin-prednisone interactions.

C. The patient may require administration of insulin while taking prednisone.

A patient who has type 2 diabetes is being prepared for an elective coronary angiogram. Which information would the nurse anticipate might lead to rescheduling the test? a. The patient's most recent A1C was 7.5%. b. The patient's blood glucose is 128 mg/dL. c. The patient took the prescribed metformin today. d. The patient took the prescribed enalapril 4 hours ago.

C. The patient took the prescribed metformin today.

One of the self-regulatory actions of the immune system is to identify self-antigens. What is this ability called? A. Antigen specificity B. Autoantibody formation C. Tolerance D. Antigen diversity

C. Tolerance

A patient admitted with a head injury has admission vital signs of temperature 98.6° F (37° C), blood pressure 128/68, pulse 110, and respirations 26. Which of these vital signs, if taken 1 hour after admission, will be of most concern to the nurse? a. Blood pressure 130/72, pulse 90, respirations 32 b. Blood pressure 148/78, pulse 112, respirations 28 c. Blood pressure 156/60, pulse 60, respirations 14 d. Blood pressure 110/70, pulse 120, respirations 30

Correct Answer: C Rationale: Systolic hypertension with widening pulse pressure, bradycardia, and respiratory changes represent Cushing's triad and indicate that the ICP has increased and brain herniation may be imminent unless immediate action is taken to reduce ICP. The other vital signs may indicate the need for changes in treatment, but they are not indicative of an immediately life-threatening process. Cognitive Level: Application Text Reference: p. 1469 Nursing Process: Assessment NCLEX: Physiological Integrity

A patient has a systemic blood pressure (BP) of 120/60 mm Hg and an intracranial pressure of 24 mm Hg. The nurse determines that the cerebral perfusion pressure (CPP) of this patient indicates a. high blood flow to the brain. b. normal intracranial pressure (ICP). c. impaired brain blood flow. d. adequate cerebral perfusion.

Correct Answer: C Rationale: The patient's CPP is 56, below the normal of 70 to 100 mm Hg and approaching the level of ischemia and neuronal death. The patient has low cerebral blood flow/perfusion. Normal ICP is 0 to 15 mm Hg. Cognitive Level: Application Text Reference: p. 1468 Nursing Process: Assessment NCLEX: Physiological Integrity

What is the classic source for transmission of the small infectious pathogen (prion protein) in Creutzfeldt-Jakob disease? A. Dogs B. Ticks C. Seafood D. Cows

D The source is beef obtained from animals contaminated with bovine spongiform encephalopathy. The disease is also known as mad cow disease. Reference: 1533

Effectiveness of antiviral drugs administered to treat HIV infection is evaluated by periodic assessment of A) megakaryocytes. B) red blood cell counts. C) lymphocyte counts. D) viral load.

D. All antiretroviral drugs work to reduce the viral load, which is the number of viral RNA copies per mL of blood.

A client with a traumatic brain injury is diagnosed as being brain dead. Which assessment finding supports this​ diagnosis? a. Complete unawareness of self b. Aware of environment but unable to communicate c. Absence of spontaneous respirations d. Neck extended and the jaw is clenched

c. Absence of spontaneous respirations Rationale Brain death is the cessation and irreversibility of all brain​ functions, including the brainstem. Since the brainstem controls​ respirations, absence of respirations would be a nursing assessment finding in brain death. Complete unawareness of self describes a persistent vegetative state. An extended neck with clenched jaw describes the decerebrate posturing. Aware of the environment but unable to communicate describes​ locked-in syndrome.

The nurse is caring for a patient diagnosed with syndrome of inappropriate antidiuretic hormone (SIADH). What is the nurse's best action? a. Encourage increased fluid and water intake b. Teach about risk for malignancies c. Monitor for changes in level of consciousness d. Assess labwork for potassium level changes

c. Monitor for changes in level of consciousness As the name suggests, SIADH is a condition in which antidiuretic hormone (ADH) is secreted despite normal or low plasma osmolarity, resulting in water retention and dilutional hyponatremia. In response to increased plasma volume, aldosterone secretion increases and further contributes to sodium loss. Hyponatremia frequently manifests with changes in level of consciousness from confusion to coma. A large number of clinical conditions can cause SIADH including malignancies, pulmonary disorders, injury to the brain, and certain pharmacologic agents. Malignancies often lead to SIADH versus SIADH causing malignant conditions. Water intoxication can lead to hyponatremia, therefore water intake is restricted. The most affected electrolyte from SIADH is sodium versus potassium.

Following a parathyroidectomy, which electrolyte should the nurse most closely monitor? a. Potassium b. Sodium c. Magnesium d. Calcium

d. Calcium Because the parathyroids are located on the thyroid gland, similar concerns for postoperative monitoring apply. Additionally, calcium levels are monitored to avoid hypocalcemic crisis.

When providing care to a client with increased intracranial pressure (IICP) requiring mechanical ventilation, the nurse is aware that intracranial pressure can increase due to: a. hypoxemia and hypercapnia. b. hyperventilation. c. oxygenation with a partial pressure of arterial oxygen at about 100 mmHg. d. partial pressure of arterial carbon dioxide of about 35 mmHg.

a. hypoxemia and hypercapnia. Rationale Hypoxemia and hypercapnia can increase intracranial pressure. It is important to maintain adequate oxygenation with a partial pressure of arterial oxygen at about 100 mmHg and a partial pressure of arterial carbon dioxide of about 35 mmHg. Hyperventilation decreases the PaCO2 and would cause the ICP to decrease.

The nurse is preparing to conduct a neurologic assessment interview with a client. Which general questions should the nurse use when conducting this​ assessment? ​(Select all that​ apply.) a. ​"Are you experiencing any​ pain?" b. ​"Are you having any problems with your​ memory?" c. ​"How many fingers am I holding up at this​ time?" d. ​"Do you have any problems with balance or​ coordination?" e. ​"Do you have a history of seizures or​ fainting?"

a. ​"Are you experiencing any​ pain?" b. ​"Are you having any problems with your​ memory?" d. ​"Do you have any problems with balance or​ coordination?" e. ​"Do you have a history of seizures or​ fainting?" Rationale General questions to include in a neurologic assessment interview include asking about​ pain, history of seizures or​ fainting, and asking about problems with memory and coordination or balance. Asking to identify the number of fingers would focus on the​ client's vision.

A 19-year-old female with type 1 DM was admitted to the hospital with altered consciousness and the following lab values: serum glucose 500 mg/dl (high) and serum K+ 2 (low). Her parents state that she has been sick with the "flu" for a week. The diagnosis is hyperosmolar hyperglycemia nonketotic syndrome (HHNKS). What relationship do these values have with her insulin deficiency? a. Increased glucose utilization causes the shift of fluid from the intravascular to the intracellular space. b. Decreased insulin causes hyperglycemia and osmotic diuresis. c. Increased glucose and fatty acid metabolism stimulates renal diuresis and electrolyte loss. d. Increased insulin use results in protein catabolism, tissue wasting, and electrolyte loss.

b

A patient presents to the clinic with observable edema and erythema of the left forearm. A brief history reveals no exposure to potential irritating agents. On palpation, the nurse finds the area very warm and tender. What is the most likely cause of the patient's symptoms? a. An allergic reaction b. A complement cascade c. IgE reactions d. Clonal diversity

b. A complement cascade A complement cascade is responsible for the dilation of blood vessels and leaking of fluid from the vascular system to the area of insult, resulting in the swelling and redness associated with an inflammatory response. An allergic reaction can cause edema and erythema, but the question does not provide enough information to determine the specific cause of the swelling and redness. IgE is a specific immunoglobulin associated with signs and symptoms of allergic rhinitis. Clonal diversity refers to the maturation process of cells.

A​ 78-year-old female client presents for an annual exam. The client admits to smoking a ​½ a pack of cigarettes per day for the last 50 years. The nurse would expect that the client would also complain of a deficit in what type of sensory​ stimuli? a. Visual b. Gustatory c. Tactile d. Auditory

b. Gustatory Rationale The two most common sensory deficits associated with tobacco use are the sense of taste​ (gustatory) and the sense of smell​ (olfactory). ​Visual, auditory, and tactile senses are not affected.

Which important teaching point should the nurse include in the plan of care for a patient diagnosed with Cushing's disease? a. Daily weight using same scale b. Wash hands frequently c. Use exfoliating soaps when bathing d. Avoid yearly influenza vaccine

b. Wash hands frequently Cushing's syndrome is characterized by chronic excess glucocorticoid (cortisol) secretion from the adrenal cortex. This is caused by the hypothalamus, or the anterior pituitary gland, or the adrenal cortex. Cushing's syndrome can also be caused by taking corticosteroids in the form of medication (such as prednisone) over time - referred to as exogenous Cushing syndrome. Regardless of the cause, excess secretion of cortisol has a systemic affect affecting immunity, metabolism, and fat distribution (truncal obesity), reduced muscle mass, loss of bone density, hypertension, fragility to microvasculature, as well as thinning of the skin. Washing hands is important because the patient's immune system is suppressed due to the excess glucocorticoid level. Daily weights are not indicated. Exfoliating soaps may damage thin skin. The patient should receive vaccinations due to being immunocompromised.

A 12-year-old female is newly diagnosed with type 1 DM. When the parents ask what causes this, what is the nurse's best response? a. A familial, autosomal dominant gene defect b. Obesity and lack of exercise c. Immune destruction of the pancreas d. Hyperglycemia from eating too many sweets

c

A 13-year-old male who uses insulin to control his type 1 diabetes experiences hunger, lightheadedness, tachycardia, pallor, headache, and confusion during gym class. The most probable cause of these symptoms is: a. Hyperglycemia resulting from incorrect insulin administration b. Dawn phenomenon caused by eating a snack before gym class c. Hypoglycemia caused by increased exercise d. Somogyi effect caused by insulin sensitivity

c

A nurse wants to determine if there is kidney dysfunction in a patient with diabetes. Which of the following is the earliest manifestation? a. Polyuria b. Glycosuria c. Microalbuminuria d. Decreased glomerular filtration

c

Which of the following diseases should the nurse teach the patient to prevent as it is the ultimate cause of death in the patient with diabetes? a. Renal disease b. Stroke c. Cardiovascular disease d. Cancer

c

Cardiovascular disease can be a consequence of which immune function disorder? a. Type IV hypersensitivity reaction b. Primary immune deficiency c. Autoimmune disorder d. Secondary immune suppression

c. Autoimmune disorder administration of corticosteroid medications. Cardiovascular disease can be a consequence of autoimmune disorders such as Grave's disease or systemic lupus erythematosus. Option A is incorrect because type IV hypersensitivity disorders cause conditions such as serum sickness. Option B is incorrect because primary immune deficiencies are disorders in which immune components are missing. They affect all body systems and are not organ specific. Option D is incorrect because secondary immune suppression refers to conditions caused by external immune suppression (e.g., administration of corticosteroid medications).

The nurse is preparing to administer medications to a patient with rheumatoid arthritis (RA). The nurse should explain which goal of treatment to the patient? a. Eradicate the disease b. Enhance immune response c. Control inflammation d. Manage pain

c. Control inflammation Rheumatoid arthritis is an autoimmune disorder that involves inflammation on the synovial joints & other tissues. While there is not a cure for RA, symptoms & pain are best relieved by treating the underlying inflammation.

A client recovering from a stroke is unable to swallow and has an absent gag reflex. Which cranial nerve should the nurse suspect is affected in this​ client? a. Hypoglossal b. Trigeminal c. Glossopharyngeal d. Spinal accessory

c. Glossopharyngeal Rationale The glossopharyngeal nerve produces the gag and swallowing reflexes. The trigeminal nerve has three branches. The mandibular branch receives sensory impulses from the​ tongue, lower​ teeth, skin of​ chin, and lower lip. The motor action of this branch includes teeth clenching and movement of the mandible. The hypoglossal nerve moves the tongue for​ swallowing, movement of food during​ chewing, and speech. The spinal accessory nerve moves the trapezius and sternocleidomastoid muscles and controls some movement of​ larynx, pharynx, and soft palate.

A client recovering from a stroke has an absent gag reflex and has lost 7 lb over the last 5 days. Which intervention does the nurse anticipate for this​ client? a. Administration of dextrose​ 50% and water b. Intramuscular injection of a corticosteroid c. Placement of a feeding tube d. Administration of hypotonic intravenous fluids

c. Placement of a feeding tube Rationale The client is losing fluid and is unable to safely eat or drink. The nurse should prepare for this client to have a feeding tube placed. Dextrose​ 50% and water is a treatment for status epilepticus. Corticosteroids are indicated to treat inflammation. Hypotonic fluids will increase intracranial pressure. Isotonic or hypertonic fluids would be indicated for this client.

When the immunologist says that pathogens possess virulence, what does virulence mean? a. Spreads from one individual to others and causes disease. b. Induces an immune response. c. Causes disease. d. Damages tissue.

ANS: C Virulence is the capacity of a pathogen to cause severe disease—for example, measles virus is of low virulence; rabies virus is highly virulent. Communication is the ability to spread from one individual to others and cause disease. Immunogenicity is the ability of pathogens to induce an immune response. Damaging tissues is the pathogen's mechanism of action.

12. When soluble antigens from infectious agents enter circulation, what is tissue damage a result of? a. Complement-mediated cell lysis b. Phagocytosis by macrophages c. Phagocytosis in the spleen d. Neutrophil granules and toxic oxygen products

ANS: D Of the options available, only the components of neutrophil granules as well as the several toxic oxygen products produced by these cells, damage the tissue.

13. Which nursing action will be most useful in assisting a college student to adhere to a newly prescribed antiretroviral therapy (ART) regimen? a. Give the patient detailed information about possible medication side effects. b. Remind the patient of the importance of taking the medications as scheduled. c. Encourage the patient to join a support group for students who are HIV positive. d. Check the patient's class schedule to help decide when the drugs should be taken.

ANS: D The best approach to improve adherence is to learn about important activities in the patient's life and adjust the ART around those activities. The other actions are also useful, but they will not improve adherence as much as individualizing the ART to the patient's schedule. DIF: Cognitive Level: Apply (application) REF: 228 TOP: Nursing Process: Planning MSC:

9. A student asks about the mechanism that results in type II hypersensitivity reactions. What description by the professor is best? a. Antibodies coat mast cells by binding to receptors that signal its degranulation, followed by a discharge of preformed mediators. b. Antibodies bind to soluble antigens that were released into body fluids, and the immune complexes are then deposited in the tissues. c. Cytotoxic T lymphocytes or lymphokine-producing helper T 1 cells directly attack and destroy cellular targets. d. Antibodies bind to the antigens on the cell surface.

ANS: D The mechanism that results in a type II hypersensitivity reaction begins with antibody binding to tissue-specific antigens or antigens that have attached to particular tissues. The cell can be destroyed by antibody IgG or IgM and activation of the complement cascade through the classical pathway.

13. How are target cells destroyed in a type II hypersensitivity reaction? a. Tissue damage from mast cell degranulation b. Antigen-antibody complexes deposited in vessel walls c. Cytotoxic T lymphocytes attack the cell directly. d. Natural killer cells

ANS: D The mechanism that results in a type II hypersensitivity reaction involves a subpopulation of cytotoxic cells that are not antigen specific (natural killer [NK] cells). Antibody on the target cell is recognized by Fc receptors on the NK cells, which releases toxic substances that destroy the target cell. Tissue damage from mast cell degranulation occurs in type I hypersensitivity reactions. Antigen-antibody complexes are active in type III hypersensitivity responses. Cytotoxic lymphocytes are involved in type IV hypersensitivity responses.

23. Which blood cell carries the carbohydrate antigens for blood type? a. Platelets b. Neutrophils c. Lymphocytes d. Erythrocytes

ANS: D The reaction that causes a blood transfusion recipient's red blood cells to clump together is related to the ABO antigens located on the surface of only erythrocytes.

30. Urticaria is a manifestation of a which type of hypersensitivity reaction? a. IV b. III c. II d. I

ANS: D Urticaria, or hives, is a dermal (skin) manifestation of only type I allergic reactions.

A 19-year-old female with type 1 DM was admitted to the hospital with the following lab values: serum glucose 500 mg/dl (high), urine glucose and ketones 4+ (high), and arterial pH 7.20 (low). Her parents state that she has been sick with the "flu" for a week. Which of the following statements best explains her acidotic state? a. Increased insulin levels promote protein breakdown and ketone formation. b. Her uncontrolled diabetes has led to renal failure. c. Low serum insulin promotes lipid storage and a corresponding release of ketones. d. Insulin deficiency promotes lipid metabolism and ketone formation.

d

The nurse is caring for a patient who was started on intravenous antibiotic therapy earlier in the shift. As the second dose is being infused, the patient reports feeling dizzy and having difficulty breathing and talking. The nurse notes that the patient's respirations are 26 breaths/min with a weak pulse of 112 beats/min. The nurse suspects that the patient is experiencing which condition? a. Suppressed immune response b. Hyperimmune response c. Allergic reaction d. Anaphylactic reaction

d. Anaphylactic reaction Signs & symptoms of a systemic anaphylactic (type I, IgE mediated) reaction include: headache, dizziness, angioedema, narrowed airway, wheezing, respiratory distress, dyspnea, hypotension, tachycardia, cardiac arrest, and GI distress.

A nurse at the public health center is discussing hearing screening guidelines with a​ 53-year-old client. How often would the nurse recommend the client getting his hearing​ checked? a. Every 10 years b. Prior to leaving the hospital c. Annually d. Every 3 years

d. Every 3 years Rationale The American​ Speech-Language-Hearing Association recommendations hearing screenings for individuals over the age of 50 every 3 years. Prior to the age of​ 50, it is recommended that hearing screens occur every 10 years or for a change in hearing or complaint of hearing loss. Infants are usually screened after their​ birth, and prior to leaving the hospital. Annual hearing screens for​ low-risk clients are not recommended.

The nurse is caring for a patient with a diagnosis of multiple sclerosis (MS). The nurse should be aware of which associated response? a. Primary immunodeficiency b. Secondary immunodeficiency c. Optimal immune response d. Exaggerated immune response

d. Exaggerated immune response MS is an autoimmune disease, which is a form of exaggerated immune response. MS is not a problem of immunodeficiency, nor is it an optimal immune response.

A client presumed to be brain dead is being assessed for a response to caloric stimulation, the process of irrigating the ear with ice cold water to test the oculovestibular reflex. What finding would be expected if, in fact, the client is brain dead? a. The eyes would move to the side opposite the irrigated side. b. Dilation of both pupils would occur with the irrigation. c. The eyes would not move toward the irrigated side. d. The eyes would move toward the irrigated side and then return to midline.

c. The eyes would not move toward the irrigated side. Rationale A normal response to the caloric stimulation test is for the client's eyes to move first toward the irrigated side, followed by a return to midline. Thus, in an individual with brain death on whom caloric stimulation is conducted, the eyes will not move first toward the irrigated side and then return to midline. Pupillary dilation is not what is examined with this particular procedure.

8. A patient has been diagnosed with human immunodeficiency virus (HIV) infection. What rationale for taking more than one antiretroviral medication should the nurse give to the patient to improve compliance? A. Viral replication will be inhibited. B. They will decrease CD4+ T cell counts. C. It will prevent interaction with other drugs. D. More than one drug has a better chance of curing HIV.

A The major advantage of using several classes of antiretroviral drugs is that viral replication can be inhibited in several ways, making it more difficult for the virus to recover and decreasing the likelihood of drug resistance that is a major problem with monotherapy. Combination therapy also delays disease progression and decreases HIV symptoms and opportunistic diseases. HIV cannot be cured. CD4+ T-cell counts increase with therapy. There are dangerous interactions with many antiretroviral drugs and other commonly used drugs.

12. A patient who is infected with human immunodeficiency virus (HIV) is being taught by the nurse about health promotion activities such as good nutrition; avoiding alcohol, tobacco, drug use, and exposure to infectious agents; keeping up to date with vaccines; getting adequate rest; and stress management. What is the rationale behind these interventions that the nurse knows? A. Delaying disease progression B. Preventing disease transmission C. Helping to cure the HIV infection D. Enabling an increase in self-care activities

A These health promotion activities along with mental health counseling, support groups, and a therapeutic relationship with health care providers will promote a healthy immune system, which may delay disease progression. These measures will not cure HIV infection, prevent disease transmission, or increase self-care activities.

A patient who has a neurologic disease that affects the pyramidal tract is likely to manifest which signs? A. Impaired muscle movement B. Decreased deep tendon reflexes C. Decreased level of consciousness D. Impaired sensation of touch, pain and temperature

A Among the most important descending tracts are the corticobulbar and corticospinal tracts, collectively called the pyramidal tract. These tracts carry volitional (voluntary) impulses from the cortex to the cranial and peripheral nerves. Dysfunction of the pyramidal tract is likely to manifest as impaired movement. Diseases affecting the pyramidal tract do not result in changes in the level of consciousness, impaired reflexes, or decreased sensation.

During admission of a patient with a severe head injury to the emergency department, you place the highest priority on assessment of A. patency of airway. B. presence of a neck injury. C. neurologic status with the Glasgow Coma Scale. D. cerebrospinal fluid leakage from the ears or nose.

A An initial priority in the emergency management of a patient with a severe head injury is for you to ensure that the patient has a patent airway.

The following patients are in the emergency department. Which one is a priority for treatment? A. A college student with suspected bacterial meningitis B. A teenager who had a partial seizure an hour earlier and is now awake C. Patient with a history of stroke last year with residual hemiparesis D. Patient with suspected trigeminal neuralgia (tic douloureux)

A Bacterial meningitis is considered a medical emergency. The other patients are currently stable.

The patient with AD is significantly more agitated and restless today. What action should you take first? A. Look at urine characteristics. B. Assess room temperature. C. Reassure the patient that she is safe. D. Allow the patient an area to pace in.

A Behavioral problems are often the patient's way of responding to a precipitating factor. Initially, the physical status should be evaluated. Urinary problems and pneumonia are the most common causes. Assessing the environment and reassuring the patient should then take place. The behavior should not be allowed to continue without trying to assess a cause for it. Reference: 1529

You approach the patient with AD to provide her bath. The patient states, "Go away! I'm not taking a bath." What is your initial response? A. Leave and reapproach in a few minutes. B. Ask the patient why she feels that way. C. Inform the patient that the physician will be notified. D. Obtain additional help and proceed with the bath.

A Behavioral problems occur in about 90% of patients with AD. They can respond to redirection, reapproach, distraction, and reassurance. Persons with AD have limited verbal skills and are not able to respond to "why" questions. You should not threaten with restraints or to call the physician. It is always preferable to try a nonthreatening approach first, especially for something that is not absolutely essential.

The aim of biofeedback in the patient who experiences headaches is to A. relax muscles and increase peripheral blood flow. B. raise the blood pressure. C. decrease the pulse rate. D. relax the diaphragm and increase the oxygen saturation.

A Biofeedback involves the use of physiologic monitoring equipment to give the patient information regarding muscle tension and peripheral blood flow (e.g., skin temperature of the fingers). The patient is trained to relax the muscles and raise the finger temperature, and is given reinforcement (operant conditioning) in accomplishing these changes. Reference: 1488

What is the appropriate task to delegate to the licensed practical nurse (LPN) who is taking care of a patient with Pick's disease? A. To administer sertraline (Zoloft) B. To administer the Mini-Mental State Examination (MMSE) C. To find out about the caregiver's stress level D. To make the initial referral to a day care center

A Pick's disease is frontotemporal dementia, in which the frontal and temporal anterior lobes of the brain shrink. It is characterized by disturbances in behavior, sleep, personality, and eventually memory with eventual dementia. LPNs can administer routine drugs. They should not perform assessments, make referrals, or generate nursing care plans. Reference: 1533

What is the standard to evaluate the degree of impaired consciousness for a patient with an acute head trauma? A. Best eye opening, verbal response, and motor response B. National Institutes of Health (NIH) Stroke Scale C. Romberg test D. Widening pulse pressure, bradycardia, and respirations

A The Glasgow Coma Scale (GCS) is a standardized tool used to assess the degree of impaired consciousness, and it consists of three components. The NIH stroke scale is used for a suspected stroke and includes other components of cranial nerve assessment, motor testing, and sensory testing. The Romberg test measures balance and is used for suspected cerebellar dysfunction. The components in the last option are Cushing's triad and an indication of increased ICP, not LOC.

Which statement is the proper nursing action when testing visual acuity? A. Record the lowest line the patient reads with 50% accuracy. B. Have the patient remove glasses before reading the Snellen test. C. Have the patient point to the readable line if the patient is aphasic. D. Have the patient tell the number of fingers held up if a Snellen test not available.

A The Snellen chart is read from 20 feet away, and the number on the lowest line that the patient can read with 50% accuracy is recorded. If the patient wears glasses normally, he or she should wear them during the test. Acuity may not be testable by these means if the patient does not read English or is aphasic. If a Snellen test is not available, the patient is asked to read newsprint for gross assessment of acuity, and the distance from the patient to the newsprint should be recorded. Using the number of fingers held up tests only gross visual ability.

You know the pathologic process of Huntington's disease (HD) involves a deficiency of A. acetylcholine. B. dopamine. C. serotonin. D. endorphins.

A The pathologic process of HD involves the basal ganglia and the extrapyramidal motor system. However, instead of a deficiency of dopamine, HD involves a deficiency of the neurotransmitters acetylcholine and γ-aminobutyric acid (GABA). Reference: 1515

What is the cause of the clinical manifestations of Parkinson's disease? A. Decreased levels of dopamine B. Decreased levels of acetylcholine C. Increased levels of angiotensinogen D. Increased levels of relaxin

A The pathologic process of PD involves degeneration of the dopamine-producing neurons in the substantia nigra of the midbrain. Reference: 1506

What is a key distinction of delirium compared with dementia? A. Sudden cognitive impairment B. Confusion related to time C. Distorted perceptions D. Worse in the morning

A The person with delirium is more likely to exhibit sudden cognitive impairment, disorientation, or clouded sensorium. Dementia is usually insidious. Delirium is rapid, often occurring at night. Confusion related to time and distorted perceptions are true for both. Depression is characterized by worse symptoms in the morning. Reference: 1533-1534

The patient relates to you that the health care provider suggested the patient may be diagnosed with restless legs syndrome (RLS). The patient asks you about the initial symptoms of RLS. You would identify which of the following as initial manifestation(s) of RLS? A. Numbness and tingling in the legs B. Periodic inability to control leg movements C. Cramp-like pain in the thighs D. Redness of the lower legs.

A The severity of RLS sensory symptoms ranges from infrequent minor discomfort (paresthesias, including numbness, tingling, and "pins and needles" sensation) to severe pain. Sensory symptoms often appear first and are manifested as an annoying and uncomfortable (but usually not painful) sensation in the legs. The sensation is often compared with the sensation of bugs creeping or crawling on the legs. Reference: 1514

Which statement is true for a patient who has pathology in Wernicke's area of the cerebrum? A. Receptive speech is affected. B. The parietal lobe is involved. C. Sight processing is abnormal. D. An abnormal Romberg test is present.

A The temporal, not parietal, lobe contains the Wernicke area, which is responsible for receptive speech and integration of somatic, visual, and auditory data. Sight processing occurs in the occipital lobe. The Romberg test is used to assess the position sense of the lower extremities.

A classic sign of an absence (petit mal) seizure is A. blank stare. B. frequent smacking of lips. C. urinary incontinence. D. asking the same question over and over.

A The typical clinical manifestation is a brief staring spell that lasts only a few seconds, so it often occurs unnoticed. There may be an extremely brief loss of consciousness. Reference: 1493

Which statement by the wife of a patient with AD demonstrates an accurate understanding of her husband's medication regimen? A. "I'm really hoping his medications will slow down his mental losses." B. "We're both holding out hope that this medication will cure his disease." C. "I know that this won't cure him, but we learned that it may prevent a bodily decline while he declines mentally." D. "I learned that if we are vigilant about his medication schedule, he may not experience the physical effects of his disease."

A There is no cure for AD, and drug therapy aims at improving or controlling the decline in cognition. Medications do not directly address the physical manifestations of AD. Reference: 1525

The patient says, "The smoodle pinkered, and I want him square, and I want to plunthery him." What impairment do you suspect? A. Wernicke's aphasia B. Broca's aphasia C. Dysarthria D. Aphasia

A Wernicke's aphasia is damage to the left temporal lobe, although it can result in damage to the right lobe. The patient may speak in long sentences that have no meaning, add unnecessary words, or even create words. In Broca's aphasia, the patient is able to understand but speaks only in short phrases that are difficult to produce. Dysarthria is a disturbance in the muscular control of speech. Impairments may involve pronunciation, articulation, and phonation. This condition does not affect meaning or comprehension of language but does affect the mechanics of speech. Aphasia is a total loss of comprehension and use of language or total inability to communicate.

You are called to the patient's room and find the patient in a clonic reaction. Your priority action is to A. record the time sequence of all of the patient's movements and responses as they occur. B. turn the patient to the side. C. call the health care provider. D. start oxygen by mask at 6 L/miN

A When a seizure occurs, you should carefully observe and record details of the event because the diagnosis and subsequent treatment often rest solely on the seizure description. All aspects of the seizure should be noted. What events preceded the seizure? When did the seizure occur? How long did each phase (aural [if any], ictal, postictal) last? Reference: 1499

A patient with a suspected closed head injury has bloody nasal drainage. You suspect that this patient has a cerebrospinal fluid (CSF) leak when observing which of the following? A. A halo sign on the nasal drip pad B. Decreased blood pressure and urinary output C. A positive reading for glucose on a Test-tape strip D. Clear nasal drainage along with the bloody discharge

A When drainage containing CSF and blood is allowed to drip onto a white pad, the blood coalesces into the center within a few minutes, and a yellowish ring of CSF encircles the blood, giving a halo effect. The presence of glucose is unreliable for determining the presence of CSF because blood also contains glucose.

You would correctly identify that the most common early symptom(s) of myasthenia gravis are A. weakness, fatigue, and ptosis. B. significant unilateral weakness. C. nausea, dizziness, and dysphagia. D. numbness and tingling of the extremities.

A he primary feature of MG is fluctuating weakness of skeletal muscle. Strength is usually restored after a period of rest. The muscles most often involved are those used for moving the eyes and eyelids, chewing, swallowing, speaking, and breathing. Reference: 1512

The nurse is administering intravenous acyclovir (Zovirax) to a patient with a viral infection. Which administration technique is correct? a. Infuse intravenous acyclovir slowly, over at least 1 hour. b. Infuse intravenous acyclovir by rapid bolus. c. Refrigerate intravenous acyclovir. d. Restrict oral fluids during intravenous acyclovir therapy.

A Intravenous acyclovir is stable for 12 hours at room temperature and often precipitates when refrigerated. Intravenous infusions must be diluted as recommended (e.g., with 5% dextrose in water or normal saline) and infused with caution. Infusion over longer than 1 hour is suggested to avoid the renal tubular damage seen with more rapid infusions. Adequate hydration should be encouraged (unless contraindicated) during the infusion and for several hours afterward to prevent drug-related crystalluria.

A client's baseline vital signs are as follows: temperature 98.8° F oral, pulse 74 beats/min, respirations 18 breaths/min, and blood pressure 124/76 mm Hg. The client's temperature suddenly spikes to 103° F. Which corresponding respiratory rate should the nurse anticipate in this client as part of the body's response to the change in status? 1. Respiratory rate of 12 breaths/min 2. Respiratory rate of 16 breaths/min 3. Respiratory rate of 18 breaths/min 4. Respiratory rate of 22 breaths/min

4

A health care provider has prescribed propylthiouracil (PTU) for a client with hyperthyroidism, and the nurse develops a plan of care for the client. The nurse should assess for which condition as a priority? 1. Relief of pain 2. Signs of renal toxicity 3. Signs of hyperglycemia 4. Signs of hypothyroidism

4

A nurse is assigned to the care of a client hospitalized with a diagnosis of hypothermia. The nurse anticipates that the client will exhibit which findings on assessment of vital signs? 1. Increased heart rate and increased blood pressure 2. Increased heart rate and decreased blood pressure 3. Decreased heart rate and increased blood pressure 4. Decreased heart rate and decreased blood pressure

4

The nurse has just administered ibuprofen (Motrin) to a child with a temperature of 38.8° C (102° F). The nurse should also take which action? 1. Withhold oral fluids for 8 hours. 2. Sponge the child with cold water. 3. Plan to administer salicylate (aspirin) in 4 hours. 4. Remove excess clothing and blankets from the child.

4

The nurse is caring for a client with acquired immunodeficiency syndrome (AIDS) who is experiencing night fever and night sweats. Which nursing intervention would be the least helpful in managing this symptom? 1. Keep liquids at the bedside. 2. Make sure the pillow has a plastic cover. 3. Keep a change of bed linens nearby in case they are needed. 4. Administer an antipyretic after the client has a spike in temperature.

4

The nurse is caring for an unconscious client who is experiencing persistent hyperthermia with no signs of infection. The nurse interprets that the hyperthermia may be related to damage to the client's thermoregulatory center in which structure? 1. Cerebrum 2. Cerebellum 3. Hippocampus 4. Hypothalamus

4

The postpartum nurse is taking the vital signs of a client who delivered a healthy newborn 4 hours ago. The nurse notes that the client's temperature is 100.2° F. What is the priority nursing action? 1. Document the findings. 2. Retake the temperature in 15 minutes. 3. Notify the health care provider (HCP). 4. Increase hydration by encouraging oral fluids.

4

A 16-year-old adolescent with recently diagnosed type 1 diabetes will receive NPH (Novolin N) insulin subcutaneously. The nurse teaches the adolescent about peak action of the drug and the risk for hypoglycemia. How many hours after the drug does the NPH peak? 1 1 to 2 hours 2 2 to 4 hours 3 5 to 10 hours 4 4 to 12 hours

4. NPH (Novolin N) insulin peaks in 4 to 12 hours; it has an onset time of 1½ to 4 hours and a duration of 18 to 24 hours. NPH insulin does not peak 1 to 2 hours, 2 to 4 hours, or 5 to 10 hours after administration.

What antibody binds to a mast cell? a. Cytotropic b. Allergen c. Antigen d. Fc

A A cytotropic antibody (also called a reagin) is the antibody that binds to the mast cell. An allergen is an environmental antigen that induces an allergic response. An antigen stimulates the immune response and antibody to bind to the mast cell. The Fc is the portion of the mast cell where IgE binds.

Which statement is true regarding hypersensitivity reactions? a. They require sensitization against a particular antigen. b. They occur after the primary immune response. c. Reactions are always delayed. d. The most delayed reaction is anaphylaxis.

A A hypersensitivity reaction requires sensitivity to an antigen and occurs after an adequate secondary immune response. These reactions may then be immediate or delayed. The most immediate reaction is anaphylaxis.

Which definition is true? a. Allergy means the deleterious effects of hypersensitivity. b. Immunity means an altered immunologic response. c. Hypersensitivity means the protective response to an antigen. d. Autoimmunity means the normal response to foreign antigens.

A Allergy means the deleterious effects of hypersensitivity to environmental antigens. Immunity is an organism's ability to resist disease. Hypersensitivity is an altered immunologic response to an antigen that results in disease or damage to the host. Autoimmunity is a disturbance in the immunologic tolerance of self-antigens.

A patient who is diagnosed with genital herpes is taking topical acyclovir. The nurse will provide which teaching for this patient? (Select all that apply.) a. "Be sure to wash your hands thoroughly before and after applying this medicine." b. "Apply this ointment until the lesion stops hurting." c. "Use a clean glove when applying this ointment." d. "If your partner develops these lesions, then he can also use the medication." e. "You will need to avoid touching the area around your eyes." f. "You will have to practice abstinence when these lesions are active."

A, C, E, F This medication needs to be applied as long as prescribed, and the medication needs to be applied with clean gloves. Prescriptions should not be shared; if the partner develops these lesions, the partner will have to be evaluated before medication is prescribed, if needed. Eye contact should be avoided. The presence of active genital herpes lesions requires sexual abstinence.

The patient taking topiramate (Topamax) for prophylactic treatment of migraine headache should be instructed to (select all that apply) A. not abruptly discontinue the drug. B. maintain adequate fluid intake. C. take the medication with food. D. avoid physical activities that cause exertion. E. discontinue the drug if it is ineffective after two weeks.

A,B Instruct patient to not abruptly discontinue as this may cause seizures and take adequate fluid intake to decrease risk of renal stone development. Topiramate must be used for 2 to 3 months to determine its effectiveness.

Histamine release leads to which of the following? (Select all that apply.) a. Bronchial smooth muscle contraction b. Bronchoconstriction c. Decreased vascular permeability d. Vasoconstriction e. Edema

A,B,E Histamine contracts bronchial smooth muscle, which causes bronchoconstriction. Vascular permeability, edema, and vasodilation also increase.

You are discharging a patient admitted with a transient ischemic attack (TIA). For which medications do you expect to provide discharge instructions (select all that apply)? A. Clopidogrel (Plavix) B. Enoxaparin (Lovenox) C. Dipyridamole (Persantine) D. Enteric-coated aspirin (Ecotrin) E. Tissue plasminogen activator (tPA)

A,C,D Aspirin is the most frequently used antiplatelet agent. Other drugs used to prevent clot formation include clopidogrel (Plavix), ticlopidine (Ticlid), dipyridamole (Persantine), combined dipyridamole and aspirin (Aggrenox), and anticoagulant drugs, such as oral warfarin (Coumadin). The tPA is a fibrinolytic medication used to treat acute ischemic stroke, not prevent TIAs.

You are providing care for a patient who has been admitted to the hospital with a head injury who requires regular neurologic vital signs. Which assessments are components of the patient's score on the Glasgow Coma Scale (select all that apply)? A. Eye opening B. Abstract reasoning C. Best verbal response D. Best motor response E. Cranial nerve function

A,C,D The three dimensions of the Glasgow Coma Scale are eye opening, best verbal response, and best motor response.

What are the common psychosocial reactions of the patient to the stroke (select all that apply)? A. Depression B. Disassociation C. Intellectualization D. Sleep disturbances E. Denial of the severity of the stroke

A,D,E The patient with a stroke may experience many losses, including sensory, intellectual, communicative, functional, role behavior, emotional, social, and vocational losses. Some patients experience long-term depression and symptoms such as anxiety, weight loss, fatigue, poor appetite, and sleep disturbances. The time and energy required to perform previously simple tasks can result in anger and frustration. Frustration and depression are common in the first year after a stroke. A stroke is usually a sudden, extremely stressful event for the patient, caregiver, family, and significant others. The family is often affected emotionally, socially, and financially, as well as changing roles and responsibilities. Reactions vary considerably but may involve fear, apprehension, denial of the severity of the stroke, depression, anger, and sorrow.

A nurse is providing medication to a client receiving Phenytoin (Dilantin). The nurse tells the client that: A. Good oral hygiene is needed, including brushing and flossing B. The daily medication dose should be taken before a scheduled serum drug level is drawn C. The medication dose may be self-adjusted, depending on side effects. D. Alcohol may be used in moderation while taking this medication

A. Gingival hyperplasia is a side effect of this medication. Therefore the client should have good oral hygiene and regular dental examinations.

What should a nurse teach the client to do to avoid lipodystrophy when self-administering insulin therapy? 1 Exercise regularly. 2 Rotate injection sites. 3 Use the Z-track technique. 4 Avoid massaging the injection site.

2. Fibrous scar tissue can result from the trauma of repeated injections at the same site. Exercise is unrelated to lipodystrophy, but it reduces blood glucose, which decreases insulin requirements. Insulin is given subcutaneously; the Z-track technique is used with some intramuscular injections. Gentle pressure over the injection site after insulin administration promotes absorption.

An adolescent with type 1 diabetes who has a history of inadequate adherence to therapy is admitted to the hospital with a blood glucose level of 700 mg/dL. A continuous insulin infusion is started. What complication should the nurse make a priority of detecting while the adolescent is receiving the infusion? 1 Hypovolemia 2 Hypokalemia 3 Hypernatremia 4 Hypercalcemia

2. Insulin causes potassium to move into the cells along with glucose, thereby reducing the serum potassium level. Hypokalemia can lead to lethal cardiac dysrhythmias. Insulin does not result in reduced blood volume, alter the sodium level directly, or affect calcium mobilization.

A client with type 1 diabetes is diagnosed with diabetic ketoacidosis and initially treated with intravenous (IV) fluids followed by an IV bolus of regular insulin. The nurse anticipates that the health care provider will prescribe a continuous infusion of: 1 Novolin L insulin. 2 Novolin R insulin. 3 Novolin N insulin. 4 Novolin U insulin.

2. Regular insulin is the only insulin that is administered intravenously. Novolin L insulin cannot be administered intravenously. Novolin N insulin cannot be administered intravenously. Novolin U insulin cannot be administered intravenously.

A client has a tentative diagnosis of Cushing syndrome. The nurse's physical assessment of this client is likely to reveal the presence of: 1.Fever and tachycardia 2.Lethargy and constipation 3.Hypertension and moon face 4.Hyperactivity and exophthalmos

3.Hypertension and moon face

When a patient wants to know how many vertebrae make up the vertebral column, the nurse responds with _____

33

4. During HIV infection a. reverse transcriptase helps HIV fuse with the CD4+ T cell. b. HIV RNA uses the CD4+ T cell's mitochondria to replicate. c. the immune system is impaired predominantly by the eventual widespread destruction of CD4+ T cells. d. a long period of dormancy develops during which HIV cannot be found in the blood and there is little viral replication.

4. Correct answer: c Immune dysfunction in HIV disease is caused predominantly by damage to and destruction of CD4+ T cells (i.e., T helper cells or CD4+ T lymphocytes).

At 4:30 PM, a client who is receiving human insulin (Humulin N) every morning states, "I feel very nervous." The nurse observes that the client's skin is moist and cool. What is the nurse's most accurate interpretation of what the client is likely experiencing? 1.Polydipsia 2.Ketoacidosis 3.Glycogenesis 4.Hypoglycemia

4.Hypoglycemia

A client with type 1 diabetes receives 30 units of Humulin N insulin at 7 am. At 3:30 pm the client becomes diaphoretic, weak, and pale. The nurse determines that these physiological responses are associated with: 1.Diabetic coma 2.Somogyi effect 3.Diabetic ketoacidosis 4.Hypoglycemic reaction

4.Hypoglycemic reaction

A nurse administers a tube of glucose gel to a client who is hypoglycemic. What should the nurse consider about this reversal of hypoglycemia? 1.It liberates glucose from hepatic stores of glycogen. 2.Insulin action is blocked as it competes for tissue sites. 3.Glycogen is supplied to the brain as well as other vital organs. 4.It provides a glucose substitute for rapid replacement of deficits

4.It provides a glucose substitute for rapid replacement of deficits

An 8-year-old child is being given insulin glargine (Lantus) before breakfast. What is the most appropriate information for the nurse to give the parents concerning a bedtime snack? 1 Offer a snack to prevent hypoglycemia during the night. 2 Give the child a snack if signs of hyperglycemia are present. 3 Avoid a snack because the child is being treated with long-acting insulin. 4 Keep a snack at the bedside in case the child gets hungry during the night.

1. Insulin glargine is released continuously throughout the 24-hour period; a bedtime snack will prevent hypoglycemia during the night. Providing a snack when signs of hyperglycemia are present is unsafe because it intensifies hyperglycemia; if hyperglycemia is present, the child needs insulin. Because insulin glargine is a long-acting insulin, bedtime snacks are recommended to prevent a hypoglycemic episode during the night. When hypoglycemia develops, the child will be asleep; the snack should be eaten before going to bed.

A client with type 1 diabetes has an above-the-knee amputation because of severe lower extremity arterial disease. What is the nurse's primary responsibility two days after surgery when preparing the client to eat dinner? 1.Checking the client's serum glucose level 2.Assisting the client out of bed into a chair 3.Placing the client in the high-Fowler position 4.Ensuring the client's residual limb is elevated

1.Checking the client's serum glucose level

A client is admitted to the hospital with diabetic ketoacidosis. The nurse identifies that the elevated ketone level present with this disorder is caused by the incomplete oxidation of: 1.Fats 2.Protein 3.Potassium 4.Carbohydrates

1.Fats

A nurse prepares to administer metformin (Glucophage XR) to an older adult who has asked that it be crushed because it is difficult to swallow. The nurse explains that this drug cannot be crushed because it: 1.Is released slowly. 2.Is difficult to crush. 3.Irritates mucosal tissue. 4.Has an unpleasant taste if crushed

1.Is released slowly.

The nurse provides education about signs and symptoms of hypoglycemia to a client with newly diagnosed type 1 diabetes. The nurse concludes that the teaching was effective when the client states, "I will drink orange juice and eat a slice of bread when I feel: 1.Nervous and weak." 2.Flushed and short of breath." 3.Thirsty and have a headache." 4.Nauseated and have abdominal cramps."

1.Nervous and weak."

After assessing a client, a nurse concludes that the client may be experiencing hyperglycemia. Which clinical findings commonly associated with hyperglycemia support the nurse's conclusion? (Select all that apply.) 1.Polyuria 2.Polydipsia 3.Polyphagia 4.Polyphrasia 5.Polydysplasia

1.Polyuria 2.Polydipsia 3.Polyphagia

11. Which strategy can the nurse teach the patient to eliminate the risk of HIV transmission? a. Using sterile equipment to inject drugs b. Cleaning equipment used to inject drugs c. Taking lamivudine (Epivir) during pregnancy d. Using latex or polyurethane barriers to cover genitalia during sexual contact

11. Correct answer: a Rationale: Access to sterile equipment is an important risk-elimination tactic. Some communities have needle and syringe exchange programs (NSEPs) that provide sterile equipment to users in exchange for used equipment. Cleaning equipment before use is a risk-reducing activity. It decreases the risk when equipment is shared, but it takes time, and a person in drug withdrawal may have difficulty cleaning equipment. Lamivudine alone is not appropriate for treatment in pregnancy. Barrier methods reduce but do not eliminate risk.

A nurse is monitoring intracranial pressure. A nurse recalls the normal upper limit of intracranial pressure is ___ mm Hg

14

A client visits the health care provider's office for a routine physical examination and reports a new onset of intolerance to cold. Since hypothyroidism is suspected, which additional information would be noted during the client's assessment? 1. Weight loss and tachycardia 2. Complaints of weakness and lethargy 3. Diaphoresis and increased hair growth 4. Increased heart rate and respiratory rate

2

A client who visits the health care provider's office for a routine physical reports new onset of intolerance to cold. Knowing that this is a frequent complaint associated with hypothyroidism, the nurse should check for which manifestations? 1. Weight loss and thinning skin 2. Complaints of weakness and lethargy 3. Diaphoresis and increased hair growth 4. Increased heart rate and respiratory rate

2

A nurse has a prescription to administer a medication to a client who is experiencing shivering as a result of hyperthermia. Which medication should the nurse anticipate to be prescribed? 1. Buspirone (BuSpar) 2. Chlorpromazine (Thorazine) 3. Prochlorperazine (Compazine) 4. Fluphenazine (Prolixin Decanoate)

2

The nurse has applied a hypothermia blanket to a client with a fever. A priority for the nurse is to inspect the skin frequently to detect which complication of hypothermia blanket use? 1. Frostbite 2. Skin breakdown 3. Venous insufficiency 4. Arterial insufficiency

2

The nurse is performing an assessment on a client with a diagnosis of hyperthyroidism. Which assessment finding should the nurse expect to note in this client? 1. Dry skin 2. Bulging eyeballs 3. Periorbital edema 4. Coarse facial features

2

The nursing instructor asks a nursing student to identify the risk factors associated with the development of thyrotoxicosis. The student demonstrates understanding of the risk factors by identifying an increased risk for thyrotoxicosis in which client? 1. A client with hypothyroidism 2. A client with Graves' disease who is having surgery 3. A client with diabetes mellitus scheduled for a diagnostic test 4. A client with diabetes mellitus scheduled for débridement of a foot ulcer

2

5. Which statements accurately describe HIV infection (select all that apply)? a. Untreated HIV infection has a predictable pattern of progression. b. Late chronic HIV infection is called acquired immunodeficiency syndrome (AIDS). c. Untreated HIV infection can remain in the early chronic stage for a decade or more. d. Untreated HIV infection usually remains in the early chronic stage for 1 year or less. e. Opportunistic diseases occur more often when the CD4+ T cell count is high and the viral load is low.

5. Correct answers: a, b, c The typical course of untreated HIV infection follows a predictable pattern. However, treatment can significantly alter this pattern, and disease progression is highly individualized. Late chronic infection is another term for acquired immunodeficiency syndrome (AIDS). The median interval between untreated HIV infection and a diagnosis of AIDS is about 11 years.

6. A diagnosis of AIDS is made when an HIV-infected patient has a. a CD4+ T cell count below 200/µL. b. a high level of HIV in the blood and saliva. c. lipodystrophy with metabolic abnormalities. d. oral hairy leukoplakia, an infection caused by Epstein-Barr virus.

6. Correct answer: a AIDS is diagnosed when an individual with HIV infection meets one of several criteria; one criterion is a CD4+ T cell count below 200 cells/L. Other criteria are listed in Table 14-10.

8. HIV antiretroviral drugs are used to a. cure acute HIV infection. b. decrease viral RNA levels. c. treat opportunistic diseases. d. decrease pain and symptoms in terminal disease.

8. Correct answer: b Rationale: The goals of drug therapy in HIV infection are to (1) decrease the viral load, (2) maintain or raise CD4+ T cell counts, and (3) delay onset of HIV infection-related symptoms and opportunistic diseases.

9. Opportunistic diseases in HIV infection a. are usually benign. b. are generally slow to develop and progress. c. occur in the presence of immunosuppression. d. are curable with appropriate drug interventions.

9. Correct answer: c Rationale: Management of HIV infection is complicated by the many opportunistic diseases that can develop as the immune system deteriorates (Table 14-10).

35. A person with type O blood is considered to be the universal blood donor because type O blood contains which of these? a. No antigens b. No antibodies c. Both A and B antigens d. Both A and B antibodies

ANS: A Because individuals with type O blood lack both types of antigens, they are considered universal donors, meaning that anyone can accept their red blood cells. Type O individuals, who have neither A or B antigen but have both anti-A and anti-B antibodies, cannot accept blood from any of the other three types.

An immunologist is discussing endotoxin production. Which information should the immunologist include? Endotoxins are produced by: a. gram-negative bacteria. b. gram-positive bacteria. c. gram-negative fungi. d. gram-positive fungi.

ANS: A Endotoxins are produced by gram-negative bacteria. They are not produced by gram-positive bacteria or any type of fungi.

10. When mismatched blood is administered causing an ABO incompatibility, how are the erythrocytes destroyed? a. Complement-mediated cell lysis b. Phagocytosis by macrophages c. Phagocytosis in the spleen d. Natural killer cells

ANS: A Erythrocytes are destroyed by complement-mediated lysis in individuals with autoimmune hemolytic anemia or as a result of an alloimmune reaction to ABO-mismatched transfused blood cells. Erythrocytes are not destroyed by phagocytosis or natural killer cells.

What is the nurse's best response about developing diabetes to the patient whose father has type 1 diabetes mellitus? a. "You have a greater susceptibility for development of the disease because of your family history." b. "Your risk is the same as the general population, because there is no genetic risk for development of type 1 diabetes." c. "Type 1 diabetes is inherited in an autosomal dominant pattern. Therefore the risk for becoming diabetic is 50%." d. "Because you are a woman and your father is the parent with diabetes, your risk is not increased for eventual development of the disease. However, your brothers will become diabetic."

ANS: A Even though type 1 diabetes does not follow a specific genetic pattern of inheritance, those with one parent with type 1 diabetes are at an increased risk for development of the disease.

On admission to the clinic, the nurse notes a moderate amount of serous exudate leaking from the patient's wound. The nurse realizes what information about this fluid? a. Contains the materials used by the body in the initial inflammatory response. b. Indicates that the patient has an infection at the site of the wound. c. Is destroying healthy tissue. d. Results from ineffective cleansing of the wound area.

ANS: A Exudate is fluid moved from the vascular spaces to the area around a wound. It contains the proteins, fluid, and white blood cells (WBCs) needed to contain possible pathogens at the site of injury. Exudate appears as part of all inflammatory responses and does not mean an infection is present. Exudate is part of normal inflammatory responses which contain self-monitoring mechanisms to help prevent damage to healthy tissue. Exudate appears at wound sites regardless of cleaning done to the area of injury.

34. Which statement is true regarding immunodeficiency? a. Immunodeficiency is generally not present in other family members. b. Immunodeficiency is never acquired; rather, it is congenital. c. Immunodeficiency is almost immediately symptomatic. d. Immunodeficiency is a result of a postnatal mutation.

ANS: A Generally, the genetic mutations that cause immunodeficiency are sporadic and not inherited; a family history exists in only approximately 25% of individuals. The sporadic mutations occur before birth, but the onset of symptoms may be early or later, depending on the particular syndrome. The immunodeficiency can be either congenital or acquired.

A patient has ICP monitoring with an intraventricular catheter. A priority nursing intervention for the patient is a. aseptic technique to prevent infection b. constant monitoring of ICP waveforms c. removal of CSF to maintain normal ICP d. sampling CSF to determine abnormalities

A. Aseptic technique to prevent infection- An intraventricular catheter is a fluid coupled system that can provide direct access for microorganisms to enter the ventricles of the brain, and aseptic technique is a very high nursing priority to decrease the risk for infection. Constant monitoring of ICP waveforms is not usually necessary, and removal of CSF for sampling or to maintain normal ICP is done only when specifically ordered

The nurse is preparing to teach a 43-yr-old man who is newly diagnosed with type 2 diabetes about home management of the disease. Which action should the nurse take first? a. Assess the patient's perception of what it means to have diabetes. b. Ask the patient's family to participate in the diabetes education program. c. Demonstrate how to check glucose using capillary blood glucose monitoring. d. Discuss the need for the patient to actively participate in diabetes management.

A. Assess the patient's perception of what it means to have diabetes.

A patient has a nursing diagnosis of risk for ineffective cerebral tissue perfusion related to cerebral edema. An appropriate nursing intervention for the patient is a. avoiding positioning the patient with neck and hip flexion b. maintaining hyperventilation to a PaCO2 of 15 to 20 mm Hg c. clustering nursing activities to provide periods of uninterrupted rest d. routine suctioning to prevent accumulation of respiratory secretions

A. Avoiding positioning the patient with neck and hip flexion- Nursing care activities that increase ICP include hip and neck flexion, suctioning, clustering care activities, and noxious stimuli; they should be avoided or performed as little as possible in the patient with increased ICP. Lowering the PaCO2 below 20 mm Hg can cause ischemia and worsening of ICP; the PaCO2 should be maintained at 30 to 35 mm Hg.

Which is a noninvasive method that the nurse uses to assess a​ client's temperature?​ (Select all that​ apply.) A. Axillary B. Rectal C. Oral D. Temporal artery E. Tympanic membrane

A. Axillary D. Temporal Artery

A nurse is preparing to teach about nerves. Which information should the nurse include? The axon leaves the cell body at the: A. Axon hillock B. Nissl Body C. Node of Ranvier D. Myselin Sheath

A. Axon Hillock

Which information will the nurse include in teaching a female patient who has peripheral arterial disease, type 2 diabetes, and sensory neuropathy of the feet and legs? a. Choose flat-soled leather shoes. b. Set heating pads on a low temperature. c. Use a callus remover for corns or calluses. d. Soak feet in warm water for an hour each day.

A. Choose flat-soled leather shoes.

The nurse is assessing a client with a recent head injury. Upon previous​ assessment, the​ client's vital signs were blood pressure​ 140/90 mmHg, pulse 74​ beats/min, and respirations 22​ breaths/min and irregular. Upon this​ assessment, the nurse notes a blood pressure of​ 152/70 mmHg, pulse of 48​ beats/min, and more irregular respirations. Which condition does the nurse suspect is​ occurring? A. Cushing triad B. Autonomic dysreflexia C. Decreasing intracranial pressure​ (ICP) D. Epidural hematoma

A. Cushing triad

Which nursing action is most important in assisting an older patient who has diabetes to engage in moderate daily exercise? a. Determine what types of activities the patient enjoys. b. Remind the patient that exercise improves self-esteem. c. Teach the patient about the effects of exercise on glucose level. d. Give the patient a list of activities that are moderate in intensity.

A. Determine what types of activities the patient enjoys.

The nurse is listing consequences of malignant hyperthermia. Which consequences should be included? (select all that apply) A. Disseminated intravascular coagulation B. Renal failure C. Cardiac dysrhythmias D. Gastroenteritis E. Pulmonary edema

A. Disseminated intravascular coagulation B. Renal failure C. Cardiac dysrhythmias E. Pulmonary edema

The nurse is monitoring a client with increased intracranial pressure who is experiencing status epilepticus. Which intervention has the highest priority for this​ client? A. Establishing and maintaining an airway B. Administering a solution of​ 50% dextrose C. Administering diazepam intravenously D. Contacting the provider for orders

A. Establishing and maintaining an airway

The nurse has administered 4 oz of orange juice to an alert patient whose blood glucose was 62 mg/dL. Fifteen minutes later, the blood glucose is 67 mg/dL. Which action should the nurse take next? a. Give the patient 4 to 6 oz more orange juice. b. Administer the PRN glucagon (Glucagon) 1 mg IM. c. Have the patient eat some peanut butter with crackers. d. Notify the health care provider about the hypoglycemia.

A. Give the patient 4 to 6 oz more orange juice.

The nurse is preparing a presentation to parents about vehicle safety and​ heat-related injuries. Which important teaching point should be​ included? A. Keep important articles in the backseat to ensure checking the area before leaving the vehicle. B. Store car keys in a visible place and within the​ children's reach in case of an emergency. C. Leave a child alone in the car only if the outside temperature is below 80degrees°F. D. Stay with a child in the car for up to 10 minutes with the windows cracked open.

A. Keep important articles in the backseat to ensure checking the area before leaving the vehicle ​Rationale: Estimates indicate that numerous children die from vehicle​ hyperthermia, or sustain heat​ exhaustion, heat​ stroke, and thermal burn after being left in vehicles on warm days. One way of ensuring that such incidents do not happen is to teach caregivers to place something​ important, such as their wallet or cell​ phone, in the backseat of the car. This will ensure that they check the backseat before leaving the vehicle. Advise them to always look before they​ lock, when not in use. According to reports and​ findings, leaving a child in a car with a cracked window for even a short amount of​ time, holds the potential for lethal consequences. Children should never be left unattended or around vehicles. Though it is a good practice to keep car keys in a visible​ place, ensure that they are out of reach of children.

A patient with diabetes is starting on intensive insulin therapy. Which type of insulin will the nurse discuss using for mealtime coverage? a. Lispro (Humalog) b. Glargine (Lantus) c. Detemir (Levemir) d. NPH (Humulin N)

A. Lispro (Humalog)

Which nursing action can the nurse delegate to experienced unlicensed assistive personnel (UAP) who are working in the diabetic clinic? a. Measure the ankle-brachial index. b. Check for changes in skin pigmentation. c. Assess for unilateral or bilateral foot drop. d. Ask the patient about symptoms of depression.

A. Measure the ankle-brachial index.

A student nurse asks the nurse what controls reflex activities concerned wtih heart rate and blood pressure. What is the nurses best response? These reflext activities are controlled by the: a. Medulla oblangata b. Pons c. Midbrain d. Cerebrum

A. Medulla Oblongata

A 19-year old male has sustained a transaction of C-7 in an MVA reddening his quadriplegic. He describes his pain as burning, sharp, and shooting. This is characteristic of A. Neuropathic pain B. Ghost pain C. Mixed pain syndrome D. Nociceptive pain

A. Neuropathic pain Damage to the brain, spinal cord, peripheral nerves resulting in burning or shooting sensation

When providing health promotion teaching at a senior citizen's center, the nurse would include information about which of the following medications used to decrease the duration of influenza A and B? A) oseltamivir (Tamiflu) B) ganciclovir (Cytovene) C) enfuvirtide (Fuzeon) D) indinavir (Crixivan)

A. Oseltamivir (Tamiflu) and zanamivir (Relenza) are active against influenza virus types A and B and have been shown to reduce the duration of influenza infection by several days.

A client with increased intracranial pressure​ (IICP) is also hyponatremic. Which intravenous medication should the nurse anticipate administering to address fluid​ removal? A. Osmotic diuretic B. Antiepileptic C. Corticosteroid D. Proton pump inhibitor

A. Osmotic diuretic ​Rationale: Osmotic diuretics are used in clients with IICP to promote water excretion and to address hyponatremia

11. Which information would be most important to help the nurse determine if the patient needs human immunodeficiency virus (HIV) testing? a. Patient age b. Patient lifestyle c. Patient symptoms d. Patient sexual orientation

A. Patient age The current CDC policy is to offer routine testing for HIV to all individuals age 13 to 64. Although lifestyle, symptoms, and sexual orientation may suggest increased risk for HIV infection, the goal is to test all individuals in this age range.

Pramlintide (Symlin) is prescribed as supplemental drug therapy to the treatment plan for a patient with type 1 diabetes mellitus. What information should the nurse include when teaching the patient about the action of this medication? A. Pramlintide slows gastric emptying. B. Pramlintide increases glucagon excretion. C. Pramlintide stimulates glucose production. D. Pramlintide corrects insulin receptor sensitivity.

A. Pramlintide slows gastric emptying. Correct Pramlintide is a synthetic form of the naturally occurring hormone amylin. It works by slowing gastric emptying, suppressing glucagon secretion and hepatic glucose production, and increasing satiety (sense of having eaten enough). It is only administered via subcutaneously injection.

Controlling pain is important to promoting wellness. Unrelieved pain has been associated with a. Prolonged stress response and a cascade of harmful effects system wide. b. Large tidal volumes and decreased lung capacity c. Decreased tumor growth and longevity d. Decreased carbohydrate, protein, and fat destruction

A. Prolonged stress response and a cascade of harmful effects system-wide

Innate immunity utilizes cells that produce cytokines, which mediate immune cell actions. How are the actions of cytokines described? (Select all) A. Rapid and self limiting B. Pleotrophic and redundant C. Cell specific and targeted D. Dendritic and morphologic E. Cell specific and morphologic

A. Rapid and self limiting B. Pleotrophic and redundant

Which actions describe the beneficial effects produced by sulfonylurea oral hypoglycemics? (Select all that apply.) A. Stimulate insulin secretion from beta cells B. Increase hepatic glucose production C. Enhance action of insulin in various tissues D. Inhibit breakdown of insulin by liver

A. Stimulate insulin secretion from beta cells Correct C. Enhance action of insulin in various tissues Correct D. Inhibit breakdown of insulin by liver Correct The sulfonylureas stimulate insulin secretion from the beta cells of the pancreas; enhance the actions of insulin in muscle, liver, and adipose tissue; and prevent the liver from breaking insulin down as fast as it ordinarily would (reduced hepatic clearance). Increased hepatic glucose production would serve to

Which actions describe the beneficial effects produced by sulfonylurea oral hypoglycemics? (Select all that apply.) A. Stimulate insulin secretion from beta cells B. Increase hepatic glucose production C. Enhance action of insulin in various tissues D. Inhibit breakdown of insulin by liver

A. Stimulate insulin secretion from beta cells Correct C. Enhance action of insulin in various tissues Correct D. Inhibit breakdown of insulin by liver Correct The sulfonylureas stimulate insulin secretion from the beta cells of the pancreas; enhance the actions of insulin in muscle, liver, and adipose tissue; and prevent the liver from breaking insulin down as fast as it ordinarily would (reduced hepatic clearance). Increased hepatic glucose production would serve to increase serum glucose levels, the opposite effect of oral hypoglycemic drugs.

A patient presents to the emergency room complaining of vomiting with severe back and leg pain. The patient's home medications include daily oral corticosteroids. Vital signs reveal a low blood pressure and there are peaked T waves on the electrocardiogram. What is the nurse's priority intervention? a. Start an intravenous line b. Collect urine specimen c. Administer antiemetic d. Administer narcotic analgesia

A. start an intravenous line The patient is exhibiting signs of adrenal insufficiency (Addison's disease) given the regular use of corticosteroids. Cortisone, hydrocortisone (Cortef), prednisone, and fludrocortisone (Florinef) are used for the treatment of adrenocorticoid deficiency. Treatment of Addisonian crisis includes administration of hydrocortisone, saline solution, and sugar (dextrose) to correct the insufficiency. The priority intervention is to start an intravenous line so that appropriate treatments may be administered. A urine specimen may be collected but is not the priority intervention. Since the patient is vomiting, administration of antiemetics or analgesia would be given through an intravenous line. The nurse should also assess for changes in the level of consciousness; so administration of analgesia may be contraindicated if any decrease in level of consciousness occurs.

1. A patient who has vague symptoms of fatigue and headaches is found to have a positive enzyme immunoassay (EIA) for human immunodeficiency virus (HIV) antibodies. In discussing the test results with the patient, the nurse informs the patient that a. the EIA test will need to be repeated to verify the results. b. a viral culture will be done to determine the progress of the disease. c. it will probably be 10 or more years before the patient develops acquired immunodeficiency syndrome (AIDS). d. the Western blot test will be done to determine whether AIDS has developed.

A. the EIA test will need to be repeated to verify the results After an initial positive EIA test, the EIA is repeated before more specific testing such as the Western blot is done. Viral cultures are not part of HIV testing. Because the nurse does not know how recently the patient was infected, it is not appropriate to predict the time frame for AIDS development. The Western blot tests for HIV antibodies, not for AIDS.

A patient is suspected of having a cranial tumor. The signs and symptoms include memory deficits, visual disturbances, weakness of right upper and lower extremities, and personality changes. You recognize that the tumor is most likely located in which lobe? A. Frontal lobe B. Parietal lobe C. Occipital lobe D. Temporal lobe

A A frontal lobe tumor may result in hemiplegia, seizures, memory deficit, personality and judgment changes, and visual disturbances.

The patient had an acute ischemic stroke 4 hours ago and has an elevated blood pressure. What action should you take? A. Document the findings because the increased pressure is needed to perfuse the brain. B. Administer an antihypertensive medication to prevent additional damage. C. Hyperventilate the patient to cause vasodilatation. D. Teach patient about a low sodium diet.

A After a stroke, temporary hypertension is needed to perfuse the area of swelling. No treatment is done unless the pressure is above 220/110 mm Hg in the first few hours. Aggressive lowering of blood pressure is not done, because if the pressure drops, it can prevent regional perfusion and lead to local tissue damage. Hyperventilation is done if hypercapnia is identified, but it is not prophylactic.

When assessing a patient with a traumatic brain injury, you notice uncoordinated movement of the extremities. How would you document this? A. Ataxia B. Apraxia C. Anisocoria D. Anosognosia

A Ataxia is a lack of coordination of movement, possibly caused by lesions of sensory or motor pathways, cerebellar disorders, or certain medications.

The nurse is aware that viruses can enter the body through various routes. By which methods can viruses enter the body? (Select all that apply.) A. Inhalation through the respiratory tract B. Ingestion via the gastrointestinal (GI) tract C.Transplacentally from mother to infant D.Through an animal bite

ALL Viruses can enter the body through at least four routes: inhalation through the respiratory tract, ingestion via the GI tract, transplacentally via mother to infant, and inoculation via skin or mucous membranes. The inoculation route can take several forms, including sexual contact, blood transfusions, sharing of syringes or needles, organ transplantation, and animal bites (including human, animal, insect, spider, and others).

Social effects of a chronic neurologic disease include (select all that apply) A. divorce. B. job loss. C. depression. D. role changes. E. loss of self-esteem.

ALL OF THEM Social problems related to chronic neurologic disease may include changes in roles and relationships (such as divorce, job loss, and role changes); other psychologic problems (such depression and loss of self-esteem) may also have social effects. Reference: 1510

An older adult who takes medications for coronary artery disease has just been diagnosed with asymptomatic chronic human immunodeficiency virus (HIV) infection. Which information will the nurse include in patient teaching? a. Many medications have interactions with antiretroviral drugs. b. Less frequent CD4+ level monitoring is needed in older adults. c. Hospice care is available for patients with terminal HIV infection. d. Progression of HIV infection occurs more rapidly in older patients.

ANS: A The nurse will teach the patient about potential interactions between antiretrovirals and the medications that the patient is using for chronic health problems. Treatment and monitoring of HIV infection is not affected by age. A patient with asymptomatic HIV infection is not a candidate for hospice. Progression of HIV is not affected by age, although it may be affected by chronic disease.

18. A healthcare professional is teaching a patient about Raynaud phenomenon and instructs the patient to avoid cold. What is the best explanation of how cold impacts the manifestations of this disease? a. Immune complexes that are deposited in capillary beds, blocking circulation b. Mast cells that are bound to specific endothelial receptors, causing them to degranulate and creating a localized inflammatory reaction that occludes capillary circulation c. Cytotoxic T cells that attack and destroy the capillaries so that they are unable to perfuse local tissues d. Antibodies that detect the capillaries as foreign protein and destroy them using lysosomal enzymes and toxic oxygen species

ANS: A Raynaud phenomenon is a condition (type III hypersensitivity reaction) caused by the temperature-dependent deposition of immune complexes in the capillary beds of the peripheral circulation. The healthcare professional should teach the patient to avoid cold or to wear gloves to mitigate the cold. None of the other options are involved in causing this condition.

22. A patient asks the healthcare professional why tissue damage occurs in acute rejection after organ transplantation. What response by the professional is best? a. Th1 cells release cytokines that activate infiltrating macrophages, and cytotoxic T cells directly attack the endothelial cells of the transplanted tissue. b. Circulating immune complexes are deposited in the endothelial cells of transplanted tissue, where the complement cascade lyses tissue. c. Receptors on natural killer cells recognize antigens on the cell surface of transplanted tissue, which releases lysosomal enzymes that destroy tissue. d. Antibodies coat the surface of transplanted tissue to which mast cells bind and liberate preformed chemical mediators that destroy tissue.

ANS: A The recipient's lymphocytes interacting with the donor's dendritic cells within the transplanted tissue usually initiate sensitization, resulting in the induction of recipient Th1 and Tc cells against the donor's antigens. The Th1 cells release cytokines that activate infiltrating macrophages, and the Tc cells directly attack the endothelial cells in the transplanted tissue. The other options do not accurately describe how acute rejection after organ transplantation results in tissue damage.

3. The common hay fever allergy is expressed through a reaction that is mediated by which class of immunoglobulins? a. IgE b. IgG c. IgM d. T cells

ANS: A Type I reactions are mediated by antigen-specific IgE and the products of tissue mast cells. The most common allergies (e.g., pollen allergies) are type I reactions. In addition, most type I reactions occur against environmental antigens and are therefore allergic. Hay fever allergy is not mediated by IgG, IgM, or T cells.

The nurse is teaching a group of patients about management of diabetes. Which statement about basal dosing is correct? a. "Basal dosing delivers a constant dose of insulin." b. "With basal dosing, you can eat what you want and then give yourself a dose of insulin." c. "Glargine insulin is given as a bolus with meals." d. "Basal-bolus dosing is the traditional method of managing blood glucose levels."

ANS: A Basal-bolus therapy is the attempt to mimic a healthy pancreas by delivering basal insulin constantly as a basal, and then as needed as a bolus. Glargine insulin is used as a basal dose, not as a bolus with meals. Basal-bolus therapy is a newer therapy; historically, sliding-scale coverage was implemented. DIF: COGNITIVE LEVEL: Applying (Application) REF: p. 508 TOP: NURSING PROCESS: Implementation MSC: NCLEX: Physiological Integrity: Pharmacological and Parenteral Therapies

Which action is most appropriate regarding the nurse's administration of a rapid-acting insulin to a hospitalized patient? a. Give it within 15 minutes of mealtime. b. Give it after the meal has been completed. c. Administer it once daily at the time of the midday meal. d. Administer it with a snack before bedtime.

ANS: A Rapid-acting insulins, such as insulin lispro and insulin aspart, are able to mimic closely the body's natural rapid insulin output after eating a meal; for this reason, both insulins are usually administered within 15 minutes of the patient's mealtime. The other options are incorrect. DIF: COGNITIVE LEVEL: Applying (Application) REF: p. 505 TOP: NURSING PROCESS: Planning MSC: NCLEX: Safe and Effective Care Environment: Management of Care

1. The nurse is caring for a patient infected with human immunodeficiency virus (HIV) who has just been diagnosed with asymptomatic chronic HIV infection. Which prophylactic measures will the nurse include in the plan of care (select all that apply)? a. Hepatitis B vaccine b. Pneumococcal vaccine c. Influenza virus vaccine d. Trimethoprim-sulfamethoxazole e. Varicella zoster immune globulin

ANS: A, B, C Asymptomatic chronic HIV infection is a stage between acute HIV infection and a diagnosis of symptomatic chronic HIV infection. Although called asymptomatic, symptoms (e.g., fatigue, headache, low-grade fever, night sweats) often occur. Prevention of other infections is an important intervention in patients who are HIV positive, and these vaccines are recommended as soon as the HIV infection is diagnosed. Antibiotics and immune globulin are used to prevent and treat infections that occur later in the course of the disease when the CD4+ counts have dropped or when infection has occurred. DIF: Cognitive Level: Apply (application) REF: 220 TOP: Nursing Process: Implementation MSC:

40. Which disorders are considered autoimmune? (Select all that apply.) a. Crohn disease b. Addison disease c. Rheumatoid arthritis d. Systemic lupus erythematosus e. Type 2 diabetes

ANS: A, B, C, D Crohn disease, Addison disease, rheumatoid arthritis, and systemic lupus erythematosus are all diseases that result from autoimmune pathologic conditions. Insulin-dependent diabetes is also an autoimmune disorder, but type 2 diabetes is not.

A patient is taking a sulfonylurea medication for new-onset type 2 diabetes mellitus. When reviewing potential adverse effects during patient teaching, the nurse will include information about which of these effects? (Select all that apply.) a. Hypoglycemia b. Nausea c. Diarrhea d. Weight gain e. Peripheral edema

ANS: A, B, D The most common adverse effect of the sulfonylureas is hypoglycemia, the degree to which depends on the dose, eating habits, and presence of hepatic or renal disease. Another predictable adverse effect is weight gain because of the stimulation of insulin secretion. Other adverse effects include skin rash, nausea, epigastric fullness, and heartburn. DIF: COGNITIVE LEVEL: Applying (Application) REF: p. 509 TOP: NURSING PROCESS: Implementation MSC: NCLEX: Physiological Integrity: Pharmacological and Parenteral Therapies

39. Exposure to which of these could result in a type IV hypersensitivity reaction? (Select all that apply.) a. Poison ivy b. Neomycin c. Dairy products d. Nickel e. Detergents

ANS: A, B, D, E Allergens that primarily elicit type IV allergic hypersensitivities include plant resins (e.g., poison ivy, poison oak); metals (e.g., nickel, chromium); acetylates and chemicals in rubber, cosmetics, detergents; and topical antibiotics (e.g., neomycin). Food products typically result in type I hypersensitivity reactions.

25. A 30-year-old male is diagnosed with a hormone-secreting tumor of the pancreas alpha cells. Which of the following would the nurse expect to be most likely increased in this patient? a. Amylin b. Glucagon c. Insulin d. Somatostatin

ANS: B Glucagon is produced by the alpha cells of the pancreas. Amylin is secreted by the beta cells. Insulin is secreted by the beta cells. Somatostatin is produced by the delta cells.

24. A nurse is teaching a patient about insulin. Which information should the nurse include? Insulin is primarily regulated by: a. Metabolic rate b. Serum glucose levels c. Prostaglandins d. Enzyme activation

ANS: B Insulin secretion is promoted when blood levels of glucose rise. Insulin secretion is not based on metabolic rate but on blood levels of glucose. Insulin secretion is not based on prostaglandins but on blood levels of glucose. Insulin secretion is not based on enzyme activation but on blood levels of glucose.

Which question during the assessment of a patient who has diabetes will help the nurse identify autonomic neuropathy? a. "Do you feel bloated after eating?" b. "Have you seen any skin changes?" c. "Do you need to increase your insulin dosage when you are stressed?" d. "Have you noticed any painful new ulcerations or sores on your feet?"

A. "Do you feel bloated after eating?"

A patient is taking metformin for new-onset type 2 diabetes mellitus. When reviewing potential adverse effects, the nurse will include information about: (Select all that apply.) A. Abdominal bloating B. Nausea C. Diarrhea D. Headache E. Weight gain F. Metallic taste

A. Abdominal bloating B. Nausea C. Diarrhea F. Metallic taste

A patient begins taking a new drug that causes pupil dialation, vasoconstriction, decreased gastrointestinal motility, and good bumps. Which of the following receptors are activated? A. Alpha 1 B. Alpha 2 C. Beta 1 D. Beta 2

A. Alpha 1

Which of the following are neurological cells? a. Astrocyte b. Oligodendrocyte c. Neuron d. Ependymal cell e. Melanocyte

A. Astrocyte, B. Oligodendrocyte, D. Ependymal

Alan is a 30-year-old male admitted to the hospital with acute pancreatitis. He is in acute pain described as a 10/10, which is localized to the abdomen, periumbilical area, and some radiation to his back. The abdomen is grossly distended so it is difficult to assess. He is restless and agitated, with elevated pulse and blood pressure. An appropriate pain management plan of care may include A. IV Dilaudid q 4 hours PRN, hydrocodone 5/500 PO q 6 hours PRN, and acetaminophen. B. Norco 5/500 q 4 hours PO and Benadryl 25 mg PO q 6 hours. C. Phenergan 25 mg IM q 6 hours. D. Tylenol 325 mg q 6 hours.

A. IV Dilaudid q4h PRN, Norco 5/500mg q6h PRN, and acetaminophen Oral route whenever feasible, long standing pain = long acting analgesic for baseline pain and fast-onset medication for breakthrough pain

When a patient is admitted to the emergency department following a head injury, the nurse's first priority in management of the patient once a patent airway is confirmed is a. maintaining cervical spine precautions b. determining the presence of increased ICP c. monitoring for changes in neurologic status d. establishing IV access with a large-bore catheter

A. In addition to monitoring for a patent airway during emergency care of the patient with a head injury, the nurse must always assume that a patient with a head injury may have a cervical spine injury. Maintaining cervical spine precautions in all assessment and treatment activities with the patient is essential to prevent additional neurologic damage.

When a patient's vagus nerve is stimulated, what does the nurse expect to observe? a. Increased gastrointestinal activity b. Increased heart rate c. Pupil constriction d. Vasoconstriction

A. Increased gastrointestinal activity

A patient with diabetic ketoacidosis is brought to the emergency department. Which prescribed action should the nurse implement first? a. Infuse 1 L of normal saline per hour. b. Give sodium bicarbonate 50 mEq IV push. c. Administer regular insulin 10 U by IV push. d. Start a regular insulin infusion at 0.1 units/kg/hr.

A. Infuse 1 L of normal saline per hour.

Which long-acting insulin mimics natural, basal insulin with no peak action and a duration of 24 hours? A. Insulin glargine (Lantus) B. Insulin glulisine (Apidra) C. Regular insulin (Humulin R) D. NPH insulin

A. Insulin glargine (Lantus) Insulin glargine has a duration of action of 24 hours with no peaks, mimicking the natural, basal insulin secretion of the pancreas.

A client recovering from a craniotomy complains of a "runny nose". Which of the following nursing actions should be immediately implemented? A. Notify the physician B. Provide the client with soft tissues C. Monitor the client for signs of a cold D. Tell the client to use soft tissues to soak up the drainage

A. If the client has sustained a craniocerebral injury or is recovering from a craniotomy, careful observation of any drainage from the eyes, ears, nose or traumatic area is critical because this may indicate leakage of cerebrospinal fluid. Cerebrospinal fluid is colorless and generally nonpurulent, and its presence indicates a serious breach of cranial integrity. Any suspicious drainage should be reported to the physician immediately.

17. To evaluate the effectiveness of antiretroviral therapy (ART), which laboratory test result will the nurse review? a. Viral load testing c. Rapid HIV antibody testing b. Enzyme immunoassay d. Immunofluorescence assay

ANS: A The effectiveness of ART is measured by the decrease in the amount of virus detectable in the blood. The other tests are used to detect HIV antibodies, which remain positive even with effective ART. DIF: Cognitive Level: Apply (application) REF: 222 TOP: Nursing Process: Evaluation MSC:

The patient has an order for phenytoin (Dilantin) 100 mg q8hr IV. Available is a phenytoin injection containing 50 mg/mL. How many milliliters of solution should you draw up for the dose? A. 0.5 B. 2 C. 5 D. 20

B 100 mg ÷ 50 mg/mL = 2 mL.

. The patient with migraines asks you why "everyone always asks if anyone in the family has a history of migraines." Your priority response is A. "It is standard practice to assess a person's history to identify potential health risks" B. "Around 70% of individuals with migraines have a relative who also has migraines." C. "Assessing your history identifies the treatment plan." D. "This assessment provides a baseline for future assessments."

B Approximately 70% of those with migraine have a first-degree relative who also had migraine headaches. Reference: 1487

ne major goal of treatment for the patient with Huntington's disease is A. disease cure. B. symptomatic relief. C. maintaining employment. D. improving muscle strength.

B Because there is no cure for Huntington's disease (HD), collaborative care is palliative and based on symptom relief. The goal of nursing management is to provide the most comfortable environment possible for the patient and the caregiver by maintaining physical safety, treating the physical symptoms, and providing emotional and psychological support. Reference: 1515-1516

What nursing measure is indicated to reduce the potential for seizures and increased ICP in a patient with bacterial meningitis? A. Administering codeine for relief of head and neck pain B. Controlling fever with prescribed drugs and cooling techniques C. Keeping the room darkened and quiet to minimize environmental stimulation D. Maintaining the patient on strict bed rest with the head of the bed slightly elevated

B Fever must be vigorously managed because it increases cerebral edema and the frequency of seizures. Neurologic damage may result from an extremely high temperature over a prolonged period. Acetaminophen or aspirin may be used to reduce fever; other measures, such as a cooling blanket or sponge baths with tepid water, may be effective in lowering the temperature.

When a diabetic patient asks about maintaining adequate blood glucose levels, which of the following statements by the nurse relates most directly to the necessity of maintaining blood glucose levels no lower than about 74 mg/dl? a. "Glucose is the only type of fuel used by body cells to produce the energy needed for physiologic activity." b. "The central nervous system cannot store glucose and needs a continuous supply of glucose for fuel." c. "Without a minimum level of glucose circulating in the blood, erythrocytes cannot produce ATP." d. "The presence of glucose in the blood counteracts the formation of lactic acid and prevents acidosis."

ANS: B The brain cannot synthesize or store significant amounts of glucose; thus a continuous supply from the body's circulation is needed to meet the fuel demands of the central nervous system.

A patient with a history of chronic obstructive pulmonary disease (COPD) and type 2 diabetes has been treated for pneumonia for the past week. The patient has been receiving intravenous corticosteroids as well as antibiotics as part of his therapy. At this time, the pneumonia has resolved, but when monitoring the blood glucose levels, the nurse notices that the level is still elevated. What is the best explanation for this elevation? a. The antibiotics may cause an increase in glucose levels. b. The corticosteroids may cause an increase in glucose levels. c. His type 2 diabetes has converted to type 1. d. The hypoxia caused by the COPD causes an increased need for insulin.

ANS: B Corticosteroids can antagonize the hypoglycemic effects of insulin, resulting in elevated blood glucose levels. The other options are incorrect. DIF: COGNITIVE LEVEL: Analyzing (Analysis) REF: p. 515 TOP: NURSING PROCESS: Evaluation MSC: NCLEX: Physiological Integrity: Pharmacological and Parenteral Therapies

The nurse is preparing to administer insulin intravenously. Which statement about the administration of intravenous insulin is true? a. Insulin is never given intravenously. b. Only regular insulin can be administered intravenously. c. Insulin aspart or insulin lispro can be administered intravenously, but there must be a 50% dose reduction. d. Any form of insulin can be administered intravenously at the same dose as that is ordered for subcutaneous administration.

ANS: B Regular insulin is the usual insulin product to be dosed via intravenous bolus, intravenous infusion, or even intramuscularly. These routes, especially the intravenous infusion route, are often used in cases of diabetic ketoacidosis, or coma associated with uncontrolled type 1 diabetes. DIF: COGNITIVE LEVEL: Applying (Application) REF: p. 516 TOP: NURSING PROCESS: Implementation MSC: NCLEX: Safe and Effective Care Environment: Safety and Infection Control

32. Raynaud phenomenon is an example of which type of hypersensitivity? a. IV b. III c. II d. I

ANS: B The characteristics of serum sickness are observed in only systemic type III autoimmune diseases such as Raynaud phenomenon.

What common symptom should be assessed in individuals with immunodeficiency? a. Anemia b. Recurrent infections c. Hypersensitivity d. Autoantibody production

ANS: B The clinical hallmark of immunodeficiency is a propensity to unusual or recurrent severe infections. The type of infection usually reflects the immune system defect. Neither anemia, autoantibody production, nor hypersensitivity is a manifestation of immunodeficiency.

28. How many months does it take for the newborn to be sufficiently protected by antibodies produced by its own B cells? a. 1 to 2 b. 4 to 5 c. 6 to 8 d. 10 to 12

ANS: C By 6 to 8 months, the newborn should be efficiently protected by antibodies produced by its own B cells.

Which information indicates a good understanding of bacterial vaccines? Most bacterial vaccines contain: a. fully active bacteria. b. synthetic bacteria. c. dead bacteria. d. bacterial toxins.

ANS: C Vaccines are biologic preparations of weakened or dead pathogens that when administered stimulate production of antibodies or cellular immunity against the pathogen without causing disease. Vaccines are not fully active bacteria, nor are they synthetic or toxins.

A patient has cellulitis on the right forearm. The nurse would anticipate orders to administer medications to eradicate which organism? a. Candida albicans b. Group A beta-hemolytic streptococci c. Staphylococcus aureus d. E. Coli

ANS: C Staphylococcus aureus is the usual cause of cellulitis, although other pathogens may be responsible. A small abrasion or lesion can provide a portal for opportunistic or pathogenic infectious organisms to infect deeper tissues. REF: Page 268

The early stage of AD is characterized by A. no noticeable change in behavior. B. memory problems and mild confusion. C. increased time spent sleeping or in bed. D. incontinence, agitation, and wandering behavior.

B An initial sign of AD is a subtle deterioration in memory.

Which response can be expected in a patient with low oxygen concentration and acidosis? A. Decreased cerebral fluid flow with decreased cerebral pressure B. Vasodilation with increased cerebral pressure C. Systemic hypotension with decreased cerebral pressure D. Cerebral tissue hypertrophy with increased cerebral pressure

B Low concentration of oxygen ions and high concentration of hydrogen ions cause vasodilation, which can result in increased ICP if autoregulation has failed. The other options are not possible

A nurse working at a burn center is caring for a client with an electrical burn. According to the American Burn Association, how would this burn be classified? A) Minor B) Moderate C) Major D) Significant

Answer: C Explanation: According to the American Burn Association, all electrical burns are classified as major. Significant is not a classification according to the American Burn Association, and all other choices are incorrect.

6. The patient is diagnosed with vancomycin-resistant enterococci (VRE) infection in a surgical wound. What infection precautions should the nurse use to best prevent transmission of the infection to others? A. Droplet precautions B. Contact precautions C. Airborne precautions D. Standard precautions

B Contact precautions are used to minimize the spread of pathogens that are acquired from direct or indirect contact. Droplet precautions are used with pathogens that are spread through the air at close contact and that affect the respiratory system or mucous membranes (e.g., influenza, pertussis). Airborne precautions are used if the organism can cause infection over long distances when suspended in the air (e.g., tuberculosis, rubeola). Standard precautions are used with all patients and included in the transmission-based precautions above.

1. The nurse is caring for a patient newly diagnosed with human immunodeficiency virus (HIV). What does the nurse explain to the patient the criteria for diagnosis is based on? A. Presence of HIV antibodies B. CD4+ T cell count below 200/µL C. Presence of oral hairy leukoplakia D. White blood cell count below 5000/µL

B Diagnostic criteria for AIDS include a CD4+ T-cell count below 200/µL or the development of specified opportunistic infections, cancers, wasting syndrome, or dementia. The other options may be found in patients with HIV disease but do not define the advancement of HIV infection to AIDS.

13. The nurse is teaching a group of young adults who live in a dormitory about the prevention of antibiotic-resistant infections. What should be included in the teaching plan? A. Save leftover antibiotics for future uses. B. Hand washing can prevent many infections. C. Antibiotics are indicated for preventing most colds. D. Stop taking prescribed antibiotics when symptoms improve.

B Hand washing is the single most important action to prevent infections. Antibiotics are used to treat bacterial infections, not colds and flu caused by viruses. Patients should complete the entire prescription of antibiotics to prevent the development of resistant bacteria. Antibiotics should not be taken to prevent infections unless they are given prophylactically before undergoing certain surgeries and dental work.

3. The nurse has experienced a recent increase in the incidence of hospital care-associated infections (HAIs) on the unit. Which nursing action should be prioritized in the response to this trend? A. Use of gloves during patient contact B. Frequent and thorough hand washing C. Prophylactic, broad-spectrum antibiotics D. Fitting and appropriate use of N95 masks

B Hand washing remains the mainstay of the prevention of HAIs. Gloves, masks, and antibiotics may be appropriate in specific circumstances, but none of these replaces the central role of vigilant, thorough hand washing between patients and when moving from one task to another, even with the same patient.

Which statement best describes a type I reaction? a. Most type I reactions are mediated by IgA. b. Most type I reactions are allergic. c. Most occur against medications. d. Rarely does this type of reaction contribute to autoimmune diseases.

B Most type I reactions are allergic. They are mediated by IgE. Most type I reactions occur against environmental antigens and can contribute to some autoimmune diseases.

A 40-year-old male suffers from head trauma that affects cranial nerve I. Which of the following symptoms would the nurse expect? A. Visual disturbances B. Loss of sense of smell C. Loss of ability to taste D. Hearing disturbances

B. Loss of sense of smell

The nurse should initiate which nonpharmacologic therapy early in hospitalization for clients who have sustained paralysis due to a disruption in intracranial​ regulation? A. Speech B. Physical C. Psychological D. Occupational

B. Physical

A patient who is diagnosed with shingles is taking topical acyclovir, and the nurse is providing instructions about adverse effects. The nurse will discuss which adverse effects of topical acyclovir therapy? a. Insomnia and nervousness b. Temporary swelling and rash c. Burning when applied d. This medication has no adverse effects.

C Transient burning may occur with topical application of acyclovir. The other options are incorrect.

19. A heterosexual patient is concerned that they may contract human immunodeficiency virus (HIV) from a bisexual partner. What should the nurse include when teaching about preexposure prophylaxis (select all that apply.)? A. Take fluconazole (Diflucan). B. Take amphotericin B (Fungizone). C. Use condoms for risk-reducing sexual relations. D. Take emtricitabine and tenofovir (Truvada) regularly. E. Have regular HIV testing for herself and her husband.

C, D, E Using male or female condoms, having monthly HIV testing for the patient and partner, and taking emtricitabine and tenofovir regularly have shown to decrease the infection of heterosexual women having sex with a partner who participates in high-risk behavior. Fluconazole and amphotericin B are taken for Candida albicans, Coccidioides immitis, and Cryptococcus neoformans, which are all opportunistic diseases associate with HIV infection.

A 32-year-old female suffers from severe brain damage following a motor vehicle accident. After rehabilitation she notices that her through processes and goal-oriented behavior are impaired. Which area does the nurse suspect is damaged? A. Thalamus B. Limbic C. Prefrontal C. Occipital

C. Prefrontal

A hospitalized patient who is diabetic received 38 U of NPH insulin at 7:00 AM. At 1:00 PM, the patient has been away from the nursing unit for 2 hours, missing the lunch delivery while awaiting a chest x-ray. What is the best action by the nurse to prevent hypoglycemia? a. Plan to discontinue the evening dose of insulin. b. Save the lunch tray for the patient's later return. c. Request that if testing is further delayed, the patient will eat lunch first. d. Send a glass of orange juice to the patient in the diagnostic testing area.

C. Request that if testing is further delayed, the patient will eat lunch first.

Mechanical ventilation with a rate and volume to maintain a mild hyperventilation is used for a patient with a head injury. To evaluate the effectiveness of the therapy, the nurse should a. monitor oxygen saturation. b. check arterial blood gases (ABGs). c. monitor intracranial pressure (ICP). d. assess patient breath sounds.

Correct Answer: C Rationale: The purpose of hyperventilation for a patient with a head injury is reduction of ICP, and ICP should be monitored to evaluate whether the therapy is effective. Although oxygen saturation and ABGs are monitored in patient's receiving hyperventilation, they do not provide data about whether the therapy is successful in reducing ICP. Breath sounds are assessed, but they are not helpful in determining whether the hyperventilation is effective. Cognitive Level: Application Text Reference: p. 1475 Nursing Process: Evaluation NCLEX: Physiological Integrity

A woman who has type II diabetes is now pregnant. She wants to know whether to take her oral antidiabetic medication. What instructions will she receive? C)She will be switched to insulin therapy while she is pregnant.

Correct answer: C Rationale: Oral antidiabetic medications are generally not recommended for pregnant patients because of a lack of firm safety data. Insulin therapy is the currently recommended drug therapy for pregnant women.

You know that dietary teaching has been effective when the patient with multiple sclerosis makes which meal choice? A. Hamburger, fries, vanilla shake B. Steak, scalloped potatoes, French toast, iced tea C. Ham sandwich, potato chips, glass of whole milk D. Salad with tomatoes, chicken, bran muffin, strawberries, low- fat milk

D A nutritious, well-balanced diet is essential. Although there is no standard prescribed diet, a high-protein diet with supplementary vitamins is often recommended. A diet high in roughage may help relieve the problem of constipation. Reference: 1504

Which option is the most sensitive indication of increased ICP? A. Papilledema B. Cushing's triad C. Projectile vomiting D. Change in the level of consciousness (LOC)

D The LOC is the most sensitive and reliable indicator of the patient's neurologic status. Changes in LOC are a result of impaired cerebral brain flow. Papilledema and Cushing's triad are late signs. Projectile vomiting is not a sensitive indicator.

A patient is looking at a picture fo the brain and points to the convolutions on the surface of the cerebellum. The nurse should tell the patient these are called: a. Sulci b. Fissures c. Reticular formations d. Gyri

D. Gyri

You would correctly identify which age group as most often affected by absence seizures? A. Infants B. Children C. Young adults D. Older adults

The absence (petit mal) seizure usually occurs only in children and rarely continues beyond adolescence. This type of seizure may cease altogether as the child matures, or it may evolve into another type of seizure. Reference: 1493

When administering morning medications for a newly admitted patient, the nurse notes that the patient has an allergy to sulfa drugs. There is an order for the sulfonylurea glipizide (Glucotrol). Which action by the nurse is correct? a. Give the drug as ordered 30 minutes before breakfast. b. Hold the drug, and check the order with the prescriber. c. Give a reduced dose of the drug with breakfast. d. Give the drug, and monitor for adverse effects.

ANS: B There is a potential for cross-allergy in patients who are allergic to sulfonamide antibiotics. Although such an allergy is listed as a contraindication by the manufacturer, most clinicians do prescribe sulfonylureas for such patients. The order needs to be clarified. DIF: COGNITIVE LEVEL: Applying (Application) REF: p. 509 TOP: NURSING PROCESS: Implementation MSC: NCLEX: Physiological Integrity: Pharmacological and Parenteral Therapies

The nurse knows to administer acarbose (Precose), an alpha-glucosidase inhibitor, at which time? a. 30 minutes before breakfast b. With the first bite of each main meal c. 30 minutes after breakfast d. Once daily at bedtime

ANS: B When an alpha-glucosidase inhibitor is taken with the first bite of a meal, excessive postprandial blood glucose elevation (a glucose spike) can be reduced or prevented. DIF: COGNITIVE LEVEL: Applying (Application) REF: p. 510 TOP: NURSING PROCESS: Implementation MSC: NCLEX: Physiological Integrity: Pharmacological and Parenteral Therapies

1. A nurse recalls direct stimulation of the insulin-secreting cells of the pancreas by the autonomic nervous system is an example of _____ control. a. Negative feedback b. Positive feedback c. Neural d. Substrate-level dependent

ANS: C Direct stimulation of the insulin-secreting cells of the pancreas by the autonomic nervous system is a form of neural control. Stimulation of the insulin cells of the pancreas by the autonomic nervous system is a form of neural control and is not regulated as a form of negative feedback. Negative feedback works like a thermostat. Stimulation of the insulin cells of the pancreas by the autonomic nervous system is a form of neural control and is not regulated as a form of positive feedback. Stimulation of the insulin cells of the pancreas by the autonomic nervous system is a form of neural control and is not substrate-level dependent

3. A patient informed of a positive rapid antibody test result for human immunodeficiency virus (HIV) is anxious and does not appear to hear what the nurse is saying. What action by the nurse is most important at this time? a. Teach the patient how to reduce risky behaviors. b. Inform the patient about the available treatments. c. Remind the patient about the need to return for retesting to verify the results. d. Ask the patient to identify individuals who had intimate contact with the patient.

ANS: C After an initial positive antibody test result, the next step is retesting to confirm the results. A patient who is anxious is not likely to be able to take in new information or be willing to disclose information about the HIV status of other individuals. DIF: Cognitive Level: Analyze (analysis) REF: 222 TOP: Nursing Process: Implementation MSC: NCLEX:

11. When antibodies are formed against red blood cell antigens of the Rh system, how are the blood cells destroyed? a. Complement-mediated cell lysis b. Phagocytosis by macrophages c. Phagocytosis in the spleen d. Neutrophil granules and toxic oxygen products

ANS: C Antibodies against platelet-specific antigens or against red blood cell antigens of the Rh system coat those cells at low density, resulting in their preferential removal by phagocytosis in the spleen, rather than by complement-mediated lysis. These blood cells are not destroyed by complement-mediated cell lysis, phagocytosis by macrophages, neutrophil granules, or toxic oxygen products.

8. What characteristic do atopic individuals have that make them genetically predisposed to develop allergies? a. Greater quantities of histamine b. More histamine receptors c. Greater quantities of IgE d. A deficiency in epinephrine

ANS: C Atopic individuals tend to produce higher quantities of IgE and to have more crystalline fragment (Fc) receptors for IgE on their mast cells. Greater quantities of histamine, more histamine receptors, and a deficiency in epinephrine do not lead to a genetic predisposition to allergies.

A 12yearold male is newly diagnosed with type 1 DM. Which of the following tests should the nurse prepare the patient to best confirm the diagnosis? a. Fasting plasma glucose levels b. Random serum glucose levels c. Genetic testing d. Glycosylated hemoglobin measurements

a

When presenting information to a nursing student regarding migraine headaches, you would identify which age group as most affected by migraine headaches? A. Adolescents B. 20 to 30 year olds C. 40 to 50 year olds D. Older adults aged 60 and greater

B The most common age for onset of migraine is between the ages of 20 and 30 years. Migraine affects as many as 17% of females and 6% of males in the United States. Reference: 1487

Understanding classifications of pain helps nurses develop a plan of care. A 62-year old male has fallen while trimming tree branches sustaining tissue injury. He describes his condition as an aching, throbbing back. This is characteristic of A. Neuropathic pain B. Nociceptive pain C. Chronic pain D. Mixed pain syndrome

B. Nociceptive pain Normal function of the somatosensory system: eudynic= throbbing, aching, cramping

A nurse prepares to administer metformin (Glucophage XR) to an older adult who has asked that it be crushed because it is difficult to swallow. The nurse explains that this drug cannot be crushed because it: 1 Is released slowly. 2 Is difficult to crush. 3 Irritates mucosal tissue. 4 Has an unpleasant taste if crushed.

1. The slow-release formulary will be compromised, and the client will not receive the entire dose if it is chewed or crushed. The capsules are not difficult to crush. Irritation of the mucosal tissue is not the reason the medication should not be crushed; however, this drug should be given with meals to prevent gastrointestinal irritation. Although taste could be a factor, it is not the priority issue.

1. Emerging and reemerging infections affect health care by (select all that apply) a. reevaluating vaccine practices. b. revealing antimicrobial resistance. c. limiting antibiotics to those with life-threatening infection. d. challenging researchers to discover new antimicrobial therapies.

1. Correct answers: a, b, d An emerging infection is an infectious disease whose incidence has increased in the past 20 years or threatens to increase in the immediate future. Reemerging infections are those infections that were previously controlled but have resurfaced. The most common reason for reemerging infectious is low vaccination rates. Ways in which emerging and reemerging infectious have affected the health care system include revising vaccine recommendations for previously controlled infections (e.g., pertussis and measles); discovery of antimicrobial-resistant organisms; and creation of new antiinfective agents to combat new organisms or antimicrobial-resistant infections.

10. Which statement(s) about metabolic side effects of ART is (are) true (select all that apply)? a. These are annoying symptoms that are ultimately harmless. b. ART-related body changes include fat redistribution and peripheral wasting. c. Lipid abnormalities include increases in triglycerides and decreases in high-density cholesterol. d. Insulin resistance and hyperlipidemia can be treated with drugs to control glucose and cholesterol. e. Compared to uninfected people, insulin resistance and hyperlipidemia are more difficult to treat in HIV-infected patients.

10. Correct answers: b, c, d Rationale: Some HIV-infected patients, especially those who have been infected and have received ART for a long time, develop a set of metabolic disorders that include changes in body shape (e.g., fat deposits in the abdomen, upper back, and breasts along with fat loss in the arms, legs, and face) as a result of lipodystrophy, hyperlipidemia (i.e., elevated triglyceride levels and decreases in high-density lipoprotein levels), insulin resistance and hyperglycemia, bone disease (e.g., osteoporosis, osteopenia, avascular necrosis), lactic acidosis, and cardiovascular disease.

A 13-year-old-child with type 1 diabetes is receiving 15 units of Novolin R insulin and 20 units of Novolin N insulin at 7 am each day. At what time should the nurse anticipate a hypoglycemic reaction from the Novolin N to occur? 1. Before noon 2 In the afternoon 3 Within 30 minutes 4 During the evening

2 Novolin N is an intermediate-acting insulin that peaks approximately 6 to 8 hours after administration. It was administered at 7 am, so between 1 and 3 pm is when the nurse should anticipate that a hypoglycemic reaction will occur. During the evening or night is when a reaction from a long-acting insulin is expected. Long-acting insulin has a small peak 10 to 16 hours after administration. Noon is when a reaction from a short-acting insulin is expected. Short-acting insulin peaks in 2 to 4 hours after administration. Within 30 minutes of administration is when a reaction from a rapid-acting insulin is expected. Rapid-acting insulin peaks 30 to 60 minutes after administration.

The nurse is teaching an adolescent with type 1 diabetes about taking a combination of regular insulin (Novolin R) and an intermediate-acting insulin (Novolin N). The nurse asks the adolescent at what time of day the second dose of Novolin N should be administered. Which response by the adolescent demonstrates that the teaching has been understood? 1 At lunch 2 At dinnertime 3 1 hour after lunch 4 1 hour after dinner

2. The second dose of the intermediate-acting insulin (Novolin N) should be given at dinnertime. Novolin N insulin peaks in 4 to 12 hours. A second dose is often prescribed approximately 10 to 12 hours after the first dose. A blood glucose reading at bedtime will determine the evening dose of regular insulin (Novolin R). At lunch is too early because it may precipitate a hypoglycemic reaction. One hour after lunch is too early because it may precipitate a hypoglycemic reaction. One hour after dinner is too late.

A client arrives at the postpartum unit after delivery of her infant. On performing an assessment, the nurse notes that the client is shaking uncontrollably. Which nursing action would be appropriate? 1. Massage the fundus. 2. Contact the health care provider. 3. Cover the client with a warm blanket. 4. Place the client in Trendelenburg's position.

3

A nurse is caring for a newly admitted client with a diagnosis of Cushing syndrome. Why should the nurse monitor this client for clinical indicators of diabetes mellitus? 1.Cortical hormones stimulate rapid weight loss. 2.Tissue catabolism results in a negative nitrogen balance. 3.Glucocorticoids accelerate the process of gluco-neogenesis. 4.Excessive adrenocorticotropic hormone (ACTH) secretion damages pancreatic tissue

3.Glucocorticoids accelerate the process of gluco-neogenesis.

A nurse is caring for a client newly diagnosed with type 1 diabetes. When the health care provider tries to regulate this client's insulin regimen, the client experiences episodes of hypoglycemia and hyperglycemia, and 15 g of a simple sugar is prescribed. What is the reason this is administered when a client experiences hypoglycemia? 1.Inhibits glycogenesis 2.Stimulates release of insulin 3.Increases blood glucose levels 4.Provides more storage of glucose

3.Increases blood glucose levels

A client arrives at the clinic complaining of fatigue, lack of energy, constipation, and depression. Following diagnostic studies, hypothyroidism is diagnosed and levothyroxine (Synthroid) is prescribed. The nurse informs the client that which is the expected outcome of the medication? 1. Alleviate depression 2. Increase energy levels 3. Increase blood glucose levels 4. Achieve normal thyroid hormone levels

4

A client complains of being cold, and the nurse notes the presence of "goose flesh" on the client's arms. The nurse plans care, knowing that which structure is responsible for this response? 1. Arterioles 2. Sweat glands 3. Collagen fibers 4. Arrector pili muscles

4

A client has begun medication therapy with propylthiouracil (PTU). The nurse should assess the client for which condition as an adverse effect of this medication? 1. Joint pain 2. Renal toxicity 3. Hyperglycemia 4. Hypothyroidism

4

A client with type 2 diabetes is taking one glyburide (Micronase) tablet daily. The client asks whether an extra pill should be taken before exercise. What is the nurse's best reply? 1 "You will need to decrease how much you are exercising." 2 "An extra pill will help your body use glucose when exercising." 3 "The amount of medication you need to take is not related to exercising." 4 "Do not take an extra pill because you may become hypoglycemic when exercising."

4. Exercise improves glucose metabolism; exercise is associated with a risk for hypoglycemia, not hyperglycemia; an additional antidiabetic agent is contraindicated. Exercise should not be decreased because it improves glucose metabolism. Also, this response does not answer the client's question. An extra tablet probably will result in hypoglycemia because exercise alone improves glucose metabolism. Control of glucose metabolism is achieved through balanced diet, exercise, and pharmacologic therapy.

A nurse is teaching a 10-year-old child with type 1 diabetes about insulin requirements. When should the nurse explain that insulin needs will decrease? 1 When puberty is reached 2 When infection is present 3 When emotional stress occurs 4 When active exercise is performed

4. Exercise reduces the body's need for insulin. Increased muscle activity accelerates transport of glucose into muscle cells, thus producing an insulin-like effect. With increased growth and associated dietary intake, the need for insulin increases during puberty. An infectious process may require increased insulin. Emotional stress increases the need for insulin.

A client with type 2 diabetes, who is taking an oral hypoglycemic agent, is to have a serum glucose test early in the morning. The client asks the nurse, "What do I have to do to prepare for this test?" Which statement by the nurse reflects accurate information? 1."Eat your usual breakfast." 2."Have clear liquids for breakfast." 3."Take your medication before the test." 4."Do not ingest anything before the test."

4."Do not ingest anything before the test."

Which statement made by a 28-year-old client recently diagnosed with type 1 diabetes indicates that further education is necessary regarding the teaching plan? 1."I will need to have my eyes and vision examined once a year." 2."I will need to check my blood sugar at home to evaluate my response to my treatment plan." 3."I can improve metabolic and cardiac risk factors of this disease if I follow a low-calorie diet and lose weight." 4."Once I reach my target weight there is a good chance that I will be able to switch from insulin to an oral medication to control my blood sugar."

4."Once I reach my target weight there is a good chance that I will be able to switch from insulin to an oral medication to control my blood sugar."

A female client receiving cortisone therapy for adrenal insufficiency expresses concern about why she is developing facial hair. How should the nurse respond? 1."It is just another sign of the illness." 2."Do not worry because it will disappear with therapy." 3."This is not important as long as you are feeling better," 4."The drug contains a hormone that causes male characteristics."

4."The drug contains a hormone that causes male characteristics."

A client who was diagnosed recently with type 1 diabetes states, "I feel bad. I don't think I even want to go home. My spouse doesn't care about my diabetes." What is the most appropriate nursing response? 1."What can I do to make you feel better?" 2."It seems that you don't get along with your spouse." 3."It's probably temporary. Your spouse needs more time to adjust." 4."You are unhappy. Have you tried to talk with your spouse?"

4."You are unhappy. Have you tried to talk with your spouse?"

2. When teaching a patient infected with HIV regarding transmission of the virus to others, which statement made by the patient would indicate a need for further teaching? A. "I will need to isolate any tissues I use so as not to infect my family." B. "I will notify all of my sexual partners so they can get tested for HIV." C. "Unprotected sexual contact is the most common mode of transmission." D. "I do not need to worry about spreading this virus to others by sweating at the gym."

A HIV is not spread casually. The virus cannot be transmitted through hugging, dry kissing, shaking hands, sharing eating utensils, using toilet seats, or attending school with an HIV-infected person. It is not transmitted through tears, saliva, urine, emesis, sputum, feces, or sweat.

17. A patient has acquired immunodeficiency syndrome (AIDS) and the viral load is reported as undetectable. What patient teaching should be provided by the nurse related to this laboratory study result? A. The patient has the virus present and can transmit the infection to others. B. The patient is not able to transmit the virus to others through sexual contact. C. The patient will be prescribed lower doses of antiretroviral medications for 2 months. D. The syndrome has been cured, and the patient will be able to discontinue all medications.

A In human immunodeficiency virus (HIV) infections, viral loads are reported as real numbers of copies/μL or as undetectable. "Undetectable" indicates that the viral load is lower than the test is able to report. "Undetectable" does not mean that the virus has been eliminated from the body or that the individual can no longer transmit HIV to others.

A 50-year-old man complains of recurring headaches. He describes these as sharp, stabbing, and located around his left eye. He also reports that his left eye seems to swell and get teary when these headaches occur. Based on this history, you suspect that he has A. cluster headaches. B. tension headaches. C. migraine headaches. D. medication overuse headaches.

A Cluster headaches involve repeated headaches that can occur for weeks to months at a time, followed by periods of remission. The pain of cluster headache is sharp and stabbing; the intense pain lasts from a few minutes to 3 hours. Headaches can occur every other day and as often as 8 times a day. The clusters occur with regularity, usually occurring at the same time each day, during the same seasons of the year. Typically a cluster lasts 2 weeks to 3 months, and then the patient goes into remission for months to years. The pain is generally located around the eye, radiating to the temple, forehead, cheek, nose, or gums. Other manifestations may include swelling around the eye, lacrimation (tearing), facial flushing or pallor, nasal congestion, and constriction of the pupil. During the headache, the patient is often agitated and restless, unable to sit still or relax. Reference: 1487-1488

An important nursing diagnosis for the patient with amyotrophic lateral (ALS) is A. impaired gas exchange related to paralysis of respiratory muscles. B. hypothermia related to impaired regulation by the hypothalamus. C. impaired memory related to cognitive changes. D. dysreflexia related to loss of sympathetic nervous tone

A Death usually results from respiratory infection secondary to compromised respiratory function. Reference: 1515

When is dementia usually diagnosed? A. Two brain functions are impaired. B. Memory is affected. C. Positive result is obtained on a computed tomography (CT) study. D. The patient fails the Benton Visual Form Discrimination Test (BVFD).

A Dementia is usually diagnosed when two or more brain functions, such as memory loss or language skills, are significantly impaired. The Mini-Mental State Examination (MMSE) is used to assess cognitive effect. Although tests help to make the diagnosis, no single clinical test can be used to diagnose dementia, and it is primarily a diagnosis of exclusion.

The patient reports falling when he his foot got "stuck" on a crack in the sidewalk, hitting his head when he fell, and "passing out". The paramedics found the patient walking at the scene and talking before transporting the patient to the hospital. In the emergency department, the patient starts to lose consciousness. This is a classic scenario for which complication? A. Epidural hematoma B. Subdural hematoma C. Subarachnoid bleed D. Diffuse axial inju

A Epidural hematoma often results from a linear fracture crossing a major artery in the dura. The classic sign is an initial period of unconsciousness at the scene and a brief lucid interval followed by a decrease in LOC. A subdural hematoma often results from injury to the brain and veins and develops more slowly. The classic sign or symptom of subarachnoid hemorrhage is a patient describing "the worst headache of my life." Diffuse axonal injury is widespread axonal damage occurring after a traumatic brain injury.

Which nursing action should be implemented in the care of a patient who is experiencing increased ICP? A. Monitor fluid and electrolyte status astutely. B. Position the patient in a high-Fowler's position. C. Administer vasoconstrictors to maintain cerebral perfusion. D. Maintain physical restraints to prevent episodes of agitation.

A Fluid and electrolyte disturbances can have an adverse effect on ICP and must be vigilantly monitored. The head of the patient's bed should be kept at 30 degrees in most circumstances, and physical restraints are not applied unless absolutely necessary. Vasoconstrictors are not typically administered in the treatment of ICP.

Which modifiable risk factors for stroke are most important for you to include when planning a community education program? A. Hypertension B. Hyperlipidemia C. Alcohol consumption D. Oral contraceptive use

A Hypertension is the single most important modifiable risk factor, but it is still often undetected and inadequately treated.

Which measure should you prioritize when providing care for a patient with a diagnosis of multiple sclerosis (MS)? A. Vigilant infection control and adherence to standard precautions B. Careful monitoring of neurologic vital signs and frequent reorientation C. Maintenance of a calorie count and hourly assessment of intake and output D. Assessment of blood pressure and monitoring for signs of orthostatic hypotension

A Infection control is a priority in the care of patients with MS since infection is the most common precipitator of an exacerbation of the disease. Decreases in cognitive function are less likely and MS does not typically result in hypotension or fluid volume excess or deficit. Reference: 1504

The pathophysiology of multiple sclerosis (MS) is related to an attack on the A. myelin sheaths on various neurons. B. goblet cells of the muscles. C. lining of the blood vessels. D. lack of acetylcholine at the synaptic junction.

A Initially the myelin sheaths of the neurons in the brain and spinal cord are attacked. Early in the disease the myelin sheath is damaged. Reference: 1501

Computed tomography of a 68-year-old patient's head reveals that he has experienced a hemorrhagic stroke. Which option is a nursing priority intervention in the emergency department? A. Maintenance of the patient's airway B. Positioning to promote cerebral perfusion C. Control of fluid and electrolyte imbalances D. Administration of tissue plasminogen activator (tPA)

A Maintenance of a patent airway is the priority in the acute care of a patient with a hemorrhagic stroke, and it supersedes the importance of fluid and electrolyte imbalance and positioning. Use of tPA is contraindicated in hemorrhagic stroke.

Why are the data regarding mobility, strength, coordination, and activity tolerance important for you to obtain? A. Many neurologic diseases affect one or more of these areas. B. Patients are less able to identify other neurologic impairments. C. These are the first functions to be affected by neurologic disease. D. Aspects of movement are the most important function of the nervous system.

A Many neurologic disorders can cause problems in the patient's mobility, strength, and coordination. These problems can result in changes in the patient's usual activity and exercise patterns.

8. For mild-to-moderate migraines you would anticipate the use of which medication? A. Aspirin or acetaminophen B. Ibuprofen C. Sumatriptan (Imitrex) D. Codeine

A Many people with mild or moderate migraine can obtain relief with aspirin or acetaminophen. Reference: 1488

A patient's eyes jerk while the patient looks to the left. How do you record this finding? A. Nystagmus B. Cranial nerve VI palsy C. Oculocephalia D. Ophthalmic dyskinesia

A Nystagmus is fine, rapid jerking movements of the eyes.

For which patient should you prioritize an assessment for depression? A. A patient in the early stages of AD B. A patient who is in the final stages of AD C. A patient experiencing delirium resulting from dehydration D. A patient who has become delirious after an atypical drug response

A Patients in the early stages of AD are particularly susceptible to depression, because the patient is aware of his or her cognitive changes and the expected disease trajectory. Delirium is typically a short-term health problem that does not typically pose a heightened risk of depression. Reference: 1527

The priority nursing intervention for the patient with myasthenia gravis who is receiving pyridostigmine (Mestinon) includes A. timing drug administration so that chewing and swallowing are enhanced. B. withholding the drug if muscle function improves. C. assessing for constipation and paralytic ileus. D. monitoring renal and hepatic function.

A Scheduling doses of drugs so that peak action is reached at mealtime may make eating less difficult. Reference: 1513

You would correctly identify the advantage of Sinemet in the treatment of Parkinson's disease is its ability to A. restore deficient dopamine. B. increase the effect of levodopa. C. block the breakdown of dopamine. D. block uptake of catecholamines.

A Sinemet is the preferred drug because it also contains carbidopa, an agent that inhibits the enzyme dopa-decarboxylase in the peripheral tissues. Dopa-decarboxylase breaks down levodopa before it reaches the brain. The net result of the combination of levodopa and carbidopa is that more levodopa reaches the brain, and therefore less drug is needed. Reference: 1508

ou recognize that status epilepticus is a medical emergency because A. seizures continue without a return of consciousness. B. fractures of a limb may occur. C. urinary fecal incontinence may occur. D. heart rate becomes bradycardic.

A Status epilepticus is a state of continuous seizure activity or a condition in which seizures recur in rapid succession without return to consciousness between seizures. It is the most serious complication of epilepsy and is a neurologic emergency. Status epilepticus can occur due to any type of seizure. During repeated seizures the brain uses more energy than can be supplied.

Magnetic resonance imaging (MRI) has revealed a brain tumor in a patient. You recognize the patient's likely need for which treatment modality? A. Surgery B. Chemotherapy C. Radiation therapy D. Pharmacologic treatment

A Surgical removal is the preferred treatment for brain tumors.

The patient is diagnosed with a brain tumor. Which option is the correct understanding of the preferred treatment? A. Surgical removal is preferred, even if the tumor is not malignant. B. Chemotherapy is a common and effective treatment. C. Stereotactic radiosurgery is the preferred treatment. D. A large dose of intravenous steroid therapy is preferred.

A Surgical removal is the preferred treatment. It can reduce tumor mass (decreasing intracranial pressure [ICP]), provides relief of symptoms, and extend survival time. Even a benign mass has a malignant effect by taking up space. Traditional chemotherapy effectiveness is limited because of the blood-brain barrier, tumor cell heterogeneity, and tumor cell drug resistance. Stereotactic radiosurgery delivers a high, concentrated dose of radiation precisely directed and is used when conventional surgery has failed or is not an option. Corticosteroids are not an integral part of therapy, but are used to control complications of radiation therapy.

When administering a Tensilon test to a patient with a possible diagnosis of myasthenia gravis, you would realize the test is positive if the patient A. reports improved muscle strength. B. demonstrates improved cognition. C. experiences a surge of energy. D. exhibits enhanced vision.

A Tensilon test in a patient with MG reveals improved muscle contractility after intravenous injection of the anticholinesterase agent edrophonium chloride (Tensilon). (Anticholinesterase blocks the enzyme acetylcholinesterase.) Reference: 1512

18. What should the nurse teach the patients in the assisted living facility to decrease their risk for antibiotic-resistant infection (select all that apply.)? A. Wash hands frequently. B. Take antibiotics as prescribed. C. Take the antibiotic until it is gone. D. Take antibiotics to prevent illnesses like colds. E. Save leftover antibiotics to take if needed later.

A, B, C To decrease the risk for antibiotic-resistant infections, people should wash their hands frequently, follow the directions when taking the antibiotics, finish the antibiotic, do not request antibiotics for colds or flu, do not save leftover antibiotics, or take antibiotics to prevent an illness without them being prescribed by a health care provider.

Results of stimulation of the parasympathetic nervous system are (select all that apply) A. constriction of the bronchi. B. dilation of skin blood vessels. C. increased secretion of insulin. D. increased blood glucose levels. E. relaxation of the urinary sphincters

A, B, C, D Parasympathetic nervous system stimulation results in constriction of the bronchi, dilation of blood vessels to the skin, increased secretion of insulin, and relaxation of the urinary sphincter. Sympathetic nervous system stimulation results in increased blood glucose levels.

A patient is in the HIV clinic for a follow-up appointment. He has been on antiretroviral therapy for HIV for more than 3 years. The nurse will assess for which potential adverse effects of longterm antiretroviral therapy? (Select all that apply.) a. Lipodystrophy b. Liver damage c. Kaposi's sarcoma d. Osteoporosis e. Type 2 diabetes

A, B, D, E Anti-HIV drugs produce strain on the liver and may result in liver disease. A major adverse effect of protease inhibitors is lipid abnormalities, including lipodystrophy, or redistribution of fat stores under the skin. In addition, dyslipidemias such as hypertriglyceridemia can occur, and insulin resistance and type 2 diabetes symptoms can result. The increase in long-term antiretroviral drug therapy due to prolonged disease survival has led to the emergence of another long-term adverse effect associated with these medications—bone demineralization and possible osteoporosis. Kaposi's sarcoma is an opportunistic disease associated with HIV, not a result of long-term drug therapy.

Before administration of any antiviral medication, what nursing care would be performed? (Select all that apply.) A. Head-to-toe physical assessment B. Documentation of known allergies C. History of medication use D. Monitoring for adverse effects E. Baseline vital signs

A, B,C,E Before administering an antiviral drug, perform a thorough head-to-toe physical assessment and take a medical and medication history. Document any known allergies before use of these and any other medications. Also assess the patient’s baseline vital signs because of the profound effects of viral illnesses on physiologic status, especially if the patient is immunocompromised. Assess and document any contraindications, cautions, and drug interactions associated with all of the antiviral drugs. Monitoring for adverse effects would occur after the medication has been administered

A young woman has been diagnosed with relapsing-remitting multiple sclerosis and interferon β-1b (Betaseron) has been prescribed. You determine that teaching about the drug has been successful when the patient says (select all that apply) A. "I should avoid direct sunlight and use sunscreen and protective clothing when out of doors." B. "I will need to rotate injection sites with each dose I inject." C. "I should report any depression or suicidal thoughts that develop." D. "Because this drug is a corticosteroid, I should reduce my sodium intake to prevent edema." E. "Flulike symptoms are indicative of a significant side effect and should be reported immediately."

A,B,C Interferon is an immunomodulator, not a corticosteroid. The patient should rotate injection sites with each dose and the patient should watch for depression and suicidal ideation. The patient should wear sunscreen and protective clothing while exposed to sun. The patient should also be aware that flu-like symptoms are common following initiation of therapy. Reference: 1502-1503

What nursing actions can help to minimize the risk of delirium in an older, hospitalized patient (select all that apply)? A. Identification of high risk patients B. Providing patient's glasses and hearing aid C. Consistent nursing staff near the nursing station D. Having a clock and calendar available E. Administering anticholinergic drugs prophylactic

A,B,C,D Identify high-risk patients, including those with neurologic disorders, sensory impairment, or advanced age. Other risk factors include hospitalization in an intensive care unit, lack of a watch or calendar, absence of reading glasses, and untreated pain. Precipitating factors are eliminated. Assess for drug and alcohol withdrawal, fluid and electrolyte imbalance, nutritional deficiencies, and infection. Care includes protecting from harm, increasing familiarity with the environment, and reorientation and behavioral interventions. Polypharmacy is a common cause; drugs are not used prophylactically for this problem.

Which types of antiviral drugs are used to treat HIV infection? (Select all that apply.) A) Nonnucleoside reverse transcriptase inhibitors B) Protease inhibitors C) Reverse transcriptase inhibitors D) Fusion inhibitors E) Neuraminidase inhibitors

A,B,C,D. All the options represent antiretroviral drugs. Combination drug therapy is used to prevent the development of resistant strains of HIV. Neuraminidase inhibitors are used in the treatment of the influenza virus.

You are caring for a patient admitted for evaluation and surgical removal of a brain tumor. You plan interventions for this patient based on knowledge that brain tumors can lead to which complications (select all that apply)? A. Vision loss B. Cerebral edema C. Pituitary dysfunction D. Parathyroid dysfunction E. Focal neurologic deficits

A,B,C,E Depending on the location, brain tumors can have a wide variety of clinical manifestations, including vision loss and focal neurologic deficits. Tumors that put pressure on the pituitary can lead to dysfunction of the gland. As the tumor grows, clinical manifestations of increased ICP and cerebral edema can appear. Parathyroid glands are not regulated by the cerebral cortex or the pituitary.

A nurse is caring for a client with a neurological disorder is planning care to maintain nutritional status. The nurse is concerned about the client's swallowing ability. Which of the following food items would the nurse plan to avoid in this client's diet? A. Spinach B. Custard C. Scrambled eggs D. Mashed potatoes

A. Flavorful, warm, or well-chilled foods with texture stimulate the swallowing reflex. Soft and semisoft foods, such as custards or puddings, egg dishes, and potatoes, are usually effective Raw vegetables, chunky vegetables, such as diced beets, and stringy vegetables, such as spinach, corn, and peas are foods commonly excluded from the diet of a client with a poor swallowing reflex.

The nurse is caring for a client who is in the chronic phase of brain attack (stroke) and has a right-sided hemiparesis. The nurse identifies the nursing diagnosis of Imbalanced Nutrition: less than body requirements, related to inability to feed self . Which of the following is a priority nursing intervention to help improve the client's nutrition? A. Assist the client to eat with the left hand to build strength. B. Provide a pureed diet that is easy for the client to swallow. C. Inform the client that a feeding tube will be placed if progress is not made D. Provide a variety of foods on the meal tray to stimulate the client's appetite.

A. The nurse should teach the client to use both sides of the body to increase strength and build endurance. Option B is incorrect; the question does not mention swallowing difficulty, so there is no need to puree the food. Option C is incorrect; that information would come from the physician. Option D is incorrect; the problem is not the food selection but the client's ability to eat the food independently.

A patient with type 2 diabetes has a new prescription for repaglinide (Prandin). After 1 week, she calls the office to ask what to do, because she keeps missing meals. "I work right through lunch sometimes, and I'm not sure whether I need to take it. What do I need to do?" What is the nurse's best response? A. "You need to try not to skip meals, but if that happens, you will need to skip that dose of Prandin." B. "We will probably need to change your prescription to insulin injections because you can't eat meals on a regular basis." C. "Go ahead and take the pill when you first remember that you missed it." D. "Take both pills with the next meal, and try to eat a little extra to make up for what you missed at lunchtime."

A. "You need to try not to skip meals, but if that happens, you will need to skip that dose of Prandin."

Which intravenous fluid prescription noted by the nurse in orders for a client being treated for increased intracranial pressure​ (IICP) should the nurse contact the healthcare provider about for​ clarification? A. 0.5% normal saline B. ​0.9% saline C. Lactated Ringer D. ​3% saline

A. 0.5% normal saline

A patient receives aspart (NovoLog) insulin at 8:00 AM. At which time would the nurse anticipate the highest risk for hypoglycemia? a. 10:00 AM b. 12:00 AM c. 2:00 PM d. 4:00 PM

A. 10:00 AM

It is most important for the nurse to include which risk factors in a teaching plan associated with the development of type 2 diabetes mellitus? (Select all that apply.) A. Hypertension B. History of pancreatic trauma C. Weight gain of 30 pounds during pregnancy D. Body mass index greater than 25 kg/m E. Triglyceride levels between 150 and 200 mg/dL F. Delivery of a 4.99-kg baby

A. Hypertension D. Body mass index greater than 25 kg/m F. Delivery of a 4.99-kg baby

The nurse is planning to assess a​ 4-year-old child to help determine the cause of the​ child's fever. Which body system is a priority to​ assess? (Select all that​ apply.) A. Urinary B. Musculoskeletal C. Respiratory D. Gastrointestinal E. Neurologic

A. Urinary C. Respiratory D. Gastrointestinal ​Rationale: Infections of the​ urinary, respiratory, and gastrointestinal systems are the most common reason for a fever in this age range. The neurologic and musculoskeletal systems are not common systems for infections in children.

Which patient action indicates an accurate understanding of the nurse's teaching about the use of an insulin pump? a. The patient programs the pump for an insulin bolus after eating. b. The patient changes the location of the insertion site every week. c. The patient takes the pump off at bedtime and starts it again each morning. d. The patient plans a diet with more calories than usual when using the pump.

A. The patient programs the pump for an insulin bolus after eating.

A female patient is scheduled for an oral glucose tolerance test. Which information from the patient's health history is important for the nurse to communicate to the health care provider regarding this test? a. The patient uses oral contraceptives. b. The patient runs several days a week. c. The patient has been pregnant three times. d. The patient has a family history of diabetes.

A. The patient uses oral contraceptives.

The nurse is assessing a 55-yr-old female patient with type 2 diabetes who has a body mass index (BMI) of 31 kg/m2 .Which goal in the plan of care is most important for this patient? a. The patient will reach a glycosylated hemoglobin level of less than 7%. b. The patient will follow a diet and exercise plan that results in weight loss. c. The patient will choose a diet that distributes calories throughout the day. d. The patient will state the reasons for eliminating simple sugars in the diet.

A. The patient will reach a glycosylated hemoglobin level of less than 7%.

1. The nurse is advising a clinic patient who was exposed a week ago to human immunodeficiency virus (HIV) through unprotected sexual intercourse. The patient's antigen and antibody test has just been reported as negative for HIV. What instructions should the nurse give to this patient? a. "You will need to be retested in 2 weeks." b. "You do not need to fear infecting others." c. "Since you don't have symptoms and you have had a negative test, you do not have HIV)." d. "We won't know for years if you will develop acquired immunodeficiency syndrome (AIDS)."

ANS: A HIV screening tests detect HIV-specific antibodies or antigens, but typically it takes a several week delay after initial infection before HIV can be detected on a screening test. Combination antibody and antigen tests (also known as fourth-generation tests) decrease the window period to within 3 weeks after infection. It is not known based on this information whether the patient is infected with HIV or can infect others. DIF: Cognitive Level: Apply (application) REF: 221 TOP: Nursing Process: Implementation MSC:

7. A patient is having an IgE-mediated hypersensitivity reaction. What action by the healthcare professional is best? a. Give the patient an antihistamine. b. Prepare to give the patient a blood transfusion. c. Ask the patient is he/she is having pain at the site. d. Apply warm, moist heat to the affected area.

ANS: A Histamine is the most potent mediator in an IgE-mediated hypersensitivity reaction (Type 1). Histamine bound to H2 results in the degranulation of mast cells with the release of histamine. Blocking histamine receptors with antihistamines can control some type I responses. The healthcare professional would not need to give the patient blood; warm; moist heat; or ask about pain.

A nurse is conducting community education classes on skin cancer. One participant says to the nurse: "I read that most melanomas occur on the face and arms in fair-skinned women. Is this true?" The nurse's most helpful response would be which of the following? a. "That is not correct. Melanoma is more commonly found on the torso or the lower legs of women." b. "That is correct, because the face and arms are exposed more often to the sun." c. "That is not correct. Melanoma occurs on the top of the head in men but is rare in women." d. "That is incorrect. Melanoma is most commonly seen in dark-skinned individuals."

ANS: A Melanoma is more commonly found on the torso or the lower legs in women. Melanoma can occur anywhere and is not associated with direct exposure. For example, an individual can have melanoma under the skin and on the soles of the feet. Dark-skinned individuals are less likely to get melanoma. REF: Page 264

19. Eight years after seroconversion, a human immunodeficiency virus (HIV)-infected patient has a CD4+ cell count of 800/µL and an undetectable viral load. What is the priority nursing intervention at this time? a. Encourage adequate nutrition, exercise, and sleep. b. Teach about the side effects of antiretroviral agents. c. Explain opportunistic infections and antibiotic prophylaxis. d. Monitor symptoms of acquired immunodeficiency syndrome (AIDS).

ANS: A The CD4+ level for this patient is in the normal range, indicating that the patient is the stage of asymptomatic chronic infection when the body is able to produce enough CD4+ cells to maintain a normal CD4+ count. Maintaining healthy lifestyle behaviors is an important goal in this stage. AIDS and increased incidence of opportunistic infections typically develop when the CD4+ count is much lower than normal. Although the initiation of ART is highly individual, it would not be likely that a patient with a normal CD4+ level would receive ART. DIF: Cognitive Level: Apply (application) REF: 220 OBJ: Special Questions: Prioritization TOP: Nursing Process: Implementation MSC:

14. Graves disease (hyperthyroidism) is an example of which type II hypersensitivity reaction? a. Modulation b. Antibody-dependent cell-mediated cytotoxicity c. Neutrophil-mediated damage d. Complement-mediated lysis

ANS: A The antibody reacts with the receptors on the target cell surface and modulates the function of the receptor by preventing interactions with their normal ligands, replacing the ligand and inappropriately stimulating the receptor or destroying the receptor. For example, in the hyperthyroidism (excessive thyroid activity) of Graves disease, autoantibody binds to and activates receptors for thyroid-stimulating hormone (TSH) (a pituitary hormone that controls the production of the hormone thyroxine by the thyroid). Graves disease is not a result of cell-mediated cytotoxicity, neutrophil-mediated damage, or complement-mediated lysis.

21. An older adult with chronic human immunodeficiency virus (HIV) infection who takes medications for coronary artery disease and hypertension has chosen to begin early antiretroviral therapy (ART). Which information will the nurse include in patient teaching? a. Many drugs interact with antiretroviral medications. b. HIV infections progress more rapidly in older adults. c. Less frequent CD4+ level monitoring is needed in older adults. d. Hospice care is available for patients with terminal HIV infection.

ANS: A The nurse will teach the patient about potential interactions between antiretrovirals and the medications that the patient is using for chronic health problems. Treatment and monitoring of HIV infection is not affected by age. A patient beginning early ART is not a candidate for hospice. Progression of HIV is not affected by age although it may be affected by chronic disease. DIF: Cognitive Level: Apply (application) REF: 228 TOP: Nursing Process: Implementation MSC:

3. The nurse plans a presentation for community members about how to decrease the risk for antibiotic-resistant infections. Which information will the nurse include in the teaching plan (select all that apply)? a. Antibiotics may sometimes be prescribed to prevent infection. b. Continue taking antibiotics until all of the prescription is gone. c. Unused antibiotics that are more than a year old should be discarded. d. Antibiotics are effective in treating influenza associated with high fevers. e. Hand washing is effective in preventing many viral and bacterial infections.

ANS: A, B, E All prescribed doses of antibiotics should be taken. In some situations, such as before surgery, antibiotics are prescribed to prevent infection. There should not be any leftover antibiotics because all prescribed doses should be taken. However, if there are leftover antibiotics, they should be discarded immediately because the number left will not be enough to treat a future infection. Hand washing is generally considered the single most effective action in decreasing infection transmission. Antibiotics are ineffective in treating viral infections such as influenza. DIF: Cognitive Level: Apply (application) REF: 216 TOP: Nursing Process: Implementation MSC:

The nurse assesses the patient and notes all of the following. Select all of the findings that indicate the systemic manifestations of inflammation. a. Oral temperature 38.6° C/101.5° F b. Thick, green nasal discharge c. Patient complaint of pain at 6 on a 0 to 10 scale on palpation of frontal and maxillary sinuses d. WBC 20 cells/McL 109/L e. Patient reports, "I'm tired all the time. I haven't felt like myself in days."

ANS: A, D, E Systemic manifestations of inflammatory response include elevated temperature, leukocytosis, and malaise and fatigue. Purulent exudates and pain are both considered local manifestations of inflammation.

22. The registered nurse (RN) caring for an HIV-positive patient admitted with tuberculosis can delegate which action to unlicensed assistive personnel (UAP)? a. Teach the patient how to dispose of tissues with respiratory secretions. b. Stock the patient's room with the necessary personal protective equipment. c. Interview the patient to obtain the names of family members and close contacts. d. Tell the patient's family members the reason for the use of airborne precautions.

ANS: B A patient diagnosed with tuberculosis would be placed on airborne precautions. Because all health care workers are taught about the various types of infection precautions used in the hospital, the UAP can safely stock the room with personal protective equipment. Obtaining contact information and patient teaching are higher-level skills that require RN education and scope of practice. DIF: Cognitive Level: Apply (application) REF: 218 OBJ: Special Questions: Delegation TOP: Nursing Process: Implementation MSC: NCLEX:

A patient is being treated with an antibiotic for an infected orthopedic injury. What explanation should the nurse give to the patient about this medication? a. "Antibiotics will decrease the pain at the site." b. "An antibiotic helps to kill the infection causing the inflammation." c. "An antibiotic inhibits cyclooxygenase, an enzyme in the body." d. "Antibiotics will reduce the patient's fever."

ANS: B Antimicrobials treat the underlying cause of the infection which leads to inflammation. Analgesics and nonsteroidal antiinflammatory drugs (NSAIDs) help to treat pain. NSAIDs and other antipyretics are cyclooxygenase inhibitors. Antipyretics help to reduce fever.

33. Which statement is true concerning an atopic individual? a. They tend to produce less IgE. b. They tend to produce more Fc receptors. c. They tend to attract very few mast cells. d. They tend to produce very high levels of IgM.

ANS: B Atopic individuals tend to produce higher quantities of IgE and have more Fc receptors for IgE on their mast cells. Atopic individuals tend to produce more, not less, IgE. Manifestations these individuals display are influenced greatly by the release of histamine from mast cells, which are found in large numbers in the affected tissue. An IgM response is a mainstay of type II responses.

A patient comes to the clinic with a complaint of painful, itchy feet. On interview, the patient tells the nurse that he is a college student living in a dormitory apartment that he shares with five other students. What teaching should the nurse provide for this patient? a. "Don't eat with the other students." b. "Avoid sharing razors and other personal items." c. "Have a complete blood count (CBC) checked monthly." d. "Disinfect showers and bathroom floors weekly after use."

ANS: B Avoidance of sharing personal items like razors and hairbrushes can decrease the spread of pathogens that cause inflammation and infection. Not eating with the others in his college apartment won't relieve or prevent the spread of infection. A CBC monthly will not treat or prevent inflammation. Showers should be disinfected before and after each use.

2. What is a hypersensitivity reaction that produces an allergic response called? a. Hemolytic shock b. Anaphylaxis c. Necrotizing vasculitis d. Systemic erythematosus

ANS: B Examples of systemic anaphylaxis are allergic reactions to beestings, peanuts, and fish. Hemolytic shock would be a state in which erythrocytes are destroyed by complement-mediated lysis to the point of causing a state of shock. Necrotizing vasculitis is inflammation of blood vessel walls that limits perfusion. Systemic lupus erythematosus (SLE) is a chronic, multisystem, inflammatory disease and is one of the most common, complex, and serious of the autoimmune disorders.

A 5-year-old male becomes ill with a severe cough. Histologic examination reveals a bacterial infection, and further laboratory testing reveals cell membrane damage and decreased protein synthesis. Which of the following is the most likely cause of this illness? a. Endotoxin b. Exotoxin c. Hemolysis d. Septicemia

ANS: B Exotoxins are enzymes that can damage the plasma membranes of host cells or can inactivate enzymes critical to protein synthesis, and endotoxins activate the inflammatory response and produce fever. Endotoxins released by blood-borne bacteria cause the release of vasoactive enzymes that increase the permeability of blood vessels. Hemolysis is the breakdown of red cells. Septicemia is the growth of bacteria in the blood.

37. A pregnant woman has Graves disease. What test/s does the healthcare professional advise the woman about? a. Frequent tests of the newborn's muscular strength and movement. b. Blood test for hyperthyroidism c. Monthly OB checkups for fetal anomalies or pregnancy loss d. Serum complete blood count including platelet count

ANS: B Graves disease is an autoimmune disease in which maternal antibody against the receptor for TSH causes neonatal hyperthyroidism. The healthcare professional should advise the woman that her newborn will need to be tested for hyperthyroidism.

A patient is diagnosed with a sprain to her right ankle after a fall. The patient asks the nurse about using ice on her injured ankle. What is the nurse's best response? a. "Use ice only when the ankle hurts." b. "Ice should be applied for 15 to 20 minutes every 2 to 3 hours over the next 1 to 2 days." c. "Wrap an ice pack around the injured ankle for the next 24 to 48 hours." d. "Ice is not recommended for use on the sprain because it would inhibit the inflammatory response."

ANS: B Ice is used on areas of injury during the first 24 to 48 hours after the injury occurs to prevent damage to surrounding tissues from excessive inflammation. Ice should be used for a maximum of 20 minutes at a time every 2 to 3 hours. Ice must be used according to a schedule for it to be effective and not be overused. Using ice more often or for longer periods of time can cause additional tissue damage. Ice is recommended to inhibit the inflammatory process from damaging surrounding tissue.

29. Considering the effects of nutritional deficiencies on the immune system, severe deficits in calories and protein lead to deficiencies in the formation of which immune cells? a. B cells b. T cells c. Natural killer cells d. Neutrophils

ANS: B Severe deficits in calorie or protein intake lead to deficiencies in T-cell function and numbers. B cells, natural killer cells, and neutrophils are not necessarily affected by nutritional deficits.

A nurse is educating a 21-year-old lifeguard about the risk of skin cancer and the need to wear sunscreen. Which statement by the patient indicates that the need for further teaching? a. "I wear a hat and sit under the umbrella when not in the water." b. "I don't bother with sunscreen on overcast days." c."I use a sunscreen with the highest SPF number." d. "I wear a UV shirt and limit exposure to the sun by covering up."

ANS: B The sun's rays are as damaging to skin on cloudy, hazy days as on sunny days. The other options will all prevent skin cancer. REF: Page 266

A patient has been diagnosed with metabolic syndrome and is started on the biguanide metformin (Glucophage). The nurse knows that the purpose of the metformin, in this situation, is which of these? a. To increase the pancreatic secretion of insulin b. To decrease insulin resistance c. To increase blood glucose levels d. To decrease the pancreatic secretion of insulin

ANS: B Metformin decreases glucose production by the liver; decreases intestinal absorption of glucose; and improves insulin receptor sensitivity in the liver, skeletal muscle, and adipose tissue, resulting in decreased insulin resistance. The other options are incorrect. DIF: COGNITIVE LEVEL: Understanding (Comprehension) REF: p. 508 TOP: NURSING PROCESS: Planning MSC: NCLEX: Physiological Integrity: Pharmacological and Parenteral Therapies

After studying about fungi, which information indicates a correct understanding of fungi? Fungi causing deep or systemic infections: a. are easily treated with penicillin. b. are extremely rare. c. never occur with other infections. d. are commonly opportunistic.

ANS: D Diseases caused by fungi are called mycoses. Mycoses are common and can be opportunistic and occur with other infections but are not treatable with penicillin.

6. During an IgE-mediated hypersensitivity reaction, what causes bronchospasm? a. Bronchial edema caused by the chemotactic factor of anaphylaxis b. Bronchial edema caused by binding of the cytotropic antibody c. Smooth muscle contraction caused by histamine bound to H1 receptors d. Smooth muscle contraction caused by histamine bound to H2 receptors

ANS: C During an IgE-mediated hypersensitivity reaction, smooth muscle contraction caused by histamine bound to H1 receptors results in bronchospasms. The bronchospasm is not caused by edema or by histamine binding to H2 receptors.

The nurse reviews the patient's complete blood count (CBC) results and notes that the neutrophil levels are elevated, but monocytes are still within normal limits. This indicates what type of inflammatory response? a. Chronic b. Resolved c. Early stage acute d. Late stage acute

ANS: C Elevated neutrophils and monocytes within normal limits are findings indicative of early inflammatory response. Neutrophils increase in just a few hours, while it takes the body days to increase the monocyte levels. Chronic inflammation results in varying elevations in WBCs dependent on multiple issues. Elevated neutrophils are not indicative of resolved inflammation. Elevations in monocytes occur later in the inflammatory response.

A nurse is instructing her patient with ulcerative colitis regarding the need to avoid enteric coated medications. The nurse knows that the patient understands the reason for this teaching when he states which of the following? a. "The coating on these medications is irritating to my intestines." b. "I need a more immediate response from my medications than can be obtained from enteric coated medications." c. "Enteric coated medications are absorbed lower in the digestive tract and can be irritating to my intestines or inadequately absorbed by my inflamed tissue." d. "I don't need to use these medications because they cause diarrhea, and I have had enough trouble with diarrhea and rectal bleeding over the past weeks."

ANS: C Enteric coatings on medications are designed to prevent breakdown and absorption of the medication until lower in the digestive tract, usually to prevent stomach irritation or to reach a certain point in the digestive tract for optimal absorption. For the patient with ulcerative colitis, the intestinal lining is inflamed or susceptible to inflammation and can have impaired absorption; therefore, enteric coated medications should be avoided. The coating is not irritating, but the medication can be. The response time of the medication is not a concern in this instance. Enteric coated medicines do not cause diarrhea simply because they are enteric coated.

When the immunologist says that pathogens possess infectivity, what is the immunologist explaining? Infectivity allows pathogens to: a. spread from one individual to others and cause disease. b. induce an immune response. c. invade and multiply in the host. d. damage tissue.

ANS: C Infectivity is the ability of the pathogen to invade and multiply in the host. Communication is the ability to spread from one individual to others and cause disease. Immunogenicity is the ability of pathogens to induce an immune response. Damaging tissues is the pathogen's mechanism of action.

A diabetic patient is brought into the emergency department unresponsive. The arterial pH is 7.28. Besides the blood pH, which clinical manifestation is seen in uncontrolled diabetes mellitus and ketoacidosis? a. Oral temperature of 38.9° Celsius b. Severe orthostatic hypotension c. Increased rate and depth of respiration d. Extremity tremors followed by seizure activity

ANS: C Ketoacidosis decreases the pH of the blood, stimulating the respiratory control area of the brain to buffer the effects of the increasing acidosis. The rate and depth of respirations are increased (Kussmaul's respirations) to excrete more acids by exhalation.

4. A patient who is diagnosed with acquired immunodeficiency syndrome (AIDS) tells the nurse, "I feel obsessed with morbid thoughts about dying." Which response by the nurse is appropriate? a. "Thinking about dying will not improve the course of AIDS." b. "Do you think that taking an antidepressant might be helpful?" c. "Can you tell me more about the thoughts that you are having?" d. "It is important to focus on the good things about your life now."

ANS: C More assessment of the patient's psychosocial status is needed before taking any other action. The statements, "Thinking about dying will not improve the course of AIDS" and "It is important to focus on the good things in life" or suggesting an antidepressant discourage the patient from sharing any further information with the nurse and decrease the nurse's ability to develop a trusting relationship with the patient. DIF: Cognitive Level: Apply (application) REF: 227 TOP: Nursing Process: Implementation MSC: NCLEX:

21. Tissue damage caused by the deposition of circulating immune complexes containing an antibody against the host DNA is the cause of which disease? a. Hemolytic anemia b. Pernicious anemia c. Systemic lupus erythematosus d. Myasthenia gravis

ANS: C The deposition of circulating immune complexes containing an antibody against the host DNA produces tissue damage in individuals with systemic lupus erythematosus (SLE). That is not a process in hemolytic anemia, pernicious anemia, or myasthenia gravis.

20. Which of these patients who have arrived at the human immunodeficiency virus (HIV) clinic should the nurse assess first? a. Patient whose rapid HIV-antibody test is positive b. Patient whose latest CD4+ count has dropped to 250/µL c. Patient who has had 10 liquid stools in the last 24 hours d. Patient who has nausea from prescribed antiretroviral drugs

ANS: C The nurse should assess the patient for dehydration and hypovolemia. The other patients also will require assessment and possible interventions, but do not require immediate action to prevent complications such as hypovolemia and shock. DIF: Cognitive Level: Analyze (analysis) REF: 229 OBJ: Special Questions: Prioritization | Special Questions: Multiple Patients TOP: Nursing Process: Assessment MSC: NCLEX:

A patient was given a patch test to determine what allergen was responsible for their atopic dermatitis. The provider prescribes a steroid cream. What important instructions should the nurse give to the patient? a. Apply the cream generously to affected areas. b. Apply a thin coat to affected areas, especially the face. c. Apply a thin coat to affected areas; avoid the face and groin. d. Apply an antihistamine along with applying a thin coat of steroid to affected areas.

ANS: C The patient should avoid the face and groin area as these areas are sensitive and may become irritated or excoriated. An antihistamine cream would also excoriate the area if the pruritus is cause by an allergen. There may be a need to administer oral steroid if the rash is generalized. REF: Pages 266-267

The nurse is reviewing instructions for a patient with type 2 diabetes who also takes insulin injections as part of the therapy. The nurse asks the patient, "What should you do if your fasting blood glucose is 47 mg/dL?" Which response by the patient reflects a correct understanding of insulin therapy? a. "I will call my doctor right away." b. "I will give myself the regular insulin." c. "I will take an oral form of glucose." d. "I will rest until the symptoms pass."

ANS: C Hypoglycemia can be reversed if the patient eats glucose tablets or gel, corn syrup, or honey, or drinks fruit juice or a nondiet soft drink or other quick sources of glucose, which must always be kept at hand. She should not wait for instructions from her physician, nor delay taking the glucose by resting. The regular insulin would only lower her blood glucose levels more. DIF: COGNITIVE LEVEL: Applying (Application) REF: p. 518 TOP: NURSING PROCESS: Implementation MSC: NCLEX: Physiological Integrity: Pharmacological and Parenteral Therapies

A patient in the emergency department was showing signs of hypoglycemia and had a fingerstick glucose level of 34 mg/dL. The patient has just become unconscious. What is the nurse's next action? a. Have the patient eat glucose tablets. b. Have the patient consume fruit juice, a nondiet soft drink, or crackers. c. Administer intravenous glucose (50% dextrose). d. Call the lab to order a fasting blood glucose level.

ANS: C Intravenous glucose raises blood glucose levels when the patient is unconscious and unable to take oral forms of glucose. DIF: COGNITIVE LEVEL: Analyzing (Analysis) REF: p. 514 TOP: NURSING PROCESS: Implementation MSC: NCLEX: Safe and Effective Care Environment: Management of Care

5. When insulin binds to its receptors on muscle cells, an increase in glucose uptake by the muscle cells occurs. This is an example of a _____ effect by a hormone. a. Pharmacologic b. Permissive c. Biphasic d. Direct

ANS: D Direct effects are the obvious changes in cell function that result specifically from stimulation by a particular hormone as is true with insulin. Pharmacologic effects are the result of high doses of a drug. Permissive effects are less obvious hormone-induced changes that facilitate the maximal response or functioning of a cell. Biphasic effects are twofold effects.

8. A patient who is human immunodeficiency virus (HIV)-infected has a CD4+ cell count of 400/µL. Which factor is most important for the nurse to determine before the initiation of antiretroviral therapy (ART) for this patient? a. CD4+ cell count trajectory b. HIV genotype and phenotype c. Patient's tolerance for potential medication side effects d. Patient's ability to follow a complex medication regimen

ANS: D Drug resistance develops quickly unless the patient takes ART medications on a strict, regular schedule. In addition, drug resistance endangers both the patient and community. The other information is also important to consider, but patients who are unable to manage and follow a complex drug treatment regimen should not be considered for ART. DIF: Cognitive Level: Analyze (analysis) REF: 223 TOP: Nursing Process: Assessment MSC:

2. A patient who has a positive test for human immunodeficiency virus (HIV) antibodies is admitted to the hospital with Pneumocystis jiroveci pneumonia (PCP) and a CD4+ T-cell count of less than 200 cells/L. Based on diagnostic criteria established by the Centers for Disease Control and Prevention (CDC), which statement by the nurse is correct? a. "The patient will develop symptomatic HIV infection within 1 year." b. "The patient meets the criteria for a diagnosis of acute HIV infection." c. "The patient will be diagnosed with asymptomatic chronic HIV infection." d. "The patient has developed acquired immunodeficiency syndrome (AIDS)."

ANS: D Development of PCP meets the diagnostic criteria for AIDS. The other responses indicate earlier stages of HIV infection than is indicated by the PCP infection. DIF: Cognitive Level: Understand (comprehension) REF: 221 TOP: Nursing Process: Assessment MSC:

The nurse is teaching a group of patients about self-administration of insulin. What content is important to include? a. Patients need to use the injection site that is the most accessible. b. If two different insulins are ordered, they need to be given in separate injections. c. When mixing insulins, the cloudy (such as NPH) insulin is drawn up into the syringe first. d. When mixing insulins, the clear (such as regular) insulin is drawn up into the syringe first.

ANS: D If mixing insulins in one syringe, the clear (regular) insulin is always drawn up into the syringe first. Patients always need to rotate injection sites. Mixing of insulins may be ordered. DIF: COGNITIVE LEVEL: Applying (Application) REF: p. 516 TOP: NURSING PROCESS: Implementation MSC: NCLEX: Physiological Integrity: Pharmacological and Parenteral Therapies

An older client with severe burns over more than half of the body has an indwelling catheter. When evaluating the client's intake and output, what should be taken into consideration? A) The amount of urine will be elevated due to the amount of intravenous fluids administered during the initial phases of treatment. B) The amount of urine will be reduced in the first 24 to 48 hours and will then increase. C) The amount of urine will be reduced during the first 8 hours of the burn injury and will then increase as diuresis begins. D) The amount of urine output will be greatest in the first 24 hours after the burn injury.

Answer: B Explanation: The client will have an initial reduction in urinary output. Fluid is reduced in the initial phases as the body manages the insult caused by the injury and fluids are drawn into the interstitial spaces. After the shock period passes, the client will enter a period of diuresis. Diuresis begins between 24 and 36 hours after the burn injury.

A client with a burn injury is prescribed mechanical debridement of the wounds. What will the nurse plan to do when performing mechanical debridement? Select all that apply. A) Schedule the client for a homograft. B) Apply a topical agent to dissolve necrotic tissue. C) Irrigate the burn wounds. D) Apply wet-to-dry gauze dressings. E) Schedule the client for hydrotherapy.

Answer: C, D, E Explanation: Mechanical debridement is done by applying and removing wet-to-dry gauze dressings, using hydrotherapy, or using irrigation. Applying a topical agent to dissolve necrotic tissue is an example of enzymatic debridement. The application of a homograft is a type of dressing and not a type of debridement.

The nurse is concerned that a client is at a high risk for a burn injury. What did the nurse assess in this client? Select all that apply. A) Part-time employment at a convenience store B) Diagnosis of hypertension C) Age 71 years D) Utilizes public transportation for grocery shopping E) Currently smokes 1 pack per day of cigarettes

Answer: C, E Explanation: Older clients are more vulnerable to fire and burn injury because of decreased visual acuity, depth perception, sense of smell, and hearing, and impaired mobility. Alterations in cognition, such as dementia, are also risk factors. Careless smoking is another risk factor. All of these factors increase the risk of accidentally starting a fire and diminish the ability to survive it. Hypertension does not increase the client's risk for experiencing a burn injury. Part-time employment and use of public transportation do not increase the client's risk of experiencing a burn injury.

10. A parent does not want their child to have any extra immunizations for diseases that no longer occur. What teaching about immunization should the nurse provide this mother? A. There is currently no need for those older vaccines. B. There is a reemergence of some of the infections, such as pertussis. C. There is no longer an immunization available for some of those diseases. D. The only way to protect your child is to have the federally required vaccines.

B Teaching the parent that some of the diseases are reemerging and the damage they can do to her child gives the mother the information to make an informed decision. The immunizations still exist and do protect individuals.

You would expect the health care provider to order which of the following as the initial treatment for acute cluster headache? A. Ice cap for 30 minutes, four times a day B. 100% oxygen delivered at a rate of 6-8 L/min for 10 minutes C. Aspirin D. Indomethacin

B Acute treatment of cluster headache is inhalation of 100% oxygen delivered at a rate of 6-8 L/min for 10 minutes, which may relieve headache by causing vasoconstriction and increasing synthesis of serotonin in the central nervous system. It can be repeated after a 5-minute rest. Reference: 1490

Which statements accurately describe MCI (select all that apply)? A. Always progresses to AD B. Caused by variety of factors and may progress to AD C. Should be aggressively treated with acetylcholinesterase drugs D. Caused by vascular infarcts, which if treated, can delay progression to AD E. Patient is usually not aware that there is a problem with his or her memory

B Although some individuals with MCI revert to normal cognitive function or do not go on to develop AD, those with MCI are at high risk for AD. No drugs have been approved for the treatment of MCI. Research is being conducted to determine whether patients with MCI can benefit from the medications used in AD (e.g., acetylcholinesterase inhibitors). A person MCI is often aware of a significant change in memory.

An elderly patient fell at home. Which information from the patient's history makes this patient at high risk for an intracerebral bleed? A. History of a heart condition B. Taking warfarin (Coumadin) C. Has lost consciousness for 5 seconds D. History of migraine headaches

B Anticoagulant use is associated with increased hemorrhage and more severe head injury. A heart condition may have caused the syncope that caused the fall, but it was not solely responsible for increased bleeding. Concussions are usually minor injuries that resolve, and the typical signs include a brief disruption in level of consciousness (LOC). If the loss of consciousness is less than 5 minutes, patients are usually discharged. Headache by itself does not indicate a risk for intracerebral bleeding.

You would question the use of sumatriptan (Imitrex) in which patient? A. A patient with a history of gastric ulcer disease B. A patient with a history of heart disease C. A person allergic to yeast D. A person allergic to cheese

B Because these drugs cause vasoconstriction of coronary arteries, they need to be avoided in patients with heart disease. Reference: 1489

You are caring for a patient admitted with a subdural hematoma after a motor vehicle accident. Which change in vital signs would you interpret as a manifestation of increased intracranial pressure? A. Tachypnea B. Bradycardia C. Hypotension D. Narrowing pulse pressure

B Changes in vital signs indicative of increased ICP are known as Cushing's triad, which consists of increasing systolic pressure with a widening pulse pressure, bradycardia with a full and bounding pulse, and irregular respirations.

A patient has ptosis secondary to myasthenia gravis. Which assessment finding would you expect to see in this patient? A. Redness and swelling of the conjunctiva B. Drooping of the upper lid margin in one or both eyes C. Redness, swelling, and crusting along the lid margins D. Small, superficial white nodules along the lid margin

B Ptosis is the term used to describe drooping of the upper lid margin, which may be either unilateral or bilateral. Ptosis can be a result of mechanical causes, such as an eyelid tumor or excess skin, or from myogenic causes such as myasthenia gravis. Reference: 1512, 1513

Which measure is most effective in preventing the spread of viral meningitis? A. Have close personnel wear surgical masks. B. Avoid touching respiratory secretions. C. Obtain yearly vaccinations. D. Gargle daily with salt wate

B Viral meningitis is usually acquired through direct contact with respiratory secretions. Bacterial meningitis has respiratory precautions until the cultures are negative, and precautions require a particulate respirator mask rather than surgical mask. There is no yearly vaccination for viral meningitis. The use of Haemophilus influenzae vaccine has decreased the incidence of bacterial meningitis caused by this organism, and the vaccine against Neisseria meningitides provides protection against that organism that causes bacterial meningitis. Gargling with salt water is a helpful home measure, but not touching infectious secretions is more effective.

A neurologist is teaching about the region responsible for motor aspects of speech. Which area is the neurologist discussing? a. Wernicke area b. Broca area c. Brodman 4 d. Brodmann area 6

B. Broca Area

When teaching a patient about insulin glargine (Lantus), which statement by the nurse about this drug is correct? A. "You can mix this insulin with NPH insulin to enhance its effects on glucose metabolism." B. "You cannot mix this insulin with regular insulin and thus will have to take two injections." C. "It is often combined with regular insulin to decrease the number of insulin injections per day." D. "The duration of action for this insulin is 8 to10 hours, so you will need to take it twice a day."

B. "You cannot mix this insulin with regular insulin and thus will have to take two injections." Correct Insulin glargine is a long-acting insulin with duration of action up to 24 hours. It should not be mixed with any other insulins. It is usually dosed once daily, but it may be dosed every 12 hours depending on the patient's glycemic response.

To monitor for complications in a patient with type 2 diabetes, which tests will the nurse in the diabetic clinic schedule at least annually? (Select all that apply.) a. Chest x-ray b. Blood pressure c. Serum creatinine d. Urine for microalbuminuria e. Complete blood count (CBC) f. Monofilament testing of the foot

B. Blood pressure C. Serum creatinine D. Urine for microalbuminuria F. monofilament testing of the foot

On a​ hot, humid​ day, a client presents with a body temperature of​ 40.9°C (105.6°F), dry and flush​ skin, vomiting, low blood​ pressure, and muscle cramps. Which type of injury should the nurse suspect based on the​ manifestations? A. Malignant hyperthermia B. Heat stroke C. Hypothermia D. Normothermia

B. Heat stroke Rationale: The nurse should suspect heat​ stroke, which can occur during hot weather and high humidity and results in dysfunction of the​ brain's thermoregulation center. Signs and symptoms of​ heat-related injuries include​ paleness, dizziness, nausea and​ vomiting, fatigue, low blood​ pressure, muscle​ cramps, and fainting. Late signs include​ irritability, confusion,​ stupor, and coma. Hypothermia is a core body temperature below​ 35°C (95°F), and is classified as​ mild, 32dash-​35°C ​(89.6dash-​95°F); ​moderate, 28dash-​32°C ​(82.4dash-​89.6°F), or​ severe, below​ 28°C (less than​ 82.4°F). The usual range of core body temperature is called normothermia. The normal range for adults is between​ 36°C and​ 38.5°C (96.8°F and​ 101.3°F). Malignant hyperthermia is a potentially​ fatal, inherited disorder that results from the​ body's reaction to volatile inhalation of anesthetic gases and​ succinylcholine, a depolarizing neuromuscular blocker.

A nurse recalls characteristics of upper motor neurons include? a. Directly inner acting muscles b. Influencing and modifying spinal reflex arcs c. Cell bodies located in the gray matter of spinal cord d. Dendritic processes extending out of the CNS

B. Influencing and modifying spinal reflex arcs

Jan is a 70-year-old retired nurse who is interested in nondrug, mind-body therapies, self-management, and alternative strategies to deal with joint discomfort from rheumatoid arthritis. What options should you consider in her plan of care considering her expressed wishes? a. Stationary exercise bicycle, free weights, and spinning class b. Mind-body therapies such as music therapy, distraction techniques, meditation, prayer, hypnosis, guided imagery, relaxation techniques, and pet therapy c. Chamomile tea and IcyHot gel d. Acupuncture and attending church services

B. Mind-body therapies such as music therapy, distraction techniques, meditation, prayer, hypnosis, guided imagery, relaxation techniques, and pet therapy Enhancing the mind's capacity to affect body functions

When a presynaptic neuron is stimulated in a patients body by an electrical current, neurotransmitters are released from the: A. Synapse B. Synaptic Bouton C. Synaptic Cleft D. Receptor

B. Synaptic Bouton

A patient with a head injury has bloody drainage from the ear. To determine whether CSF is present in the drainage, the nurse a. examines the tympanic membrane for a tear b. tests the fluid for a halo sign on a white dressing c. tests the fluid with a glucose identifying strip or stick d. collects 5 mL of fluid in a test tube and sends it to the laboratory for analysis

B. Tests the fluid for a halo sing on a white dressing- Testing clear drainage for CSF in nasal or ear drainage may be done with a Dextrostik or Tes-Tape strip, but if blood is present, the glucose in the blood will produce and unreliable result. To test bloody drainage, the nurse should test the fluid for a halo or ring that occurs when a yellowish ring encircles blood dripped onto a white pad or towel

Which patient action indicates accurate understanding of the nurse's teaching about administration of aspart (NovoLog) insulin? a. The patient avoids injecting the insulin into the upper abdominal area. b. The patient cleans the skin with soap and water before insulin administration. c. The patient stores the insulin in the freezer after administering the prescribed dose. d. The patient pushes the plunger down while removing the syringe from the injection site.

B. The patient cleans the skin with soap and water before insulin administration.

7. A pregnant woman who was tested and diagnosed with human immunodeficiency virus (HIV) infection is very upset. What should the nurse teach this patient about her baby's risk of being born with HIV infection? A. "The baby will probably be infected with HIV." B. "Only an abortion will keep your baby from having HIV." C. "Treatment with antiretroviral therapy will decrease the baby's chance of HIV infection." D. "The duration and frequency of contact with the organism will determine if the baby gets HIV infection."

C On average, 25% of infants born to women with untreated HIV will be born with HIV. The risk of transmission is reduced to less than 2% if the infected pregnant woman is treated with antiretroviral therapy. Duration and frequency of contact with the HIV organism is one variable that influences whether transmission of HIV occurs. Volume, virulence, and concentration of the organism as well as host immune status are variables related to transmission via blood, semen, vaginal secretions, or breast milk.

What is the best preventive measure for West Nile virus encephalitis? A. Yearly vaccination B. Contact isolation C. Avoidance of standing water D. Prophylactic acyclovir (Zovirax)

C Encephalitis is usually caused by a virus that is transmitted by ticks and mosquitoes. To prevent encephalitis, avoid standing water where mosquitoes can breed. There is no vaccination. Contact isolation does not prevent contracting the disease. Acyclovir is used to treat the disease.

You trace the number 8 on the patient's palm, and ask the patient to identify the number. What is this diagnostic test indicating? A. Function of cranial nerves XI and XII B. Stereognosis C. Cortical integration in the parietal lobe D. Spinocerebellar tracts

C Graphesthesia, the ability to feel writing on skin, is a test of cortical integration of sensory perceptions (which occurs in the parietal lobes). Cranial nerve XI (accessory) is tested by shrugging the shoulders and turning the head to the side, and cranial nerve XII (hypoglossal) is tested by protruding the tongue. Stereognosis is the ability to perceive the form and nature of objects. Spinocerebellar tracts carry information about muscle tension and body position to the cerebellum.

During neurologic testing, the patient is able to perceive pain elicited by a pinprick. Based on this finding, which assessment may be omitted? A. Position sense B. Patellar reflexes C. Temperature perception D. Heel-to-shin movements

C If pain sensation is intact, assessment of temperature sensation may be omitted because both sensations are carried by the same ascending pathways.

Bladder training for a male patient who has urinary incontinence after a stroke includes A. limiting fluid intake. B. keeping a urinal in place at all times. C. assisting the patient to stand to void. D. catheterizing the patient every 4 hours.

C In the acute stage of stroke, the primary urinary problem is poor bladder control and incontinence. Nurses should promote normal bladder function and avoid the use of indwelling catheters. A bladder retraining program consists of (1) adequate fluid intake, with most given between 8:00 AM and 7:00 PM; (2) scheduled toileting every 2 hours using a bedpan, commode, or bathroom; and (3) observing signs of restlessness, which may indicate the need for urination. Intermittent catheterization may be used for urinary retention (not urinary incontinence). During the rehabilitation phase after a stroke, nursing interventions focused on urinary continence include (1) assessment for bladder distention by palpation; (2) offering the bedpan, urinal, commode, or toilet every 2 hours during waking hours and every 3 to 4 hours at night; (3) focusing the patient on the need to urinate with a direct command; (4) assistance with clothing and mobility; (5) scheduling most fluid intake between 7:00 AM and 7:00 PM; and (6) encouraging the usual position for urinating (standing for men and sitting for women).

A male patient with a diagnosis of Parkinson's disease (PD) has been recently admitted to a long-term care facility. Which action should the health care team take in order to promote adequate nutrition for this patient? A. Provide multivitamins with each meal. B. Provide a diet that is low in complex carbohydrates and high in protein. C. Provide small, frequent meals throughout the day that are easy to chew and swallow. D. Provide the patient with a minced or pureed diet that is high in potassium and low in sodium.

C Nutritional support is a priority in the care of individuals with PD. Such patients may benefit from meals that are smaller and more frequent than normal and which are easy to chew and swallow. Multivitamins are not necessary at each meal, and vitamin intake, along with protein intake, must be monitored to prevent contraindications with medications. It is likely premature to introduce a minced or pureed diet and a low carbohydrate diet is not indicated. Reference: 1509

What is most important finding for you to act on for a patient who had a craniotomy? A. Sodium: 134 mEq/L B. While blood cell (WBC) count: 11,000/μL C. Urine specific gravity: 1.001 D. Blood urea nitrogen (BUN): 25 mg/dL

C Patients need frequent monitoring for sodium regulation, onset of diabetes insipidus, and severe hypovolemia. Normal specific gravity for urine should not be below 1.003 and this low value is a priority.

The patient had a stroke on the left side of the brain (right hemiplegia). He is eating dinner and suddenly bursts into tears. How do you respond? A. Ask the patient about his feelings. B. Give him a tissue. C. Distract the patient. D. Obtain an antidepressant for patient.

C Patients who have had strokes often exhibit emotional responses that are not appropriate for the situation. The behavior is out of context and often is unrelated to the underlying emotional state of the patient. Initially it is important to just distract the patient.

Which sensory-perceptual deficit is associated with a left-brain stroke? A. Overestimation of physical abilities B. Difficulty judging position and distance C. Slow and possibly fearful performance of tasks D. Impulsivity and impatience at performing tasks

C Patients with a left-brain stroke (right hemiplegia) commonly are slower in organization and performance of tasks and may have a fearful, anxious response to a stroke. Overconfidence, spatial disorientation, and impulsivity are more commonly associated with a right-brain stroke.

Which gastrointestinal complication would you expect in the patient with multiple sclerosis? A. Diarrhea B. Bowel obstruction C. Constipation D. Rectal prolapse

C Problems with defecation usually involve constipation rather than fecal incontinence. Reference: 1502

A 65-year-old woman was just diagnosed with Parkinson's disease. The priority nursing intervention is A. searching the Internet for educational videos. B. evaluating the home for environmental safety. C. promoting physical exercise and a well-balanced diet. D. designing an exercise program to strengthen and stretch specific muscles.

C Promotion of physical exercise and a well-balanced diet are major concerns for nursing care for patients with Parkinson's disease. Reference: 1509

During the nursing assessment the patient states that she is experiencing headaches bilaterally that are described as pressing or tightening. The headaches last from a few minutes to days and the pain is described as a 5 on a scale of 1 to 10. These signs and symptoms are consistent with which type of headache? A. Sinus B. Cluster C. Tension D. Migraine

C Tension-type headache, the most common type of headache, is characterized by its bilateral location and pressing/tightening quality. Tension-type headaches are usually of mild or moderate intensity and not aggravated by physical activity.

A young adult calls the clinic to ask for a prescription for "that new flu drug." He says he has had the flu for almost 4 days and just heard about a drug that can reduce the symptoms. What is the nurse's best response to his request? a. "Now that you've had the flu, you will need a booster vaccination, not the antiviral drug." b. "We will need to do a blood test to verify that you actually have the flu." c. "Drug therapy should be started within 2 days of symptom onset, not 4 days." d. "We'll get you a prescription. As long as you start treatment within the next 24 hours, the drug should be effective."

C These drugs need to be started within 2 days of influenza symptom onset; they can be used for prophylaxis and treatment of influenza. The other options are incorrect.

You are assigned to four patients on the clinical unit. Which patient should you assess first? A. Patient with a skull fracture whose nose is bleeding B. Elderly patient with a stroke who is confused and whose daughter is present C. Patient with meningitis who is suddenly agitated and reporting a headache of 10 on a 0 to 10 scale D. Patient who had a craniotomy for a brain tumor 3 days earlier and has continued emesis

C The patient with meningitis should be seen first. Patients with meningitis must be observed closely for manifestations of increased ICP, which is thought to be a result of swelling around the dura and increased cerebrospinal fluid (CSF) volume. Sudden change in the level of consciousness or a change in behavior along with a sudden, severe headache may indicate an acute episode of increased ICP. The postoperative cranial surgery patient should be seen second; although nausea and vomiting are common after cranial surgery and can result in increased ICP. Nausea and vomiting should be treated with antiemetics. The patient with a skull fracture needs to be evaluated for CSF leakage occurring with the nosebleed and should be seen third. Confusion after a stroke may be expected; the patient should be safe with a family member present. Reference: 1452-1453

What is a typical description of a person diagnosed with MCI? A. Does not remember knowing a person B. Forgets what an item is used for C. Slow to recall people's names D. Loses sense of time and day

C The person with MCI frequently forgets people's names and is slow to recall them. The other options describe typical behaviors seen in AD.

Vascular dementia is associated with A. transient ischemic attacks. B. bacterial or viral infection of neuronal tissue. C. cognitive changes resulting from cerebral ischemia. D. abrupt changes in cognitive function that are irreversible

C Vascular dementia is the loss of cognitive function resulting from ischemic, ischemic-hypoxic, or hemorrhagic brain lesions caused by cardiovascular disease. This type of dementia is the result of decreased blood supply from narrowing and blocking of arteries that supply the brain.

Which nursing intervention is most appropriate when communicating with a patient suffering from aphasia after a stroke? A. Present several thoughts at once so that the patient can connect the ideas. B. Ask open-ended questions to provide the patient the opportunity to speak. C. Use simple, short sentences accompanied by visual cues to enhance comprehension. D. Finish the patient's sentences to minimize the frustration associated with slow speech.

C When communicating with a patient with aphasia, you should present one thought or idea at a time; ask questions that can be answered with yes, no, or another simple word; use visual cues; and allot time for the individual to comprehend and respond to the conversation.

Which nursing intervention is most appropriate when caring for patients with dementia? A. Avoid direct eye contact. B. Lovingly call the patient "honey" or "sweetie." C. Give simple directions, focusing on one thing at a time. D. Treat the patient according to their age-related behavior.

C When dealing with patients with dementia, tasks should be simplified, giving directions using gestures or pictures and focusing on one thing at a time. It is best to treat these patients as adults, with respect and dignity, even when their behavior is childlike. You should use gentle touch and direct eye contact. Calling the patient "honey" or "sweetie" can be condescending and does not demonstrate respect.

A patient is receiving cidofovir (Vistide) as part of treatment for a viral infection, and the nurse is preparing to administer probenecid, which is also ordered. Which is the rationale for administering probenecid along with the cidofovir treatment? a. Probenecid has a synergistic effect when given with cidofovir, thus making the antiviral medication more effective. b. The probenecid also prevents replication of the virus. c. Concurrent drug therapy with probenecid reduces the nephrotoxicity of the cidofovir. d. The probenecid reduces the adverse gastrointestinal effects of the cidofovir.

C Probenecid is recommended as concurrent drug therapy with cidofovir to help alleviate the nephrotoxic effects of probenecid. The other options are incorrect.

Nursing management of a patient with a brain tumor includes (select all that apply) A. discussing with the patient methods to control inappropriate behavior. B. using diversion techniques to keep the patient stimulated and motivated. C. assisting and supporting the family in understanding changes in behavior. D. limiting self-care activities until the patient has regained maximum physical functioning. E. planning for seizure precautions and teaching the patient and caregiver about antiseizure drugs.

C,E Nursing interventions should be based on a realistic appraisal of the patient's condition and prognosis after cranial surgery. You should provide support and education to the caregiver and family related to the patient's behavior changes. You should be prepared to manage seizures and teach the caregiver and family about antiseizure medications and how to manage a seizure. The overall goal is to foster patient independence for as long as possible and to the highest degree possible. You should decrease stimuli in the patient's environment to prevent increases in ICP.

When a patient asks what the somatic nervous system controls, how should the nurse respond? It controls: A. The heart B. The spinal cord C. Skeletal Muscle D. Smooth muscle organs

C. Skeletal muscle

An elderly Chinese woman is interested in biologically based therapies to relieve osteoarthritis pain (OA). You are preparing a plan of care for her OA. Options most conducive to her expressed wishes may include a. Pilates, breathing exercises, and aloe vera. b. Guided imagery, relaxation breathing, and meditation. c. Herbs, vitamins, and tai chi. d. Alternating ice and heat to relieve pain and inflammation.

C. Herbs, vitamins, and tai chi Biological based measures

A school nurse is recommending a​ school-wide initiative to reduce the risk of​ heat-related injuries in athletes. Which recommendation should the nurse​ include? A. Cancel athletic games when the temperature is above 80degrees°F. B. Reduce athletic activities at the school. C. Increase access to​ fresh, cold water. D. Encourage the school to move athletic activities indoors.

C. Increase access to fresh, cold water ​Rationale: Increasing access to fresh cold water and encouraging frequent water breaks can decrease the risk of hyperthermia. It is highly unlikely that the school will reduce athletic​ activities, and it may not be possible to cancel games when the temperature is above​ 80°F. The school may not have the facilities to host indoor​ athletics, and it​ doesn't address the needs of athletes when they play at other locations.

Natural killer cells are specialized lymphocytes that are the major parts of which of the following types of immunity? A. Cell-mediated immunity B. Adaptive immunity C. Innate immunity D. Humoral (antibody) immunity

C. Innate immunity

most significant drug interactions with use of antivirals occur when antivirals are administered via which route? A. Topically B. Rectally C. Intravenously D. Optically

C. Intravenously Significant drug interactions that occur with antiviral drugs arise most often when they are administered via systemic routes such as intravenously and orally. Many of these drugs are also applied topically to the eye or body, however, and the incidence of drug interactions associated with these routes of administration is much lower

Which oral hypoglycemic drug has a quick onset and short duration of action, enabling the patient to take the medication 30 minutes before eating and skip the dose if he or she does not eat? A. Acarbose (Precose) B. Metformin (Glucophage) C. Repaglinide (Prandin) D. Pioglitazone (Actos)

C. Repaglinide (Prandin) Correct Repaglinide is known as the "Humalog of oral hypoglycemic drugs." The drug's very fast onset of action allows patients to take the drug with meals and skip a dose when they skip a meal.

A 54-year old man is recovering from a skull fracture with a subacute subdural hematoma. He has return of motor control and orientation but appears apathetic and has reduced awareness of his environment. When planning discharge or the patient, the nurse explains to the patient and the family that a. continuous improvement in the patient's condition should occur until he has returned to pre trauma status b. the patient's complete recovery may take years, and the family should plan for his long term dependent care c. the patient is likely to have long term emotional and mental changes that may require continued professional help d. role changes in family members will be necessary because the patient will be dependent on his family for care and support

C. Residual mental and emotional changes of brain trauma with personality changes are often the most incapacitating problems following head injury and are common in patients who have been comatose longer than 6 hours. Families must be prepared for changes in the patient's behavior to avoid family-patient friction and maintain family functioning, and professional assistance may be required. There is no indication he will be dependent on others for care, but he likely will not return to pre trauma status

The nurse monitoring a patient for a therapeutic response to oral antidiabetic drugs will look for A. fewer episodes of diabetic ketoacidosis (DKA). B. weight loss of 5 pounds. C. hemoglobin A1C levels of less than 7%. D. glucose levels of 150 mg/dL.

C. hemoglobin A1C levels of less than 7%.

The patient with bacterial meningitis has irritation of cranial nerve (CN) II. What symptom would you expect the patient to have? A. Inability to hear whispered words B. Abnormal pronation and supination test result C. Papilledema D. Dysphagia

CN II is the optic nerve, and when compressed from the increased ICP, papilledema (swelling of the optic disc) is often present and blindness may occur. The acoustic nerve is CN VIII. Assessing abnormal pronation or supination is a coordination/balance test for cerebellar function. Dysphagia can be a symptom of CN IX and CN X abnormalities.

When the nurse applies a painful stimulus to the nailbeds of an unconscious patient, the patient responds with internal rotation, adduction, and flexion of the arms. The nurse documents this as a. decorticate posturing. b. decerebrate posturing. c. localization of pain. d. flexion withdrawal.

Correct Answer: A Rationale: Internal rotation, adduction, and flexion of the arms in an unconscious patient is documented as decorticate posturing. Extension of the arms and legs is decerebrate posturing. Because the flexion is general, it does not indicate localization of pain or flexion withdrawal. Cognitive Level: Comprehension Text Reference: p. 1472 Nursing Process: Assessment NCLEX: Physiological Integrity

When caring for a patient who has had a head injury, which assessment information is of most concern to the nurse? a. The blood pressure increases from 120/54 to 136/62. b. The patient is more difficult to arouse. c. The patient complains of a headache at pain level 5 of a 10-point scale. d. The patient's apical pulse is slightly irregular.

Correct Answer: B Rationale: The change in level of consciousness (LOC) is an indicator of increased ICP and suggests that action by the nurse is needed to prevent complications. The change in BP should be monitored but is not an indicator of a need for immediate nursing action. Headache is not unusual in a patient after a head injury. A slightly irregular apical pulse is not unusual. Cognitive Level: Application Text Reference: p. 1470 Nursing Process: Assessment NCLEX: Physiological Integrity

A patient with possible cerebral edema has a serum sodium level of 115 mEq/L (115 mmol/L), a decreasing level of consciousness (LOC) and complains of a headache. All of the following orders have been received. Which one should the nurse accomplish first? a. Administer acetaminophen (Tylenol) 650 mg orally. b. Administer 5% hypertonic saline intravenously. c. Draw blood for arterial blood gases (ABGs). d. Send patient to radiology for computed tomography (CT) of the head.

Correct Answer: B Rationale: The patient's low sodium indicates that hyponatremia may be causing the cerebral edema, and the nurse's first action should be to correct the low sodium level. Acetaminophen (Tylenol) will have minimal effect on the headache because it is caused by cerebral edema and increased ICP. Drawing ABGs and obtaining a CT scan may add some useful information, but the low sodium level may lead to seizures unless it is addressed quickly. Cognitive Level: Application Text Reference: p. 1470 Nursing Process: Implementation NCLEX: Physiological Integrity

The nurse notes that a patient with a head injury has a clear nasal drainage. The most appropriate nursing action for this finding is to a. obtain a specimen of the fluid and send for culture and sensitivity. b. take the patient's temperature to determine whether a fever is present. c. check the nasal drainage for glucose with a Dextrostik or Testape. d. have the patient to blow the nose and then check the nares for redness.

Correct Answer: C Rationale: If the drainage is cerebrospinal fluid (CSF) leakage from a dural tear, glucose will be present. Fluid leaking from the nose will have normal nasal flora, so culture and sensitivity will not be useful. A dural tear does increase the risk for infections such as meningitis, but the nurse should first determine whether the clear drainage is CSF. Blowing the nose is avoided to prevent CSF leakage. Cognitive Level: Application Text Reference: p. 1481 Nursing Process: Implementation NCLEX: Physiological Integrity

When assessing a patient with a head injury, the nurse recognizes that the earliest indication of increased intracranial pressure (ICP) is a. vomiting. b. headache. c. change in level of consciousness (LOC). d. sluggish pupil response to light.

Correct Answer: C Rationale: LOC is the most sensitive indicator of the patient's neurologic status and possible changes in ICP. Vomiting and sluggish pupil response to light are later signs of increased ICP. A headache can be caused by compression of intracranial structures as the brain swells, but it is not unexpected after a head injury. Cognitive Level: Comprehension Text Reference: p. 1470 Nursing Process: Assessment NCLEX: Physiological Integrity

A patient with a head injury has an arterial blood pressure is 92/50 mm Hg and an intracranial pressure of 18 mm Hg. Which action by the nurse is appropriate? a. Document and continue to monitor the parameters. b. Elevate the head of the patient's bed. c. Notify the health care provider about the assessments. d. Check the patient's pupillary response to light.

Correct Answer: C Rationale: The patient's cerebral perfusion pressure is only 46 mm Hg, which will rapidly lead to cerebral ischemia and neuronal death unless rapid action is taken to reduce ICP and increase arterial BP. Documentation and monitoring are inadequate responses to the patient's problem. Elevating the head of the bed will lower the ICP but may also lower cerebral blood flow and further decrease CPP. Changes in pupil response to light are signs of increased ICP, so the nurse will only take more time doing this without adding any useful information. Cognitive Level: Analysis Text Reference: pp. 1468-1469 Nursing Process: Implementation NCLEX: Physiological Integrity

15. A patient was exposed to human immunodeficiency virus (HIV) 2 weeks ago through sharing needles with other substance users. What symptoms will the nurse teach the patient to report that would indicate the patient has developed an acute HIV infection? A. Cough, diarrhea, headaches, blurred vision, muscle fatigue B. Night sweats, fatigue, fever, and persistent generalized lymphadenopathy C. Oropharyngeal candidiasis or thrush, vaginal candidal infection, or oral or genital herpes D. Flu-like symptoms such as fever, sore throat, swollen lymph glands, nausea, or diarrhea

D Clinical manifestations of an acute infection with HIV include flu-like symptoms between 2 to 4 weeks after exposure. Early chronic HIV infection clinical manifestations are either asymptomatic or include fatigue, headache, low-grade fever, night sweats, and persistent generalized lymphadenopathy. Intermediate chronic HIV infection clinical manifestations include candidal infections, shingles, oral or genital herpes, bacterial infections, Kaposi sarcoma, or oral hairy leukoplakia. Late chronic HIV infection or acquired immunodeficiency syndrome (AIDS) includes opportunistic diseases (infections and cancer).

14. The nurse is providing postoperative care for a patient with human immunodeficiency virus (HIV) infection after an appendectomy. What type of precautions should the nurse observe to prevent the transmission of this disease? A. Droplet precautions B. Contact precautions C. Airborne precautions D. Standard precautions

D Standard precautions are indicated for prevention of transmission of HIV to the health care worker. HIV is not transmitted by casual contact or respiratory droplets. HIV may be transmitted through sexual intercourse with an infected partner; exposure to HIV-infected blood or blood products; and perinatal transmission during pregnancy, at delivery, or though breastfeeding.

Drugs or diseases that impair the function of the extrapyramidal system may cause loss of A. sensations of pain and temperature. B. regulation of the autonomic nervous system. C. integration of somatic and special sensory inputs. D. automatic movements associated with skeletal muscle activity.

D A group of descending motor tracts carries impulses from the extrapyramidal system, which includes all motor systems (except the pyramidal system) concerned with voluntary movement. It includes descending pathways originating in the brainstem, basal ganglia, and cerebellum. The motor output exits the spinal cord through the ventral roots of the spinal nerves

What information provided by the patient can help differentiate a hemorrhagic stroke from a thrombotic stroke? A. Sensory disturbance B. A history of hypertension C. Presence of motor weakness D. Sudden onset of severe headache

D A hemorrhagic stroke usually causes sudden onset of symptoms, including neurologic deficits, headache, nausea, vomiting, decreased level of consciousness, and hypertension. Ischemic stroke symptoms may progress in the first 72 hours as infarction and cerebral edema increase. Reference: 1464

Why is it difficult to compare the assessment of muscle strength of older adults with that of younger adults? A. Stroke is more common in older adults. B. Nutritional status is better in young adults. C. Most young people exercise more than older people. D. Muscle bulk and strength decrease with age.

D Changes associated with aging include decreases in muscle strength and agility related to a decrease in muscle bulk.

Paralysis of lateral gaze indicates a lesion of cranial nerve A. II. B. III. C. IV. D. VI.

D Cranial nerves III (oculomotor), IV (trochlear), and VI (abducens) are responsible for eye movement. The lateral rectus eye muscle is innervated by cranial nerve VI and is the primary muscle that is responsible for lateral eye movement

Which option indicates a sign of Cushing's triad, an indication of increased intracranial pressure (ICP)? A. Heart rate increases from 90 to 110 beats/minute B. Kussmaul respirations C. Temperature over 100.4° F (38° C) D. Heart rate decreases from 75 to 55 beats/minute

D Cushing's triad is systolic hypertension with a widening pulse pressure, bradycardia with a full and bounding pulse, and slowed respirations. The rise in blood pressure is an attempt to maintain cerebral perfusion, and it is a neurologic emergency because decompensation is imminent. The other options are not part of Cushing's triad.

What is a common cause of delirium? A. Prion protein B. Microscopic deposits of Pick bodies C. Abnormal deposits of protein α-synuclein D. Severe, acute illness

D Delirium is often the result of the interaction of the patient's underlying condition with a precipitating event. Common causes in a vulnerable (especially older) adult include fluid and electrolyte abnormalities, drug administration, surgery, acute or worsening significant illness, and withdrawal from drugs or alcohol. Prion protein is found in Creutzfeldt-Jakob disease. Microscopic deposits of Pick bodies are found in frontotemporal dementia (Pick's disease). Abnormal deposits of protein α-synuclein are found in Lewy body dementia. Reference: 1534

Dementia is defined as a A. syndrome that results only in memory loss. B. disease associated with abrupt changes in behavior. C. disease that is always due to reduced blood flow to the brain. D. syndrome characterized by cognitive dysfunction and loss of memory.

D Dementia is a syndrome characterized by dysfunction or loss of memory, orientation, attention, language, judgment, and reasoning. Personality changes and behavioral problems such as agitation, delusions, and hallucinations may result.

When assessing motor function of a patient admitted with a stroke, you notice mild weakness of the arm demonstrated by downward drifting of the extremity. How would you accurately document this finding? A. Athetosis B. Hypotonia C. Hemiparesis D. Pronator drift

D Downward drifting of the arm or pronation of the palm is identified as pronator drift. Hemiparesis is weakness of one side of the body, hypotonia describes flaccid muscle tone, and athetosis is a slow, writhing, involuntary movement of the extremities

The pain reliever of choice for patients experiencing a tension type headache is A. meperidine hydrochloride (Demerol). B. morphine sulfate (Morphine). C. codeine sulfate (Codeine). D. acetaminophen (Tylenol).

D Drug treatment for tension-type headache usually involves a nonopioid analgesic (e.g., aspirin, acetaminophen) used alone or in combination with a sedative, muscle relaxant, tranquilizer, or codeine.

What is a key clinical manifestation of classic bacterial meningitis? A. Temperature B. Nystagmus C. Tachycardia D. Nuchal rigidity

D Fever, severe headache, vomiting, and nuchal rigidity (neck stiffness) are key clinical manifestations of meningitis.

Which of the following patients is at highest risk for a stroke? A. An obese, 45-year-old Native American B. A 35-year-old Asian American woman who smokes C. A 32-year-old, white woman taking oral contraceptives D. A 65-year-old African American man with hypertension

D Nonmodifiable risk factors for stroke include age (>65 years), male gender, ethnicity or race (African Americans > Hispanics, Native Americans/Alaska Natives, and Asian Americans > whites), and family history of stroke or personal history of a transient ischemic attack or stroke. Modifiable risk factors for stroke include hypertension (most important), heart disease (especially atrial fibrillation), smoking, excessive alcohol consumption (causes hypertension), abdominal obesity, sleep apnea, metabolic syndrome, lack of physical exercise, poor diet (high in saturated fat and low in fruits and vegetables), and drug abuse (especially cocaine). Other risk factors for stroke include a diagnosis of diabetes mellitus, increased serum cholesterol, birth control pills (high levels of progestin and estrogen), history of migraine headaches, inflammatory conditions, hyperhomocysteinemia, and sickle cell disease.

A patient being monitored has an ICP pressure of 12 mm Hg. You understand that this pressure reflects A. a severe decrease in cerebral perfusion pressure. B. an alteration in the production of cerebrospinal fluid. C. the loss of autoregulatory control of intracranial pressure. D. a normal balance between brain tissue, blood, and cerebrospinal fluid.

D Normal ICP ranges from 5 to 15 mm Hg. A sustained pressure above the upper limit is considered abnormal.

You assess a patient for signs of meningeal irritation and observe her for nuchal rigidity. Which option indicates the presence of this sign of meningeal irritation? A. Tonic spasms of the legs B. Curling in a fetal position C. Arching of the neck and back D. Resistance to flexion of the nec

D Nuchal rigidity is a clinical manifestation of meningitis. During assessment, the patient resists passive flexion of the neck by the health care provider.

What is a common priority nursing need for a patient with Lewy body dementia? A. Dysphasia B. Apraxia C. Visual agnosia D. Dysphagia

D Patients with Lewy body dementia have problems with dysphagia and immobility. Swallowing problems can lead to impaired nutrition. All dementia patients can have problem with comprehending language and oral communication (dysphasia), inability to manipulate objects or perform purposeful acts (apraxia), and inability to recognize objects of sight (visual agnosia). Dysphagia (trouble with swallowing) is a priority.

A wife indicates she has been providing care to her husband, who was diagnosed with bacterial meningococcal meningitis. What is your most important action related to the wife? A. Teach airborne isolation precautions. B. Assess her for respiratory infection. C. Explain the signs of meningitis. D. Provide prophylactic antibiotics.

D Persons who have close contact with anyone who has bacterial meningitis should be given prophylactic antibiotics, and this supersedes all other actions because she has been exposed. Meningococcal meningitis is highly contagious.

Which statement accurately describes mild cognitive impairment (MCI)? A. People with MCI are often unaware that they have any deficiencies. B. Most people with MCI go on to develop dementia. C. You need to emphasize the need to take drug therapy consistently. D. You must monitor the person for disorientation or feeling lost in a familiar place.

D Persons with MCI should be monitored for the 10 signs of Alzheimer's disease (AD) and signs that they are progressing into dementia. The person with MCI is often aware of a significant change in memory, which also is noticed by family members. A person with dementia is unaware of deficits. Having MCI puts an individual at risk for dementia, and an estimated 15% of people with MCI eventually develop dementia. No drugs are approved for the treatment of MCI

Which patient may be at greatest risk for delirium? A. A patient with fibromyalgia whose chronic pain has recently worsened B. An elderly patient whose recent computed tomography study shows brain atrophy C. A patient with a fracture who has spent the night in the emergency department D. An elderly patient who takes multiple medications for various health problems

D Polypharmacy is implicated in many cases of delirium, and this phenomenon is especially common among older adults. Brain atrophy, if associated with cognitive changes, is indicative of dementia. Alterations in sleep and environment and pain may cause delirium, but this is less of a risk than taking multiple medications by an older adult.

An infant has been hospitalized with a severe lung infection caused by the respiratory syncytial virus (RSV) and will be receiving medication via the inhalation route. The nurse expects which drug to be used? a. Acyclovir (Zovirax) b. Ganciclovir (Cytovene) c. Amantadine (Symmetrel) d. Ribavirin (Virazole)

D The inhalational form of ribavirin (Virazole) is used primarily in the treatment of hospitalized infants with severe lower respiratory tract infections caused by RSV. The other drugs listed are not used for the treatment of RSV

Which patient is most at risk for delirium? A. A 50-year-old woman with cholecystitis B. A 19-year-old man with a fractured femur C. A 42-year-old woman having an elective hysterectomy D. A 78-year-old man admitted to the medical unit with complications related to heart failure

D Risk factors that can precipitate delirium include age 65 years or older, male gender, and severe, acute illness (e.g., heart failure). The 78-year-old man has the most risk factors for delirium.

You assess that an 87-year-old woman with Alzheimer's disease is continually rubbing, flexing, and kicking out her legs throughout the day. The night shift reports that this same behavior escalates at night, preventing her from obtaining her required sleep. The next step the nurse should take is to A. ask the physician for a daytime sedative for the patient. B. request soft restraints to prevent her from falling out of her bed. C. ask the physician for a nighttime sleep medication for the patient. D. assess the patient more closely, suspecting a disorder such as restless legs syndrome

D The severity of restless legs syndrome (RLS) sensory symptoms ranges from infrequent minor discomfort (paresthesias, including numbness, tingling, and "pins and needles" sensation) to severe pain. The discomfort occurs when the patient is sedentary and is most common in the evening or at night. The pain at night can produce sleep disruptions and is often relieved by physical activity such as walking, stretching, rocking, or kicking. In the most severe cases, patients sleep only a few hours at night, resulting in daytime fatigue and disruption of the daily routine. The motor abnormalities associated with RLS consist of voluntary restlessness and stereotyped, periodic, involuntary movements. The involuntary movements usually occur during sleep. Symptoms are aggravated by fatigue. Reference: 1514

In a patient with a disease that affects the myelin sheath of nerves, such as multiple sclerosis, which glial cells are affected?

D Types of glial cells include oligodendrocytes, astrocytes, ependymal cells, and microglia, and each has specific functions. Oligodendrocytes are specialized cells that produce the myelin sheath of nerve fibers, and they are primarily found in the white matter of the central nervous system.

What is the most common visual field change resulting from a brain lesion? A. Diplopia B. Blurred vision C. Presbyopia D. Hemianopsia

D Visual field changes resulting from brain lesions are usually diagnosed as hemianopsia (one half of the visual field) or quadrantanopsia (one fourth of the visual field) or monocular vision.

A patient is taking a combination of antiviral drugs as treatment for early stages of a viral infection. While discussing the drug therapy, the patient asks the nurse if the drugs will kill the virus. When answering, the nurse keeps in mind which fact about antiviral drugs? a. They are given for palliative reasons only. b. They will be effective as long as the patient is not exposed to the virus again. c. They can be given in large enough doses to eradicate the virus without harming the body's healthy cells. d. They may also kill healthy cells while killing viruses.

D Because viruses reproduce in human cells, selective killing is difficult; consequently, many healthy human cells, in addition to virally infected cells, may be killed in the process, and this results in the serious toxicities that are involved with these drugs. The other options are incorrect.

A patient who is HIV- positive has been receiving medication therapy that includes zidovudine (Retrovir). However, the prescriber has decided to stop the zidovudine because of its doselimiting adverse effect. Which of these conditions is the dose-limiting adverse effect of zidovudine therapy? a. Retinitis b. Renal toxicity c. Hepatotoxicity d. Bone marrow suppression

D Bone marrow suppression is often the reason that a patient with HIV infection has to be switched to another anti-HIV drug such as didanosine. The two drugs can be taken together, cutting back on the dosages of both and thus decreasing the likelihood of toxicity. The other options are incorrect.

A nurse analyzes the results of a Romberg test performed on a client with Parkinson's disease. Which finding during testing best indicates that the client has a positive Romberg test? A. Client marches in place B. Client stands quietly C. Client sways slightly D. Client begins to fall

D.

A diabetic patient has proliferative retinopathy, nephropathy, and peripheral neuropathy. What should the nurse teach this patient about exercise? A. "Jogging for 20 minutes 5 to 7 days a week would most efficiently help you to lose weight." B. "One hour of vigorous exercise daily is needed to prevent progression of disease." C. "Avoid all forms of exercise because of your diabetic complications." D. "Swimming or water aerobics 30 minutes each day would be the safest exercise routine for you."

D. "Swimming or water aerobics 30 minutes each day would be the safest exercise routine for you."

The nurse is providing education to a patient for the prescription glipizide (Glucotrol). The nurse explains this medication is more effective when administered at which time? A. At bedtime B. In the morning C. 15 minutes postprandial D. 30 minutes before a meal

D. 30 minutes before a meal Correct Glipizide works best if given 30 minutes before meals. This allows the timing of the insulin secretion induced by the glipizide to correspond to the elevation in the blood glucose level induced by the meal.

After change-of-shift report, which patient will the nurse assess first? a. A 19-yr-old patient with type 1 diabetes who was admitted with dawn phenomenon b. A 35-yr-old patient with type 1 diabetes whose most recent blood glucose reading was 230 mg/dL c. A 68-yr-old patient with type 2 diabetes who has severe peripheral neuropathy and reports burning foot pain d. A 60-yr-old patient with hyperosmolar hyperglycemic syndrome who has poor skin turgor and dry oral mucosa

D. A 60-yr-old patient with hyperosmolar hyperglycemic syndrome who has poor skin turgor and dry oral mucosa

A nurse is preparing to teach about functions to maintain homeostasis and instinctive behavioral patterns. Which area of the brain is the nurse discussing? a. Thalamus b. Medulla c. Cerebellum d. Hypothalamus

D. Hypothalamus

The nurse is preparing a patient for a computed tomography scan using iodine contrast media. Which medication should the nurse question if prescribed one day before the scheduled procedure? A. Acarbose (Precose) B. Pioglitazone (Actos) C. Repaglinide (Prandin) D. Metformin (Glucophage)

D. Metformin (Glucophage) Correct The concurrent use of metformin with iodinated (iodine-containing) radiologic contrast media has been associated with both acute renal failure and lactic acidosis. Therefore, metformin should be discontinued the day of the test and for at least 48 hours after the patient undergoes any radiologic study that requires the use of such contrast media.

Select all the organs from the following list that are part of the immune system. a. Adenoids b. Appendix c. Bone marrow d. Gallbladder e. Liver f. Thyroid gland

a, b, c The lymphoid organs of the immune system are the adenoids, appendix, and bone marrow. Other organs of the immune system include the lymph nodes, thymus gland, tonsils, and spleen. The gallbladder, liver, and thyroid gland are not part of the immune system.

The immune system ages as people age. Which immunological changes occur with aging? (Select all that apply.) a. Decreased immunoglobulin effectiveness b. Diminished hypersensitivity response c. Increased number of T lymphocytes d. Decreased B lymphocyte function e. Elevated autoantibody synthesis

a, d, e Older adults do have diminished capacity for hypersensitivities; therefore they have fewer allergies and immune reactions than when they were younger. B lymphocyte function is decreased secondary to decreased memory cells. Memory cells are important for permanent immunity from many diseases. Decreased B lymphocyte function places older adults at increased risk for infection. Antibodies are directed against the self. As the immune system ages, the ability to differentiate self from nonself is slowed, which can result in antibodies being produced against the self. Option A is incorrect because older adults produce effective immunoglobulins; however, they produce fewer immunoglobulins (antibodies) than younger adults, which places them at increased risk for infections. Option C is incorrect because older adults produce fewer T lymphocytes, which are important for acquired immunity. This places older adults at increased risk for infections.

The nurse is assessing a 10-year-old client's level of consciousness in the emergency department because the mother feels her child is disoriented and lethargic. Which questions would the nurse want to ask the parent in assessing possible increased intracranial pressure (IICP) as a cause of her confusion and lethargy? (Select all that apply.) a. "Has your child ever had a shunt, brain hemorrhage, or brain surgery?" b. "Does your family have animals at home?" c. "Could your child been ingesting any household products, drugs, or medications?" d. "Has the child had strep throat recently?" e. "Has your child had any recent head trauma, falls, or accidents that you're aware of?"

a. "Has your child ever had a shunt, brain hemorrhage, or brain surgery?" c. "Could your child been ingesting any household products, drugs, or medications?" e. "Has your child had any recent head trauma, falls, or accidents that you're aware of?" Rationale Health history is important when assessing for altered levels of consciousness. Asking about recent or past head trauma, falls, accidents, shunting, brain hemorrhage, or cranial surgery is useful in discerning possible reasons for altered level of consciousness (LOC). Parents should be asked about intentional or accidental ingestion or exposure to drugs or other substances. There is no particular association between a recent strep throat infection and an altered level of consciousness, although other illnesses such as meningitis or viral syndromes might predispose the client to problems with intracranial regulation. Recent animal exposure usually is not related to altered levels of consciousness.

The father of an adolescent client with a recent head injury asks the nurse to explain what the family can expect in the way of treatment, especially medications, for their son. The nurse should respond with: a. "Head injury clients tend to be agitated, so I expect the doctor will keep him sedated to decrease his movement and brain activity." b. "Even though he has fluid and swelling inside his head from the injury, he probably won't be given diuretics as these are used more in cardiac clients." c. "It's likely he that won't be given anti-fever medications since drugs like acetaminophen are contraindicated in head injury." d. "Even though he might be restless, it's probable that he won't be sedated so that we can more accurately monitor his level of consciousness."

a. "Head injury clients tend to be agitated, so I expect the doctor will keep him sedated to decrease his movement and brain activity." Rationale The best response that the nurse can give is to tell the father that individuals with head injury and increased intracranial pressure are typically sedated due to restlessness and agitation. Such behavior raises blood pressure, intracranial pressure, and cerebral metabolism, and can cause further damage to the brain. Thus, head injury clients are not kept awake for the purpose of better monitoring of their level of consciousness. Diuretics are considered mainstays in treating clients with Increased ICP (IICP). Antipyretics are also an important adjunct to treatment when clients develop fever since hyperthermia increases cerebral metabolic rate and exacerbates an increase in ICP.

The nurse is teaching a patient with a new diagnosis of systemic lupus erythematosus (SLE) about her disease. The nurse recognizes that the patient understands the information when making which statement? a. "I need to avoid getting infections because they will increase the immune response in my body, which can make my SLE worse." b. "I need to be sure to take all the available immunizations to keep me from getting sick." c. "Because of my SLE, my immune system is already diminished, so I need to avoid people with the flu." d. "As long as I take all my prescribed medications, I won't have to make any lifestyle changes as a result of my SLE."

a. "I need to avoid getting infections because they will increase the immune response in my body, which can make my SLE worse." SLE is a hyperimmunity problem. Pathogens trigger the immune response in the body, which can exacerbate the SLE. Immunizations trigger the immune response in the body to help create antibodies. In patients with autoimmune diseases such as SLE, immunizations can exacerbate the disease. SLE is not the result of immunosuppression. Lifestyle changes are required with most chronic illnesses such as SLE. Patients cannot depend on medications alone.

The nurse notes that a client has muscle fasciculations of both bicep muscles. What additional information should the nurse assess in this​ client? ​(Select all that​ apply.) a. Body temperature b. Last solid food intake c. Urine output d. List of medications taking e. Blood pressure and pulse

a. Body temperature c. Urine output d. List of medications taking Rationale Fasciculations occur in clients with disease or trauma to the lower motor​ neurons, as a side effect of​ medications, in​ fever, in sodium​ deficiency, and in uremia. Fasciculations are not associated with food intake or blood pressure and pulse measurements.

The nurse establishes a diagnosis of Ineffective Tissue Perfusion: Cerebral for a client with increased intracranial pressure (IICP). Which interventions would not be included in the client's plan of care? a. Cluster nursing care. b. Provide a quiet environment, limiting noxious stimuli. c. Preoxygenate the mechanically-ventilated client with 100% oxygen before suctioning. d. Assess for bladder distention and bowel constipation.

a. Cluster nursing care. Rationale Nursing care should be planned so that certain activities are not clustered together. For example, turning the client, getting the client on the bedpan, and suctioning should not be done within the same time period since multiple procedures can increase ICP. Nursing care should be scheduled to provide rest periods between procedures. Providing a quiet environment, preoxygenating before suctioning, and assessing for bowel and bladder fullness are appropriate for the client with IICP.

The nurse assessing a client with suspected increased intracranial pressure (IICP) should be aware of which physiologic mechanisms related to IICP? (Select all that apply.) a. Interruption of the cerebral blood flow, particularly oxygen and glucose, leads to brain tissue ischemia and disruption of the cerebral metabolism. b. Pressure and metabolic autoregulation mechanisms have an unlimited ability to maintain cerebral blood flow by constricting or dilating cerebral arterioles when ICP changes. c. In pressure autoregulation, high arterial pressure causes stretch receptors in the brain's small blood vessels to dilate to increase cerebral blood flow d. If the volume of the brain, CSF, or blood in the skull increases, the volume of the others must decrease to maintain normal pressures in the cranium. d. Vasodilation refers to the relationship between the volume of the intracranial components and intracranial pressure.

a. Interruption of the cerebral blood flow, particularly oxygen and glucose, leads to brain tissue ischemia and disruption of the cerebral metabolism. d. If the volume of the brain, CSF, or blood in the skull increases, the volume of the others must decrease to maintain normal pressures in the cranium. Rationale Interruption of cerebral blood flow leads to ischemia and disrupted cerebral metabolism. Since the cranial cavity can't expand, and the brain, CSF, and blood fill the entire cranium, if one of these substances increases, the volume of the others must decrease to maintain normal pressures. In pressure autoregulation, if the arterial pressure is high, the stretch receptors in the brain's small blood vessels will cause the vessels to constrict, not dilate. This serves to maintain normal pressure. This ability, however, is limited. The relationship between volume and intracranial pressure is called compliance, not vasodilation.

A client experiences fractures of the left leg and a traumatic brain injury in a dirt bike accident and is admitted to the intensive care unit. Which assessment finding indicates increased intracranial pressure​ (IICP)? a. Irritability b. Oliguria c. Hypotension d. Nausea

a. Irritability Rationale Irritability may indicate that the client is experiencing an increase in intracranial​ pressure, especially if associated with additional signs of​ bradycardia, increased systolic​ pressure, increased pulse​ pressure, vomiting,​ headache, lethargy, and change in mental status. Nausea does not accompany the vomiting associated with increased intracranial pressure. Hypotension and oliguria are not associated with increased intracranial pressure.

The nurse is caring for a patient experiencing an immune response. She assesses the patient for development of a hyperimmune response while knowing that cytotoxic T cells are responsible for which action? a. May kill healthy cells along with foreign antigens. b. Are the most prevalent type of T lymphocyte. c. Can suppress the immune response. d. Diminish dendritic cell function.

a. May kill healthy cells along with foreign antigens Cytotoxic T lymphocytes can kill healthy tissue along with antigens. Suppressor T cells help to keep cytotoxic T cells in check. Helper T cells are the most prevalent type of T lymphocyte, not cytotoxic cells. Cytotoxic T lymphocytes do not suppress the immune response but are a factor in optimal immune functioning. Suppressor T lymphocytes help to suppress the function of cytotoxic cells. Dendritic cell function enhances cytotoxic T lymphocyte functioning.

The ICU nurse reviewing an intraventricular pressure reading of a client under care for increased intracranial pressure notes a value of 20 mmHg. This requires what action? a. Notification of the physician and immediate treatment b. Continued monitoring of the intracranial pressure c. No special action as this value is within normal limits d. Periodic observation for seizure activity

a. Notification of the physician and immediate treatment Rationale The normal range for intracranial pressure (ICP) in adults is typically 5-15 mmHg. Sustained elevated pressure is 15 mmHg in adults, and a result of 20 mmHG warrants immediate intervention and treatment. Taking no action is not an option. The nurse should continue to carefully observe the client, including observing for seizure activity, but immediate intervention for a 20 mmHg reading is the most critical action by the nurse with a reading at this level.

An 18-month-old female patient is diagnosed with her fifth ear infection in the past 10 months. The physician notes that the child's growth rate has decreased from the 60th percentile for height and weight to the 15th percentile over that same time period. The child has been treated for thrush consistently since the third ear infection. The nurse understands that the patient is at risk for which condition? a. Primary immunodeficiency b. Secondary immunodeficiency c. Cancer d. Autoimmunity

a. Primary immunodeficiency Primary immunodeficiency is a situation in which the entire immune defense system is inadequate and the individual is missing some, if not all, of the components necessary for a complete immune response. The child has had (1) four or more ear infections in the past year, (2) failure to grow normally, and (3) persistent thrush. Any two of these issues would cause the nurse to suspect primary immunodeficiency. Secondary immunodeficiency is induced by illness or treatment with immunosuppressive agents such as corticosteroids. Cancer is caused by abnormal cells that will trigger an immune response. Autoimmune diseases are caused by hyperimmunity; there is no indication of hyperimmunity.

A nurse in the intensive care unit is providing care for a client with increased intracranial pressure​ (IICP). The nurse monitors the client for which manifestations of​ IICP? ​ (Select all that​ apply.) a. Projectile vomiting b. Increased heart rate c. Decreased level of consciousness d. Dilated pupils e. Decreased blood pressure

a. Projectile vomiting c. Decreased level of consciousness d. Dilated pupils Rationale Projectile vomiting is a manifestation of increased intracranial pressure. This is caused by pressure on the brainstem from swollen brain tissue. Dilated pupils are a manifestation of increased intracranial pressure. This is caused by pressure on the cranial nerves and vision pathways within the brain. A decreased level of consciousness is a manifestation of increased intracranial pressure. This is caused by pressure on the cerebral cortex and decreased oxygenation of the brain tissues. Increased intracranial pressure causes increased blood​ pressure, especially the systolic blood pressure. This worsens until there is a wide difference between the systolic blood pressure and the diastolic blood pressure. Increased intracranial pressure causes lowered heart rate. This is caused by the​ body's attempt to compensate for increased blood pressure.

A nurse in the intensive care unit is providing care for a client with increased intracranial pressure from a traumatic brain injury. The client has a fever of 102 ​°F. Which interventions will the nurse use to promote normal intracranial​ pressure? ​(Select all that​ apply.) a. Provide supplemental oxygen b. Monitor level of consciousness c. Flex the neck to open the airway d. Suction for no more than 10 seconds per pass e. Administer acetaminophen per order

a. Provide supplemental oxygen b. Monitor level of consciousness d. Suction for no more than 10 seconds per pass e. Administer acetaminophen per order Rationale Hyperthermia increases intracranial pressure. Hyperthermia also affects hypothalamic function in clients with increased intracranial​ pressure; therefore, administering an antipyretic medication is appropriate. Prolonged suctioning can increase intracranial pressure. It also causes decreased oxygen levels. Increased intracranial pressure can cause irregular and ineffective respirations. Supplemental oxygen helps prevent hypoxia. It also helps prevent excess carbon​ dioxide, which is a vasodilator. A decreased level of consciousness can be a manifestation of pressure on the cerebral cortex. It can also be a manifestation of decreased oxygen levels in the brain. Flexing the neck increases intracranial pressure by preventing blood return from the brain. The head and neck must be kept in neutral position.

A pediatric client is admitted for a head injury after falling off of a skateboard. The client was at first combative but is now becoming lethargic. Which actions should the nurse take for this client who has increasing intracranial​ pressure? ​(Select all that​ apply.) a. Raise the side rails b. Install suction equipment at the bedside c. Insert a nasogastric tube d. Pad the side rails e. Elevate the head of the bed 30 degrees

a. Raise the side rails d. Pad the side rails e. Elevate the head of the bed 30 degrees Rationale Nursing actions for the client with increasing intracranial pressure include padding and raising the siderails to prevent injury in the event that the client begins to seize. Elevating the head of the bed will help reduce increasing pressure in the head. Inserting a nasogastric tube would cause unnecessary stimulation and further raise intracranial pressure. Suction equipment would be necessary if the client has a seizure disorder or is experiencing status epilepticus.

The nurse instructs the parents of​ school-age children on ways to prevent head injuries. Which statement made by a participant indicates that additional teaching is​ required? a. ​"My son should wear protective shoulder and knee pads when playing​ football." b. ​"My daughter needs to wear a helmet when riding the​ bicycle." c. ​"Even though he won​'t like​ it, I​'ll make sure my son wears a helmet when​ skateboarding." d. ​"I need to get my son a helmet to wear when ice​ skating."

a. ​"My son should wear protective shoulder and knee pads when playing​ football." Rationale The nurse should teach all clients about ways to prevent head injuries or​ trauma, including wearing helmets when​ bicycling, skateboarding, and skating and using​ sports-specific helmets and equipment when participating in contact sports. The son needs to wear a helmet in addition to shoulder and knee pads when playing football. The other participant comments indicate that teaching has been effective.

Which statement made by a student nurse indicates the need for further teaching about pituitary insufficiency? a. "Synthetic human growth hormone may be prescribed for children who are small for gestational age." b. "Testosterone supplements may be prescribed for women with gonadotropin deficiency." c. "Estrogen is known to regulate the action of growth hormone in men and women." d. "Chronic kidney disease treatment may include synthetic growth hormone replacement."

b. "Testosterone supplements may be prescribed for women with gonadotropin deficiency." Synthetic human growth hormone (HGH) is used for growth hormone deficiencies caused by pituitary insufficiency, as well as other conditions such as Turner's syndrome, chronic kidney disease, and children small for gestation age. Testosterone is used as supplement for men with gonadotropin deficiency. Estrogen and progesterone supplements, also referred to as hormone replacement therapy (HRT), is indicated for women with gonadotropin deficiency and for the relief of post-menopausal symptoms. Estrogen is also known to regulate secretion and action of GH in men and women.

The nurse admits for observation a 70-year-old client with symptoms of increased intracranial pressure (IICP). What diagnostic tests would the nurse not expect to be performed on this elderly client and why? a. Cranial nerve testing because his age makes it difficult to assess cranial nerve function. b. A lumbar puncture to avoid a sudden release of pressure in the skull which might cause cerebral herniation. c. Computed tomography (CT) scan because of its limited value in determining potential causes of this client's abnormal neurological findings. d. Endoscopy to search for stress gastritis and ulcers which occur more often in this population.

b. A lumbar puncture to avoid a sudden release of pressure in the skull which might cause cerebral herniation. Rationale Clients of any age with suspected increased intracranial pressure (IICP) would not be given a lumbar puncture because of the risk of cerebral herniation. Cranial nerve testing, as part of an overall neurological assessment, would be done in all age groups and adapted accordingly. Computed tomography (CT) scans, as well as MRIs, are typical diagnostic tests used to help determine causes of IICP. An endoscopy would not be done as part of the IICP assessment. IICP places individuals at higher risk for developing stress gastritis and ulcers, not the other way around.

Radioactive iodine is indicated for the treatment of hyperthyroidism. The nurse should include which teaching in this patient's plan of care? a. Isolation is required for 6-8 weeks b. An additional dose may be needed c. Thyroid replacement therapy is prescribed d. An overnight hospital stay is required

b. An additional dose may be needed Radioactive iodine (RAI) is indicated for the treatment of hyperthyroidism. It is given as an oral preparation, usually as a single dose on an outpatient basis. The radioactive iodine makes its way to the thyroid gland where it destroys some of the cells that produce thyroid hormone. The RAI is completely eliminated from the body after about 4 weeks. The extent of thyroid cell destruction is variable, thus the patient has ongoing monitoring of thyroid function. If thyroid production remains too high a second dose may be needed. The goal of this procedure is to destroy thyroid hormone producing cells; additional thyroid hormone is not prescribed.

The parents of a newborn question the nurse about the need for vaccinations: "Why does our baby need all those shots? He's so small, and they have to cause him pain." The nurse can explain to the parents that which of the following are true about vaccinations? (Select all that apply.) a. Are only required for infants b. Are part of primary prevention for system disorders c. Prevent the child from getting childhood diseases d. Help protect individuals and communities e. Are risk free f. Are recommended by the Centers for Disease Control and Prevention (CDC)

b. Are part of primary prevention for system disorders d. Help protect individuals and communities f. Are recommended by the Centers for Disease Control and Prevention (CDC)

A client with a traumatic brain injury is intubated and placed on mechanical ventilation. What should the nurse use to evaluate the effectiveness of these respiratory​ interventions? a. Motor and sensory function b. Arterial blood gas results c. Cranial nerve function d. Glasgow Coma score

b. Arterial blood gas results Rationale A client with an alteration in LOC may be unable to maintain an open airway and engage in spontaneous respirations. The client may need endotracheal intubation or mechanical ventilation. Arterial blood gases are used to guide the effectiveness of ventilation. Glasgow Coma​ score, cranial nerve​ function, and motor and sensory function are used to determine the effect of intracranial pressure on body functioning.

An older female client is residing in the nursing home. As the nurse walks into the room to assist the client for evening​ care, the client calls​ out, wondering who it is. The nurse introduces herself and enters the room. The client is wearing very thick glasses and has visual deficits. After the nurse has finished her​ care, what interventions does the nurse do to improve the safety of the​ client? ​(Select all that​ apply.) a. Moves chair beside the bed so the client has a place to sit if she gets up in the night Turns off the night light to improve sleep quality b. Asks the client where she would like her glasses placed c. Encourages the client to call for any needs d. Puts the call bell within reach of the client

b. Asks the client where she would like her glasses placed c. Encourages the client to call for any needs d. Puts the call bell within reach of the client Rationale This client in this scenario is consistent with a client who has a visual deficit. Putting the call light beside the client is an appropriate intervention for this client so it is within easy reach. Turning the night light off is not an appropriate intervention for someone with a visual deficit. Areas should be well lit. Turning off the night light increases the risk of the client not being able to see when she gets up. Moving the chair beside the bed is not an appropriate intervention for this client. Clients with visual deficits require a​ clutter-free environment. Furniture should not be moved without notifying the client. Asking the client where she would like her glasses placed is an appropriate intervention. Orienting the client to her surroundings improves her independence with her vision deficit. Encouraging the client to call if she needs help is always an appropriate nursing intervention.

The nurse is providing teaching to a client who is recovering from increased intracranial pressure (IICP). Which instructions should the nurse plan to give the client prior to discharge? (Select all that apply.) a. Eat a diet high in red meat and chicken. b. Avoid straining with bowel movements. c. Perform isometric or muscle-contracting exercises. d. There is no need to worry about keeping the head and neck in alignment when turning in bed. e. Avoid exposure to people with colds.

b. Avoid straining with bowel movements. e. Avoid exposure to people with colds. Rationale Straining with bowel movements and performing isometric exercises are to be avoided as these actions increase intracranial pressure. The client should be taught to turn in bed while maintaining the head and neck in alignment. Because coughing, sneezing, and nose blowing can all increase ICP, the client should avoid others with respiratory illness. Diet does not have a great deal of effect on ICP, but red meat is considered an unhealthy choice if eaten frequently.

During a physical​ examination, the nurse assesses the reflexes of an older client. Which reflex would require notification to the healthcare​ provider? a. Gag b. Babinski c. Corneal d. Achilles

b. Babinski Rationale The presence of the Babinski reflex from age 2 years and on indicates cerebral damage. This is the reflex that the nurse should report to the healthcare provider. The​ gag, corneal, and Achilles reflexes should be present in the older client.

A nurse in the emergency department is providing care for a client who has increased intracranial pressure​ (IICP) from a traumatic brain injury from a motor vehicle crash. The nurse anticipates orders for which diagnostic tests in the care of this​ client? ​ (Select all that​ apply.) a. Electromyogram b. Cardiac monitoring c. Intracranial pressure monitor d. ABGs e. CT of the head

b. Cardiac monitoring c. Intracranial pressure monitor d. ABGs e. CT of the head Rationale An intracranial pressure monitor will give information about intracranial pressure. This information can be used to manage the medications and fluids for this client. A CT of the head will give information about possible hemorrhage and diffuse axonal injuries. Cardiac monitoring would be essential to monitor cardiac rate and rhythm. Arterial blood gases give information about oxygen and carbon dioxide levels in the blood. This information is used to manage artificial airways and mechanical ventilation. Electromyography is used to measure skeletal muscle activity. It would not be used in the diagnosis of a client with traumatic brain injury.

Which nursing action is a priority when intervening with a client having a seizure? a. Apply physical restraints to protect the client from injury. b. Turn the client on the side to prevent aspiration. c. Use a padded tongue blade to open the client's airway. d. Elevate the head of the bed to promote ventilation.

b. Turn the client on the side to prevent aspiration. Rationale The priority nursing action is to turn the client on the side to prevent aspiration from secretions that can occur when a client is having a seizure. Applying physical restraints, using a padded tongue blade, and elevating the head of the bed can cause further physical injury to the client.

A young adult client is brought to the emergency room with a suspected narcotic overdose. The client is in respiratory acidosis and has an increased CO2 level with resulting increased intracranial pressure (IICP). The physiologic mechanism contributing to the IICP is: a. herniation of the brainstem. b. vasodilation of cerebral vessels. c. altered level of consciousness. d. vasoconstriction of cerebral vessels.

b. vasodilation of cerebral vessels. Rationale A narcotic overdose depresses the respiratory system and leads to elevated levels of carbon dioxide (CO2) in the blood and respiratory acidosis. High levels of CO2 cause vasodilation of cerebral vessels leading to increased intracranial pressure (IICP). Elevated ICP is not caused by vasoconstriction, and herniation occurs as an end result of delayed or inadequately treated IICP. Altered level of consciousness is a sign of IICP rather than the physiologic mechanism contributing to IICP.

A nursing instructor is reviewing sensory perception with the nursing students. The nursing instructor knows that the students have appropriately learned the information when he hears them make what comments regarding the sensory​ process? ​(Select all that​ apply.) a. ​"A receptor is not always necessary to process a stimulus to the​ brain." b. ​"The feeling of my stomach being full after a large meal is an example of a visceral​ sensation." c. ​"If a person can perceive stimuli in the​ environment, and​ respond, that person is exhibiting​ awareness." d. ​"When I am feeling around my purse with my hand trying to find my​ keys, I am using the process of​ stereognosis." e. ​"Kinesthetic is the sensation of​ touch."

b. ​"The feeling of my stomach being full after a large meal is an example of a visceral​ sensation." c. ​"If a person can perceive stimuli in the​ environment, and​ respond, that person is exhibiting​ awareness." d. ​"When I am feeling around my purse with my hand trying to find my​ keys, I am using the process of​ stereognosis." Rationale Awareness is the ability of an individual to perceive their​ environment, and to react appropriately. A receptor is one of the four aspects necessary for the sensory process. For an individual to process​ information, there must be a​ stimulus, a​ receptor, an impulse conduction and perception. Kinesthetic is the awareness of the position and movement of the body and its parts. Stereognosis is the ability to understand an object through touch. It is used when individuals feel for an object with their hand. A visceral feeling is related to an organ. Feeling full after a large meal is an example of a visceral feeling.

A patient with increased ICP is being monitored in the intensive care unit (ICU) with a fiberoptic catheter. Which order is a priority for you? A. Perform hourly neurologic checks. B. Take a complete set of vital signs. C. Administer the prescribed mannitol (Osmitrol). D. Give an H2-receptor blocker.

c he priority is to treat the known existing problem, and mannitol is the only thing that can do that. Because the patient is having the current pressure measured with objective numbers, treating the known problem is a priority over additional assessments. H2-blockers are given when corticosteroids are administered to help prevent gastrointestinal bleeding, but they are not a priority compared with the treatment of ICP.

The mother of a 10-year old client asks the nurse, "Why does my son seem so dazed after he has a seizure?" How should the nurse respond? a. "The grogginess after a seizure occurs from decreased oxygen getting to the brain during the seizure causing a temporary brain injury." b. "Children's brains are less resistant to the effects of seizure activity so they tend to have more confusion afterwards." c. "Seizure activity exhausts energy production in brain cells and may be producing molecules that negatively affect brain function until balance is restored." d. "He's likely just confused about what's happened to him during the seizure and just needs time to recover from the episode."

c. "Seizure activity exhausts energy production in brain cells and may be producing molecules that negatively affect brain function until balance is restored." Rationale The spontaneous and disordered discharge of activity occurring during a seizure is thought to exhaust energy metabolites or to produce locally toxic molecules. This affects level of consciousness following a seizure episode until metabolic balance is restored.

While caring for a patient preparing for a kidney transplant, the nurse knows that the patient understands teaching on immunosuppression when she makes which statement? a. "My body will treat the new kidney like my original kidney." b. "I will have to make sure that I avoid being around people." c. "The medications that I take will help prevent my body from attacking my new kidney." d. "My body will only have a problem with my new kidney if the donor is not directly related to me."

c. "The medications that I take will help prevent my body from attacking my new kidney." Immunosuppressive agents given after an organ transplant prevent the body from having a delayed hypersensitivity reaction (type IV). The body will recognize the kidney as foreign and does not treat like one's own kidney. The patient will have to avoid being around people with infection, not necessarily all people. The body may have a problem with the kidney even if the donor is a direct relative, although the chances of histological matches are improved among blood relatives.

Immune components have specific functions. Which immune component provides specific protection against bacterial and viral pathogens? a. Cytotoxic T lymphocytes b. Serum complement c. B cell immunoglobulins d. Phagocytic macrophages

c. B cell immunoglobulins Immunoglobulins produced by B cells provide humoral immunity and are the primary defense against bacterial and viral infections. Option A is incorrect because T lymphocytes are part of cellular or cell-mediated immunity. Cytotoxic T lymphocytes attack and kill antigens directly but are not the primary defense mechanism against viral and bacterial infections. Option B is incorrect because the serum complement system contributes to the immune response by amplifying and increasing the systems efficiency and efficacy, but the system is not the primary defense mechanism against viral and bacterial infections. Option D is incorrect because phagocytic macrophages are part of innate immunity and the first line of defense during an immune reaction. They are not, however, the primary defense mechanism against viral and bacterial infections.

The nurse is caring for the client with increased intracranial pressure (IICP) from a severe head injury. The nurse monitors the client for symptoms of which complication of IICP? a. Hydrocephalus b. Headache c. Brain herniation d. Cerebral edema

c. Brain herniation Rationale The client with increased intracranial pressure (IICP) is at risk for brain herniation. Cerebral edema and hydrocephalus are causes of IICP. Headache is a symptom rather than a complication of IICP.

The nurse is caring for a patient who has undergone a thyroidectomy. Which patient complaint is highest priority requiring further evaluation? a. Pain at surgical site b. Thirst c. Hoarseness d. Nausea

c. Hoarseness Thyroidectomy involves a surgical incision in the anterior neck. Hoarseness may be a sign of airway edema. A patent airway is always a priority of care for any post-operative patient. General anesthesia is used for this surgery requiring insertion of an artificial airway, therefore throat irritation and thirst is expected. Nausea may be a side effect from anesthesia. Pain is expected at the surgical site.

When the nurse is assessing a client with suspected increased intracranial pressure, the nurse is aware that which of the following is true in clients with increased intracranial pressure? a. In pressure autoregulation, stretch receptors within small blood vessels of the brain cause smooth muscle of the arterioles to dilate. b. Autoregulatory mechanisms have a great ability to maintain cerebral blood flow. c. Interruption of the cerebral blood flow leads to ischemia and disruption of the cerebral metabolism. d. The relationship between the volume of the intracranial components and intracranial pressure is known as vasodilation.

c. Interruption of the cerebral blood flow leads to ischemia and disruption of the cerebral metabolism. Rationale Interruption of the cerebral blood flow leads to ischemia and disruption of the cerebral metabolism. In pressure autoregulation, the arterioles will contract. Autoregulatory mechanisms have a limited ability to maintain cerebral blood flow. The relationship between volume and intracranial pressure is called compliance.

Which nursing action is a priority when suctioning a client with increased intracranial pressure? a. Suction the client as needed. b. Suction the client every hour. c. Limit suction passes to 10 seconds. d. Schedule suctioning with other nursing care.

c. Limit suction passes to 10 seconds. Rationale The nurse must perform the suctioning gently, limiting suction passes to 10 seconds to prevent further increase in ICP. Suctioning can cause an increase in intracranial pressure and should be used cautiously. The nurse must preoxygenate the client with 100% oxygen before suctioning and closely assess the client during suctioning. Suctioning helps maintain airway patency and prevent hypoxemia.

The nurse is admitting a 65-year-old client for observation who has symptoms of cognitive dysfunction, gait disruptions, and urinary incontinence. The nurse expects the client to undergo testing for which of the following? a. Renal failure b. Cardiac malfunction c. Normal pressure hydrocephalus d. Gastrointestinal disturbances

c. Normal pressure hydrocephalus Rationale Normal pressure hydrocephalus is most commonly seen in older adults. This may result from trauma, complications from surgery or an unknown cause. Due to the slow, insidious onset, the client may be evaluated for dementia. The symptoms listed are not those of renal failure, cardiac malfunction, or gastrointestinal disorders.

Which statement accurately describes the immunologic mechanism of a hypersensitivity reaction? a. Type 1 reactions are caused by hypersensitivity to immunoglobulin E (IgE). b. Poison ivy is an immune-complex mediated reaction. c. Type II reactions are immediate tissue-specific reactions. d. Allergic rhinitis is an IgM-mediated hypersensitivity.

c. Type II reactions are immediate tissue-specific reactions. Option A is incorrect because type 1 reactions are caused by a hypersensitivity to a specific allergen; type 1 reactions are moderated by IgE but are not a hypersensitivity to IgE itself. Option B is incorrect because poison ivy is a type IV, cell-mediated reaction, which is a delayed reaction. Option D is incorrect because allergic rhinitis is mediated by IgE.

The nurse is assessing a client who leads an​ active, healthy lifestyle. The client has no history of chronic health​ conditions, but is seeking health care due to visual changes and occasional headaches over the past few weeks. Upon​ assessment, which question should the nurse ask the​ client? a. "Are you having trouble moving your ​bowels?" b. "Do you feel nauseated after ​eating?close double quote" c. "Have you noticed an increase in ​thirst? d. "Are your headaches worse upon rising in the ​morning?"

d. "Are your headaches worse upon rising in the ​morning?" Rationale With increased intracranial​ pressure, headaches are noted to be worse in the morning and with position changes. Projectile vomiting may​ occur, but nausea is not present. Thirst does not increase for clients with IICP. Clients with IICP typically do not experience constipation or trouble with bowel movements.

In conducting a Glasgow Coma Scale Assessment on an infant, the nurse understands that the observation made to assess motor response in this age group differs in what way from the observation used in assessing motor response in an adult? a. The eyes open spontaneously in an infant, but only to noise in adults. b. Response to pain is not part of the assessment of the infant's motor response, while it is assessed in adults. c. The degree of flexion is assessed in infants, but not in adults. d. Infants are observed for spontaneous movement, while adults are asked to follow a command.

d. Infants are observed for spontaneous movement, while adults are asked to follow a command. Rationale Motor response in infants using the Glasgow Coma Scale is assessed by observing spontaneous movement while adults' ability to obey a command to move a body part is used with older children and adults. Observing for spontaneous eye opening is done with both infants and adults, but this assessment is in the category of "Eye opening" rather than "Motor response." Response to pain and degree of flexion are assessments of motor responses conducted on both infants and adults.


Conjuntos de estudio relacionados

The 30 Most Famous Landmarks in the World (3 Taj Mahal: Agra, India)

View Set

CHAPTER 30: LIABILITY OF PRINCIPALS, AGENTS, AND INDEPENDENT CONTRACTORS

View Set

Performing Oral Hygiene for an Unconscious Patient

View Set

Pharmacology test 2: units 3&4: chapters (18,19,20,21)(22,23,24,25,26,27,29)

View Set

Chapter 14 Cost Allocation, Profitability

View Set